Sei sulla pagina 1di 269

SSENTfA

Michael D. Ho, M.D


A C

Anesthesidldgy Consultants
The Essential Oral
Board Review

Anesthesiology
Consultants
7710 Beechnut, Suite 206
Houston, Texas 77074
(800) 772-1242 Office
(713)772-7716 Fax
anconsultants.com
The lisscnlial Oral Hoard Review

Introduction to the Consultant Approach

Unlike the written Boards, the oral Board examination in anesthesiology is not about rote
memorization, recall of simple facts, or multiple choice guess work. It is about becoming a
perioperative Consultant in anesthesiology. How is a Consultant defined? If you asked 100
anesthesiologists, you would probably obtain 100 different answers, each the result of differences
in training, practice settings, and personal expectations. But the American Board of
Anesthesiology must be more precise in defining Consultant as the standard by which to evaluate
its candidates. It is those criteria that we describe here.

Instead of providing a comprehensive review of all aspects of perioperative Consultation,


this workbook introduces the essential attributes of a Consultant that pertain to the oral Board
exam. The concepts presented are not meant to represent dictum or dogma. Instead, they are
simply suggestions, guidelines, and thought provoking strategies on how to best acquire and
refine Consultant skills. It is our hope that after taking our course, reading the text, and using this
workbook, you will agree that studying to become a Consultant is not only the best way to
prepare for the exam - it is the best way to practice anesthesiology.

What is a Consultant?

According to Webster's Dictionary, a Consultant is someone who gives expert or


professional advice. If it were all that simple, we would all easily qualify as Consultants.
However, according to ABA standards, a Consultant is someone who possesses the essential
attributes of Knowledge, Judgment, Adaptability, clinical skills, technical facilities, and personal
characteristics to carry out the entire scope of anesthesia practice. More specifically, the ABA
states that a Board certified anesthesiologist must be able to:

1. Communicate effectively with peers, patients, their families, and others in the medical
community.
2. Serve as an expert in matters related to anesthesiology.
3. Deliberate with others.
4. Provide advice and defend opinions in all aspects of the specialty.
5. Function as the leader of the anesthesia care team.

For the purposes of Board exam preparation, we summarize the essential attributes of a
Consultant, as described by the ABA, as follows:

1. Knowledge
2. Judgment
3. Adaptability
4. Communication

These four qualities have been given special emphasis because within each candidate
they can be identified, evaluated, enhanced, and perfected. Other ABA goals such as technical
facilities, leadership, and personal characteristics, although certainly no less important, are more
difficult to test, teach, and acquire. They do not receive the same amount of emphasis (even
though they often develop as a consequence of mastering the other four). Knowledge,
Judgment, Adaptability, and Communication remain the centerpiece of course, and should, from
now until the time you pass, be the focus of your preparation.
The Essential Oral Board Review

Knowledge

It is often said that the factual information necessary to pass the oral exam is the same
as that for the written exam. Such Knowledge includes an understanding of not only general
anesthesia, but various subspecialties, including cardiology, pulmonology, endocrinology,
neurology, gastroenterology, urology, pediatrics, obstetrics, gynecology, psychiatry, pain,
pharmacology, statistics, and surgery. In either test, demonstration of proficiency with these
topics should require a thorough understanding of both breadth and depth. However, because
the written test is multiple choice, it is still possible to pass the writtens by rote memorization,
pattern recognition, and last minute preparation.

The level of Knowledge needed for the orals is much greater. First, the type of
information covered is not of the time-tested, incontrovertible nature used for the writtens. For the
orals, it tends to be more recent, controversial, and polemical - the type that generates spirited
discussion. Second, oral Board Knowledge must not simply be regurgitated, it must be filtered,
proccessed, edited, and finally used selectively. Third, instead of being used to answer multiple
choice questions, Knowledge on the orals is used to justify, defend, rationalize, persuade, and
(most importantly) teach - the things a Consultant is supposed to do. Finally, there is little time to
muse or muddle over its use in an oral exam. Knowledge must be accurately used at a moment's
notice - as such, mastery of oral Board Knowledge must be complete and unequivocal. On the
writtens, superficial understanding or imprecise application of Knowledge may only result in
getting one question wrong. On the orals, it can undermine and destroy an entire case.

Judgment
Judgment is the rationale used to make decisions. Whether you realize it or not, every
time you order a lab, administer a drug, choose a course of action, or even choose not to act, you
are making a Judgment decision. It is the logic behind your decisions that defines Judgment.
There are many instances when the need for Judgment arises. One occurs preoperatively, when
it must be determined whether enough information is known about a patient and whether the case
should be allowed to proceed. Another occurs intraoperatively, when decisions about monitoring,
induction, and maintenance of anesthesia must be made. Yet another occurs when conflicting
goals and priorities need to be weighed before committing to an important clinical choice. The
essence of making Judgment is, therefore, to scutinize perioperative goals, priorities, pros/cons,
and risk/benefits.

Unlike the writtens, there are usually no absolutely right or wrong courses of action in the
orals - no decision that is absolutely best or worst. Recognizing the myriad approaches to
patient care, examiners are often quite lenient in allowing candidates to choose their own courses
of action. They are, however, much more discriminating in learning why that course of action was
taken. For example, no examiner really cares that you chose thiopental, succinylcholine, and
cricoid pressure for induction. But every examiner cares why you chose to perform a rapid
sequence induction in a hypertensive patient with an unruptured intracranial aneurysm.
Therefore, unless your anesthetic plan is clearly dangerous or deviates from accepted standards
of care, it is the reasons behind your decisions - the Judgments - that determine whether you
pass or fail.

Adaptability

Adaptability is the ability to adjust and respond to changing clinical conditions. Clinical
conditions can predictably change numerous times during an oral exam: whenever there is a
complication, a failed technique, an item or drug that suddenly becomes unavailable. It changes
every time the candidate is asked to explain something to a patient or surgeon, a new topic of

W
The Essential Oral Board Review

discussion appears, or a transition occurs from stem question to additional topic. It also occurs
when the examiners switch roles, or an individual examiner abruptly switches his/her tone or
demeanor (sometimes for the worse).

In general, adapting to predictable changes, especially complications deemed likely


based on the stem question, are doable and fair to ask (e.g., fever in a patient who is MH-
susceptible, wheezing in a patiejnt with asthma, bleeding during liver transplantation). But is it
fair to ask about complications that are unpredictable? Actually, with enough experience, little
should be unpredictable. The purpose of preparation is to precisely anticipate any and all
possibilties. An unexpected lost airway can be managed with the ASA Difficult Airway Algorithm.
A sudden bout of VT can be treated using the ACLS protocols. Hypoxia, seizures, inability to
ventilate, altered mental status, bradycardia, patient noncompliance, an irate surgeon - none may
be likely based on the stem question, but all are easily managed by a Consultant
anesthesiologist.

Adapting to changing different (adverse) examination conditions is, perhaps, the greatest
challenge facing candidates. Ideally, oral Board examiners should be neutral, providing neither
positive nor negative feedback. In reality, examiners have unique personalities and examining
styles, ranging from soft-spoken and supportive to confrontational and severe. After all is said
and done, on the day of the exam, every candidate has one final task: to be poised and
professional, regardless of examiner style. This ultimate Adaptibility takes practice. There is, of
course, no separate grade for "Adaptation to Examiner Style", but failure to do so often becomes
painfully apparent to both candidate and examiner.

Communication

Good Communication should be the simplest, most basic essential Consultant attribute to
acquire. Yet for many, it turns out to be the most arduous and insurmountable. Almost everyone
is capable of communicating - about the right topics, under the right conditions. The problem
with the oral exam is that, due to the nature of our practice, we become superb at doing, but
considerably less superb at talking. Like giving directions to home or work, we may travel the
route every day, but, when asked, sometimes forget the names of streets on which to turn.

According to the ABA, the level of communication expected of candidates during the
exam is not unlike that used in everyday discussion with physicians, patients, and family
members. It has been compared to the sort of explanation one would offer to a medical student -
clear, concise, confident, organized, timely, polite, and professional. Besides unfamiliarity with
speaking, what are the other reasons we struggle to meet the ABA's standards? Despite self-
proclaimed readiness, many candidates are still not ready, harboring significant deficiencies in
Knowledge, Judgment, and Adaptability. Others, especially foreign medical graduates, lack
fundamental language skills, both receptive and expressive. And no matter how confident a
candidate claims to be, everybody is affected, at least to some degree, by the artificial setting of
the exam: two expressionless examiners in a dimly lit hotel room asking obscure, unusual, often
unrelated questions in rapid succession. Candidates are told to speak only when spoken to,
restricted in what can be done, frequently cut-off, and presented with rare complications
supposedly of their own doing. It is hardly the setting we experience on a daily basis, no matter
how stressful our practice.

The solution to the problem of inadequate Communication is easy to state but hard to
achieve: practice, practice, practice! Too often, candidates devote the majority of their time and
energy to accumulating Knowledge. They overlook the fact that, on test day, such Knowledge
must be used to form an opinion - and that opinion must be clearly communicated. Some
candidates erroneously assume that if they just knew the information, Communication would
come naturally. Others believe that their Communication is just so bad, change is impossible, so
they do not even try. Neither view, of course, is valid. All change is possible, but one must
The Essential Oral Board Review

recognize the problem, establish a goal, and follow a plan. Painful and difficult though it may
seem, the plan is to take (and carefully review) as many mock oral exams as you can.

v
Interdependence of Attributes
It should be obvious that Knowledge, Judgment, Adaptability, and Communication do not
exist in isolation - they work together. For instance, it is impossible to make a legitimate
Judgment about a controversial topic without first referencing the associated body of Knowledge.
Similarly, Adaptation is only possible after weighing accepted possibilities and then making an
educated decision, i.e., Knowledge and Judgment. Of course, it does not matter what you know if
you cannot to describe it to others. Knowledge, Judgment, and Adaptability all required clear
Communication to be effective. If you want your viewpoint to be heard, you must first make sure
your words are understood.

The interdependence of the four major attributes has special implications for preparation.
Every problem on the oral Board, no matter how complicated, is based on a real case. As such,
its complications, controversies, and dilemmas have likely been previously discussed and
resolved in the past. If one searched diligently, all pertienent Judgments and Adaptations could,
perhaps in piecemeal, be learned. Similarly, organization of ideas, choice of words, and
presentation style can all be taught and committed to memory. Thus, Judgment, Adpatability,
and Communication can all be classified under a broader category - Knowledge. This
suggestion is reassuring because it means that even cases one has never seen or done can be
learned well enough to discuss as a Consultant. It also makes true the claim that the most
common reason for failing the orals is inadequate Knowledge. Knowledge, here, is defined as
not only the facts, but how to make Judgments, Adapt, and Communicate your concerns.

A senior Board examiner said it best when he once defined a Consultant as "the one
everyone turns to when no one else has the answer". Based on your Knowledge, Judgment,
Adaptability, and Communication, would you consider yourself a Consultant?

/"US
The Essential Oral Board Review

Essential Consultant Attributes

The following attributes are specific traits and skills that senior Board examiners have sought
as evidence of Consultant abilities. All tend to be variations of Knowledge, Judgment, Adaptability,
and Communication as discussed above.

1. Emphasize Breadth and Depth


a. Have a command of both general and detailed information, emphasizing both breadth and
depth. The requisite information covers not only anesthesia, but includes general medicine
and specialty topics as well. Such knowledge may, at times, seem trivial, but it is actually
essential in establishing your position as an expert in the field. Superficial understanding is not
enough.
b. Expect examiners to probe deeply several times during an exam. No matter how benign the
process, some will undoubtedly find it mean or picky. Unfortunately, asking pointed questions
is the only way to accurately determine how much you know.

2. Justify All Actions and Decision with the Application of Knowledge


a. Support everything you say and do with accurate, up-to-date information. This means
keeping up with current developments in the field. Your plans must be based on accepted
standards, not anecdote. (You should never use as justification the phrases "because that's
the way I was trained" or "that's the way we do it at my hospital.")
b. When an issue is controversial or incompletely understood, you must recognize it as such, but
still have a plan.

3. Demonstrate Sound Judgment


a. Judgment is the basis for decision-making, and usually involves concerns such as goals,
priorities, risks/benefits.
b. Judgment may come from experience, but since we cannot experience everything, it must also
draw from other sources, such as study, discussion, inquiry, analysis, and reflection. This
realization blurs the distinction between Judgment and Knowledge.
c. Part of exercising good Judgment is being reasonable. There is usually no absolutely right,
wrong, best, or worst way to do things - just a multitude of possible techniques and
approaches, most of which may be acceptable, depending on the Judgments employed.
d. During an exam, you may claim you would refrain from using "X" drug, device, or technique
because of your unfamiliarity with "X." This response may actually reflect good Judgment. It
does not, however, relieve you of the responsibility of knowing how "X" works and how it
should be applied, especially if "X" is an essential topic.

4. Be Adaptable and Flexible


a. Recognizing the variety of available choices should not leave you immobilized. Based on
available case information and application of Knowledge and Judgment, you must have
already chosen, or be able to quickly choose, a preferred plan A. If plan A fails or becomes
impossible/unwise to implement, your plans B and C should be readily available.
b. When conditions change, you must demonstrate Adaptability by responding rapidly and
appropriately. Changing conditions may appear as an alteration in either the case or tone of
the exam.
c. Sometimes when challenged, you will have to either proceed with your current plan or change
your mind. The underlying concern expressed by all is, "Are they trying to test my flexibility or
my willingness to stand firm?" The question is a bit misguided, since what you should do
depends on the situation at hand. Sometimes that means altering your thinking, other times it
means standing your ground. The best generic advice that can be given is to do whatever is
best for the patient, given what you know about the case (see item 6 below).
The Essential Oral Board Review

5. Practice with Foresight and Anticipation


a. Anticipate and expect complications. Any case, no matter how simple or complex, can have
an adverse outcome. Get in the daily habit of recognizing complex cases and identifying the
likely complications.
b. Try to prevent complications before they occur.
c. Be prepared to treat complications expeditiously at all times.

6. Recognize Ambiguity When It Exists


a. Ambiguous clinical situations in the exam should not to be ignored or unappreciated; their
presence actually tests your ability to exercise Judgment. When the scenario is unclear due to
inadequate information, you must decide whether such information is necessary before
proceeding. If it were necessary, it would be poor Judgment to proceed. Conversely, if it were
unnecessary, it would be poor Judgment to delay.
b. Another result of inadequate information is the creation of false assumptions. If the scenario is
unclear, you cannot assume it is either critical or benign. For example, consider BP values
made available to you without revealing the patient's baseline values or the presence or
absence of symptoms. Without the necessary clinical context, conclusions about their
significance are unfounded.
c. Ambiguity is best resolved by using contingency statements such as "It depends" or "If...then."
When faced with an ambiguous situation, for which the resolution of the ambiguity is important,
you should neither refuse to continue nor blindly proceed. Instead, clarify the ambiguity by
using contingency statements and state your plans for either scenario. For example, when
asked about how you would induce an obese patient whose airway you haven't examined, you
may say "If the patient had a difficult airway, I would perform an awake fiberoptic intubation.
But if his airway did not seem difficult, I would perform a rapid sequence induction."
d. This is better than responding, "Well what does the airway look like?" Never answer a
question with a question. (The only exception to that rule is "Could you please repeat the
question?")

7. Identify Priorities and Inherent Conflicts in Difficult Situations


a. Complex cases often contain conflicting goals. As a Consultant, one of your most unique
abilities is to be able to decide which ones absolutely must be resolved, and which can be
ones deferred. If several goals are important but their solutions in conflict, they must be
prioritized and plans created to address the most important one(s).
b. Sometimes, it is not enough just to say what you would do. You may wish to state clearly, as
part of your response (and without always having to be asked), your understanding of any
conflicts, how you prioritized these conflicts, and reached solutions

8. Communicate Effectively
a. No response is adequate unless it is communicated effectively. Clarity is more important than
eloquence. If you have difficulty being formal, be conversational. It is better to appear as
though you're thinking on your feet than reading from a script.
b. Good Communication means answering the question actually asked, not the one you think
they asked. Learn to listen to the question carefully.
c. Good Communication is also timely. It abhors distracting pauses and long delays, which
destroy credibility. It forgives interruptions by Examiners, and ceases entirely when the
Examiner begins to speak.
d. Good Communication is devoid of annoying colloquialisms, abbreviations and phrases (like "I
bagged the patient" or "pent, sux, tube"). Avoid "uhm"s.
e. Given the choice of being more talkative or more reticent, talk more! Remember, as a
Consultant, you are teaching.

9. Demonstrate Appropriate Confidence and Composure


a. Be authoritative and firm, but not aggressive, hostile, or cocky.
b. Be pleasant and courteous, but not obsequious or too eager to please.
c. Eye contact conveys confidence; displaying social graces makes people treat you graciously.
The Essential Oral Board Review

10. Behave as a Professional


a. Behave during the exam exactly as you would during an interview. In a sense, that's what
the exam is. Remember that you are trying to earn people's respect, and examiners tend to
favor the conservative over the liberal.
b. Examiners pass candidates they feel can serve as effective Diplomats of the Board. When
they see you for the first time, they are asking, "Based on my interaction with this candidate,
can he/she adequately represent the best of our profession?"
The Essential Oral Board Review

Types of Questions

The number and types of questions that may be asked during the oral exam are theoretically
infinite. However, in practice, many questions are used repeatedly and patterns do exist.
Uncovering them not only reveals the issues that typically face a perioperative Consultant; it
suggests how to better study. As you read the list, note the considerable overlap of the essential
attributes they assess (Knowledge, Judgment, Adaptability, and Communication.).

1. Factual Questions
Straightforward written Board-type questions generally focus on Knowledge. No Judgment or
Adaptability are required. Usually, one cannot cover up inadequate Knowledge with finesse.
Either you know the answer or you do not. Guessing is not recommended. Fortunately or not,
such questions are, in theory, the least important on the oral exam because the oral exam is not a
test of fact; it is a test of application of fact. Nevertheless, it reflects poorly on the candidate if s/he
does not know the equation for PVR.

Examples
• What is the MAC of desflurane?
• What is the mechanism of thioptenal?
• What is the equation for PVR?
• Draw a Mapleson D circuit.
What is the pathophysiology of hepatorenal syndrome?
• How do you perform a cervical plexus block?

2. What Would You Do? (WYD).


WYD questions relate to plans, backup plans, and courses of action. They test Knowledge,
Judgment, and Adaptability. In cases in which the problem is not severe or more information is
needed, no immediate action need be taken (or more information needs to be obtained). Note
that while the response, "I don't know" may be appropriate for a factual question (e.g., What is the
MAC of desflurane), it does not work for most WYD questions (e.g., The pt is blue. What would
you do?).

Examples
• How would you induce anesthesia?
• The patient is blue. What would you do?
• The patient is having a seizure. What would you do?
• Are you going to give Dantrolene?
• How would you prepare for major blood loss?
• The surgeon faints while weaning the pt from CPB. How would you proceed?

3. What Do You Think? (WYT)


WYT questions usually require identification and discussion of an existing problem. Included in
the discussion are issues such as perspective, rationale, level of significance, goals, priorities,
pros vs. cons, risk/benefit, and differential diagnoses. Like WYD, WYT questions test Knowledge,
Judgment and Adaptability. In essence, they require a Consultant opinion.

Examples
What do you think about the patient's K+ level?
Is the patient ready for surgery?
What do you think about his perioperative risks?
What are your goals during induction?
Is it safe to use induced hypotension?
Which is better, RA or GA?
Why do you think the patient's ICP is elevated?
The Essential Oral Board Review

4. Why?
In most cases, no action (e.g., no answer to WYD questions) is, in and of itself, absolutely right or
wrong. Only by knowing the reasons behind your choices can examiners decide whether your
rationale was correct. "Why?" questions test the same essential attributes as WYT - Knowledge,
Judgment and Adaptability. Among examiners, "Why?" is often considered the most essential
question. Therefore, expect to hear "Why?" frequently. (Even better than waiting to be asked,
include the answer to "Why" as a routine part of your answer.)

Examples
• Why did you ordered a cardiology consult?
• Why did you insert a PAC prior to induction?
• Why did you opt to perform a rapid sequence induction?
• Why is maintaining cerebral perfusion pressure more important than preventing aneurysm
rupture?
• Why not give the patient's own blood back to him/her?

5. What If...
Almost every plan must have a back-up plan, every choice an alternative. Obviously, "What if..."
questions focus on Adaptability. Although used synonymously with "flexibility". Adapting to a
changing scenario does not necessarily mean you must be flexible. When it comes to avoiding
succinylcholine in patients with upper motor neuron injury or avoiding a rapid sequence induction
in a patient with a documented difficult airway (e.g., can't intubate, can't ventilate), I tend to be
very inflexible. As mentioned previously, an examiner is not necessarily being mean when s/he
asks the candidate to adapt.

Examples
What if no ketamine were available?
There were no fiberoptic bronchoscope?
What if the PAC could not be inserted?
What if the BP were 60/40?
What if I told you the patient refused GA?
What if the surgeon says he wants you to transfuse the patient?"
What if the patient refused transfusions?

6. Explain Something to a Third Party


Sometimes, examiners ask you to explain something to a third party - patient, physician, family
member, nurse, administrator, etc. The task usually involves teaching some basic principles,
therefore, the question is actually testing Knowledge, Judgment, and Communication. Unless
asked to do so explicitly, avoid talking to the examiner as though you were talking to that third
party. Instead, simply describe what you would say.

Examples
• How would you explain to the medical students the need for preoperative cardiac evaluation
and p-blockade?
• How would you explain the wake-up test to a 12 year old girl scheduled for Herrington rod
insertion?
• How would you explain to a family the DNR status of their loved one in the OR?
• What would you tell the L/D nurses who have had five miscarriages over the past year and
suspect it's due to the inhaled anesthetics?
• You've been asked to confront a colleague caught abusing fentanyl. What would you say?

A common mistake made during answering these questions is confusing the questions,
"What do you think?" (WYT) with "What would you do?" (WYD). For example, "The patient is blue.
What do you think?" is different from "The patient is blue. What would you do?" To the first WYT
At
The Essential Oral Board Review

question, one could respond, "The differential diagnosis includes a low inspired 02 concentration, /Tp^
hypoventilation, intrapulmonary or intracardiac shunt, an abnormally high 62 diffusion barrier, or
hemoglobinopathy." For the second WYD question, one could respond, "I would first make sure the
patient were receiving 100% 02, hand ventilate, check my capnogram for ETC02, listen for bilateral
breath sounds, look for symmetric chest wall excursion, and review the perioperative history for clues
to the etiology." During an exam, WYD can sound much like WYT. Candidates must learn to listen
carefully, distinguish the two, and answer the exact question asked. (Those having difficulty
generating comprehensive Consultant-type responses may consider answering every WYT answer
with WYT first, then adding the WYD response. Similarly, one could follow every WYD answer with
the WYT response. If you do not, it is likely the companion question will be asked next anyway.) v^

Caveat: Jumping to Conclusions: Another common mistake made with WYT


and WYD question is to jump to a conclusion with insufficient information. Given the
fast pace of exams, the possibility that excessively slow and deliberate answers
could result in failure, and the aura of credibility that accompanies rapid responses,
there is a tendency to answer questions quickly. However, reaching a conclusion
when there is insufficient basis is not merely a minor error; it is completely wrong
(especially if you also seem overconfident or closed-minded.) There are numerous
times in medicine in which one must be uncertain. At such times, acknowledging
uncertainty is not a sign of weakness; it is shrewd recognition that there is more
diagnostic work to do. Being uncertain is vastly preferable to bluffing, being close-
minded to other possibilities, or being wrong (e.g., esophageal intubation). With
every question, develop the habit of asking whether there is sufficient data. If not,
part (if not all) of the answer may be to identify what is missing! (See page 6, Item
6)

As mentioned above, there are theoretically an infinite number of possible questions on the oral ^
Board exam. The following, either stated verbatim or as some permutation, are among the most v
common. *%
m
.a
1. Preoperative Assessment m
a. What do you think about the patient's "X" disease? ^
b. What are your anesthetic goals regarding "X" disease? ^
c. Why is the patient receiving "X" drug? How does it work? ^

2. Preoperative Optimization **
a. Is the patient ready for surgery? Why? How do you know? ^
b. What additional labs are necessary? Why? Could you proceed without them? V
c . W o u l d y o u o r d e r a C o n s u l t ? W h y ? W h a t s p e c i fi c q u e s t i o n s w o u l d y o u p o s e ? ^

3. Selective Monitoring v
a. What monitor(s) would you use? Why? **
b. Do you need to place "X" monitor prior to induction? Why? ^
c. Where would you insert "X" monitor? What's your backup site? What if you couldn't insert "X" ^
monitor? m>.
d. Why couldn't you use some other combination of monitors instead of "X"? v
e. How could "X" monitor fail? ^

4. Anesthetic Choices ^
a. How would you induce anesthesia? Why? Why not some other drug or technique? What if
V
your drug "X" were unavailable? What if the patient refused your technique?
b. How would you proceed if the patient had a difficult airway? What if "X" difficult airway device V
were unavailable?

'"Hi
V
The Essential Oral Board Review

c. How would you maintain anesthesia? Why? Why not some other drug? What if your drug
were unavailable?

5. Backup Plans
a. What if "X" technique failed?
b. What if "X" drug or device were unavailable?
c. What if "X" doesn't work?

6. Predicting Complications
a. What complications do you expect to occur? How would you prevent them or minimize their
severity?
b. What would you do if "X" occurred?

7. Responding to Events
a. "X" occurs. What would you think is going on? (WYT)
b. "X" occurs. What would you do? (WYD)
c. "X" occurs. Your response? (Open question for WYT, WYD responses, or both)

8. Differential Diagnoses
a. What could be the cause of "X" event or situation? What else could it be the cause?

9. Prioritizing Goals
a. Which is more important, doing "X" action or "Y" action?

10. Providing Information


a. Discuss, describe, explain, clarify "X."

The best way to prepare is, of course, to not only ponder these questions at your desk, but
to realize their applications in the everyday practice. Clinical application makes preparation
meaningful, which, in turn, makes it more memorable. If you have not already done so, begin to
routinely describe the pathophysiology of your pt's disease, optimize his/her preoperative problems,
explain (at least to yourself) the rationale for monitoring and anesthetic choices, anticipate likely
complications, and develop backup plans for each and every case. It should become apparent that
the best way to prepare for the exam is to simply practice good medicine and become the best
clinician possible.
12 The Essential Oral Board Review

Types of Cases

There are 2 types of cases presented during the oral exam, both distributed as written
information on separate sheets of paper.

Session 1
The case information provided will cover most of the page and, theoretically, contain all the
pertinent preoperative information necessary to proceed with the case. Little or no additional
preoperative evaluation should be necessary. The first examiner usually begins by asking
questions about Intraoperative Management (15 minutes). This is followed with questions about
Postoperative Care (10 minutes) from the second examiner. The Session concludes with the first
examiner, who asks questions about Additional Topics (10 minutes). (There is no planned
discussion of preoperative evaluation.)

Session 2
The information provided will consist of a short paragraph and only briefly describe the patient's
condition and type of surgery. Additional preoperative evaluation is necessary prior to surgery. The
first examiner usually begins by asking questions about Preoperative Evaluation (10 minutes). This
is followed by questions about Intraoperative Management (15 minutes) from the second examiner.
The Session concludes with the first examiner, who asks questions about Additional Topics (10
minutes). (There is no planned discussion of Postoperative Care.)

Session 1 Session 2
Length of printed Most of page Short paragraph
case
Information All pertinent preop information Brief description of patient's
provided necessary to proceed with case. condition and type of surgery;
additional preop information usually
necessary prior to surgery.
1st Examiner Intraop Management (15min) Preop Evaluation (10min)
2nd Examiner PostopCare (10min) Intraop Management (15min)
1st Examiner Additional Topics (10min) Additional Topics (10 min)
Total Time 35min 35min

T*
13 The Essential Oral Board Review

Dissecting the Stem


Just as there is no one best way to administer an anesthetic, there is no one best way to
dissect a stem question. The "best" anesthetic depends on the anesthesiologist's knowledge and
expertise, the practice setting, the needs of surgeon, and the wishes of the patient. In the same way,
every dissection is the unique product of the candidate's knowledge, experience, expectations, and
biases. Thus, a candidate who is very familiar with airway management may spend little time
planning intubation, whereas one less familiar with airways may struggle for minutes to recall the
ASA airway algorithm. This variation in candidate ability and attitude makes the task of creating
universal recommendations on how to dissect the stem problematic. Adding to the difficulty, the
actual approach required depends on whether the case is from Session 1 or Session 2.

Fortunately, the essential elements of dissection are the same for all candidates and all
stem questions: (1) Read the case completely, (2) Identify the major problems, (3) Identify the
nature and severity of major problems, (4) Determine whether the major problems are optimized
(optimize if necessary and possible), (5) Establish goals, (6) Weigh priorities (risk/benefits,
pros/cons), (7) Develop a plan, (8) Prepare backup plans, (9) Anticipate complications, and (10)
Reread the case.

Step 1 Read the Case Completely

Do not rush. Note carefully the pt's height, weight, medications, labs, and vital signs. Ask whether
the case is an emergency. If so, you may have to be expedient and accept a less that ideal
properative workup.

Step 2 Identify the Major Problems

It has been said that every word and number in the stem question can be considered a problem
(including age, gender, weight, height, drugs, and vital signs). Although perhaps true, most cases
on the oral exam (and real life) have 3-5 major problems. This is especially true if the problems are
grouped by system. Consider using the following mnemonic to group your problem list:

ABCDEFGHIJKLM: Airway and Breathing, Cardiovascular, Neurologic (De Brain),


Endocrine/Electrolytes/Fluids, Gastrointestinal, Hematologic,
Infectious/lmmunologic, Joints/Skin/Bone, Kidneys, Lines, and Medications.

Thus, a patient with (1) CAD, (2) HTN, (3) LVH, (4) an EF of 30%, and (5) Pacemaker may have 5
separate problems, but for the purposes of dissection, those problems can all be grouped under
one system - Cardiac (even though each of the 5 may ultimately require its own evaluation).
14 The Essential Oral Board Review

Stepl
Read the Case Completely

Step 2
Identify the Major Problems

Step 3
Determine Nature and Severity of
Major Problems (H/P, labs, consults)

Preop Step 4
Determine if Problems are Optimized
(Optimize if Necessary/Possible)

Step 5
Establish lntra-/Postop Goals
U
Intra/Postop

Step 6
Develop Plans
(Based on Assumptions)

T
Step 7
Weigh Priorities -» Plan A
(Risk/Benefits, Pros/Cons)

1
Step 8
Prepare Backup Plans (B, C, D)

Step 9
Anticipate Complications

Step 10
Reread the Case
15 The Essential Oral Board Review

Step 3 Determine Nature and Severity of Major Problems (H/P, labs, consults)

Nature refers to the diagnosis, likely or confirmed, that best explains the patient's symptoms,
signs, and clinical picture. Severity refers to the degree of that illness - mild, moderate, or
severe. In Session 2, in which only the name of a disease or mention of a symptom is given, the
whole point is to determine the nature and severity of disease. For instance, a dyspneic patient
may have many reasons he is SOB - cardiac, respiratory, musculoskeletal, neurologic, or
psychogenic. Finding out which one best accounts for the dyspnea is defining the nature of
disease. Whatever the nature, his degree of dyspnea - whether it results in an ASA physical
status classification of 2, 3, or 4-5 (corresponding to mild, moderate, or severe, respectively),
defines severity. In Session 1, enough information is usually given to allow determination of the
nature and severity of disease, and to proceed to induction of anesthesia. For instance, a patient
with well-controlled, one-vessel CAD with 50% LAD occlusion, stable angina with exertion, an EF
of 55%, and adequate p-blockade is probably ready for surrgery. Do not seek additional
information, especially preoperatively in Session 1, unless it is absolutely necessary.

Almost invariably, the workup to determine nature of disease and severity requires a thorough
understanding of physiology and pathophysiology. (How can you know what questions to ask
without knowing what manifestations of disease to look for?) The typical workup includes:

0 History and physical -* (2) Labs and studies -> (3) Consultation

Be sure to work up a problem in that order, if at all possible. A common mistake is to prematurely
resort to labs or consultation without first performing an adequate H/P. Not uncommonly, a
thorough H/P is all that is needed to determine the severity of a problem.

Caveat: Making Diagnoses: Diagnoses (and assumptions about nature and


severity) are simply working hypotheses. In medicine, there are many reasons
one may need to continually question a diagnosis:

(1) Your assumptions may about nature and severity may be incorrect (e.g., the
fever you thought was due to hyperthyroidism is really due to MH). Is there
other data you can obtain to support or refute your diagnosis?
(2) The patient's condition may change (e.g., the patient's dyspnea may have
initially have been due to asthma, but is now due to a pneumothorax).
(3) Multiple disorders may conspire to exacerbate or produce a common
problem (e.g., sepsis + massive transfusion + aspiration -> pulmonary
edema).

Locking yourself into a diagnosis is a common pitfall of inexperienced clinicians,


and falls under the category of failure to demonstrate Adaptability. Remember:
avoid rigidity and keep an open mind to taking an H/P, ordering appropriate
labs/studies, and requesting the assistance of a consultant whenever necessary
(e.g., preop, intraop, postop).
16 The Essential Oral Board Review

Step 4 Determine if Problems are Optimized (Optimize if Necessary/Possible)

In Session 1, major preoperative problems are usually already optimized, at least well enough to
start the case. Nevertheless, even in Session 1, some pre-anesthetic optimization may be
necessary and possible (e.g., giving a fluid bolus to a hypovolemic patient before an emergent
induction, administering FFP to a patient taking warfarin before performing a neuraxial technique,
obtaining a CXR or placing a chest tube in an trauma patient with decreased unilateral breath sounds
before beginning positive pressure ventilation.)

Conversely, Session 2 problems are more likely to require optimization, which may consist of
something as simple as checking a morning glucose level or involved as delaying elective surgery for
myocardial revascularization. Note that while optimization may require temporary cancellation of
surgery, in the orals, it will never mean complete cancellation. (Examiners need to finish the exam!)
This is does not mean that one should never say that the case needs to be cancelled or delayed.
(One should say whatever is necessary for best patient care.) Just expect the examiner's response
to be: (1) "OK, the problem is now optimized. Now what would you do?" or (2) "Fine, the case
becomes an emergency [for whatever reason]. Now how would you induce anesthesia?"

Also, make certain your reasons for cancellation/delay are legitimate. This means that the problem
requiring optimization, left untreated, should be so severe, it is expected to worsen outcome, and its
optimization to make outcome better (e.g., active pneumonia in a patient scheduled for elective
tonsillectomy, new onset unstable angina in a patient scheduled for Nissen fundiplication, and a
predialysis K+ of 7.0 mEq/L in a patient scheduled for total knee repair).

Caveat 1: Reasonably Seeking Consultation: Sometimes when working up or


optimizing a problem (Steps 3 and 4, respectively), it becomes necessary to seek
the assistance of an outside Consultant. This is perfectly acceptable, as long as the
anesthesiologist has first done everything reasonable to solve the problem
him/herself. What is reasonable is, of course, a matter of Judgment and will vary
from practitioner to practitioner. Most people would agree that it is reasonable for
the anesthesiologist to order an ECG or CXR to evaluate of angina, or a lumbar
MRI to rule out epidural hematoma following his/her neuraxial technique. It also
seems reasonable to seek cardiology consultation if, after adequate H/P, the
patient's symptoms seem to require cardiac catheterization or Holter monitoring.
Should an anesthesiologist perform a transthoracic echocardiogram to determine
EF or contractility? That is a matter of Judgment, depending on the setting and
skills/training of the anesthesiologist.

Caveat 2: Severity, Optimization, and Expert Opinion: Just because a disease


is severe does not necessarily mean it is not optimized. For example, a patient
could have CHF and, despite optimal medical management, be in the best condition
possible. Sometimes, we depend on expert opinion to tell us whether a patient is
optimized. As a Consultant, you are expected to know enough medicine to realize
when that expert opinion may be incorrect. For example, if a hypertensive patient's
preoperative BP were 220/110 and all he has ever been given for the problem is
furosemide, his HTN is probably not optimized, despite the internist's insistence that
it is. Similarly, if, by history, a patient could walk up 2 flights of stairs last week, but
is now SOB at rest with bilateral crackles midway up both lung fields, he is probably
not ready for elective surgery, despite the cardiologist's note "clearing" him for
surgery.
17 The Essential Oral Board Review

Caveat 3: Optimization and Clarification: In the real oral exam, especially


Session 2, questions commonly proceed to Intraoperative Management without
apparent optimization or (at least) clarification of the patient's condition (e.g., you
have been discussing the patient's EF and the examiner asks, "What would you
think if I told you the EF were 30%?", but never states explicitly it actually is 30%.)
Although this could be a cunning test of whether the candidate is aware s/he is
proceeding without optimizing a major problem, it is more likely an innocent
oversight on the part of the examiner. To be safe, such ambiguity should not be
ignored. If (and only if) the problem were significant (as it would be with a 30% EF),
consider starting your answer to "What would you do?" questions with, "Assuming
the EF really is that low..." While some feel this conservative approach is overly
precautious and uneccessary, I prefer to risk appearing too careful than too cavalier.

********************************************************

The previous Steps (2-4) address preoperative evaluation, the goals for which can
be summarized as:

1. Determine the nature and severity of disease.


2. Determine whether the disease is optimized.

The following Steps (5-9) address intra- and postoperative concerns.


********************************************************

Step 5 Establish lntra-/Postop Goals

Once the diseases have been identified, their severities described, and the important ones
optimized, intra- and postoperative goals should be established. Intraoperative goals usually entail
taking measures to control physiology, prevent complications, and improve outcome. The exact goal
chosen depends, of course, on the specific problem involved and perioperative circumstance in
question. For example:

System Problem Sample Goals


Airway/Breathing Asthma Deep anesthesia to prevent bronchospasm
Cardiovascular CAD Maintain hemodynamic stability
De Brain Traumatic brain injury Prevent secondary injury
Electrolytes Hypokalemia Avoid worsening of hypokalemia
Endocrine Chronic steroid use Avoid adrenal insufficiency
Fluids Pulmonary edema Prevent further pulmonary edema
Gastrointestinal Full stomach Reduce the risk of aspiration
Hematologic Jehovah's Witness Avoid homologous blood products
Infectious/Immune Latex allergy Avoid latex allergens
Joints/Skin Insensible fluid loss Minimize losses
Kidneys ARF Supply adequate GFR
Lines Poor access, large EBL Obtain large-bore IV access
Medications Clonidine Prevent clonidine withdrawal

Postoperative goals tend to be similar to intraoperative ones - control physiology, prevent


complications, and improve outcomes - albeit in a PACU, ICU, hospital ward, or outpatient setting
setting (e.g., continuation of p-blockade to prevent myocardial ischemia, intubation and
hyperventilation for control of elevated ICP, postop thoracic epidural analgesia [with local anesthetic]
to minimize postop pulmonary complications).
18 The Essential Oral Board Review

Note that when dissecting a stem question, numerous goals may be possible, but with limited time
and resources, only the most important should be emphasized, i.e., those expected to most
significantly affect outcome. (Thus, preoperative cessation of smoking may not be as important as
treating acute bronchospasm.)

Step 6 Develop Plans

Intra- and postoperative plans should follow directly and logically from established goals.
Intraoperative goals typically include those involving monitoring, induction, and maintenance of
anesthesia. Postoperative goals typically include those involving extubation. discharge, pain control,
and ventilators. No where is it more important to have an accurate, up-to-date understanding of the
pathophysiology, treatment options, and controversies surrounding contemporary anesthetic issues.
For example, continuing with Step 5 above:

System Problem Sample Goal Sample Plan


Airway/Breathing Asthma Deep anesthesia to Premedicate with albuterol; induce with
prevent bronchospasm high-dose fentanyl, lidocaine, and
moderate-dose sevoflurane
Cardiovascular CAD Maintain hemodynamic High-dose fentanyl (while monitoring
stability with the AL)
De Brain Traumatic brain Prevent secondary injury ABCs, moderate hypothermia to 34°C
injury (controversial)
Electrolytes Hypokalemia Avoid worsening of Avoid hyperventilation
hypokalemia
Endocrine Chronic steroid use Avoid adrenal Hydrocortisone 100 mg q 8 hrs x 3
insufficiency
Fluids Pulmonary edema Prevent further 3% NS, colloids for resuscitation
pulmonary edema (monitor CVP and PCWP)
Gastrointestinal Full stomach Reduce the risk of Bicitra, cimetidine, reglan; delay surgery;
aspiration rapid sequence induction
Hematologic Jehovah's Witness Avoid homologous blood Cell saver, induced hypotension,
products DDAVP
Infectious/Immune Latex allergy Avoid latex allergens Latex precautions
Joints/Skin Heat loss Minimize losses Warm room, humidify breathing circuit,
heated warming lamps
Kidneys ARF Supply adequate GFR Adequate hydration and perfusion
pressure, avoid high dose
vasoconstrictors, fenoldopam
Lines Poor access, large Obtain large-bore IV Large-bore IV, C-line, cut down
EBL access
Medications Clonidine Prevent clonidine Continue clonidine
withdrawal

V.
Caveat: Resolving Ambiguity: During Session 2 cases, the information provided
on severity is usually insufficient to decide precisely how to monitor, induce, and
maintain anesthesia. To resolve this inherent ambiguity, it is often necessary to
make assumptions about disease severity. Often, you have a choice when making
assumptions: Plan your case for a (presumably optimized) disease that is (1) Mild,
(2) Moderate, or (3) Severe. Ideally, you should be able to Adapt to all severities,
but since severe diseases is usually more complicated, I would at least prepare a
plan for severe disease. Also note that among the possible systems involved,
Airway/Breathing. Cardiovascular, and Neurologic problems have the greatest
impact on induction and maintenance (See page 6, Item 6).
The Essential Oral Board Review

Step 7 Weigh Priorities -> Plan A (Risk/Benefits, Pros/Cons)

If all oral Board cases consisted of one major problem with one major goal, planning an anesthetic
would be simple (and the pass rate undoubtedly higher). In actuality, stem questions on the orals
(and problems in real life) tend to contain several problems with conflicting goals, requiring a
candidate to apply accurate Knowledge and use keen Judgment. The next several pages contains
five hypothetical cases, each with problems, goals for intubation, and possible plans for intubation:

Case 1: Diabetes and GER


Problem Diabetes GER
Intubation Goal: Minimize aspiration risk Minimize aspiration risk
Possible Plan for (1) Rapid sequence induction (RSI) or (1) RSI or
Intubation: (2) Awake intubation (2) Awake intubation

In the Case 1, both diabetes and GER have exactly the same intubation goal (minimize aspiration
risk) and choices for intubation (RSI or awake intubation), so the plan is straightforward: either RSI
or awake intubation. (Of course, diabetes may be associated with CAD, stiff joint syndrome, etc.,
creating other intubation goals and plans, but for the purposes of simplifying discussion, these are
not mentioned.) Now consider a patient with different goals:

Case 2: Diabetes and Difficult Airway


Problem Diabetes Difficult Airway
Intubation Goal: Minimize aspiration risk Maintain spontaneous ventilation
Possible Plan for (1) RSI or (1) Awake intubation or
Intubation: (2) Awake intubation (2) GA with spontaneous
ventilation

In Case 2, even though the goals for diabetes and the difficult airway are different (minimizing
aspiration risk for diabetes, maintaining spontaneous ventilation for the difficult airway), they
share one common plan for intubation (awake intubation). Thus, awake direct laryngoscopy,
fiberoptic intubation, blind nasal, retrograde wire, or tracheostomy are a few among several
options for intubation. Now, consider the patient for whom both goals and plans may appear to
differ:

Case 3: Difficult Airway and Asthma


Problem Difficult Airway Asthma
Intubation Goal: Maintain spontaneous ventilation Deep anesthesia to prevent
bronchospasm
Possible Plan for (1) Awake intubation or (1) DeepGA
Intubation: (2) GA with spontaneous ventilation

In case 3, it may, at first, appear that there is no common ground for intubation (awake intubation
or GA with spontaneous ventilation for the difficult airway vs. deep GA for the asthma). But
actually, GA with spontaneous ventilation (difficult airway) and deep GA (asthma) can be
compatible if the deep GA (asthma) were achieved while maintaining spontaneous ventilation.
Therefore, possible intubation plans include: (1) GA with ketamine or (2) GA with sevoflurane,
enflurane, or halothane - all while maintaining spontaneous ventilation. (Some may also suggest
propofol or etomidate, but dose of both that allows laryngoscopy/intubation without causing
bucking also tends to produce apnea.)

Even though the above cases are gross oversimplifications, it is worth asking whether any
essential information was missing from all three. Think carefully. What essential information is
should have been provided before you were able to decide what to do? If you said severity, you
are well on your way to becoming Consultant. In each of the three case above, two problems
were given, but none mention severity. (Remember, severity is the basis for making decisions.)
20 The Essential Oral Board Review

Fortunately, since compatible plans could be found for all Cases, neglecting to include severity
seems only a minor omisision. (Actually, it may not be all that minor since mild problems need
not even be considered in making the plan (e.g., if the GER were controlled, a RSI may not even
need to be considered). Conversely, if the problems had no apparent plans in common, severity
would matter greatly, as seen in the following patient:

Case 4: Difficult Airway and GER


Problem Difficult Airway GER
Intubation Goal: Maintain spontaneous ventilation Minimize aspiration risk
Possible Plan for (1) Awake intubation or (1) RSI or
Intubation: (2) GA with spontaneous ventilation (2) Awake intubation

In Case 4, the possible plans seem incompatible. After all, it is difficult to guarantee
hemodynamic stability (CAD) with either a RSI or awake intubation (GER) since neither
adequately minimizes the chances of hypertension, hypotension, tachycardia, and bradycardia.
Similarly, a slow controlled induction (CAD) violates the goal of minimizing aspiration risk (GER).
How, then, can the conflict be resolved? By determining severity of disease. In Case 4, if the
CAD were severe and the GER mild, a slow controlled induction may be preferred. On the other
hand, if the GER were severe and the CAD (very) mild, a RSI or awake intubation could be
chosen. Severity helps resolve conflicts.

How can severity be determined? You may already know the answer: History and physical.
(Followed by ordering appropriate labs/studies, and, if necessary, consultation.) If, by H/P, it is
determined that one major problem is more severe than another, it should be easy to prioritize
problems.

What if both problems were severe and their goals in conflict? The answer is to (1) Perform a
risk/benefit, pros/cons analysis (to determine which problem receives priority) and (2) Remember
that there is not always only one best answer. In any given patient, even though two or more
problems may be severe, one often stands out as the most severe, most life-threatening, and/or
most likely to cause harm if ignored. In Case 4, the severe CAD may be considered a greater
concern than severe GER because myocardial ischemia (e.g., resulting from a RSI or awake
intubation) can cause life-threatening on induction. Thus, a slow controlled induction may seem
justified. Having said that, prioritization of severe problems is a matter of Judgment, and as such,
will vary between Consultants. Another equally qualified Consultant anesthesiologist may point
out that a slow controlled induction in a patient with severe GER risks reflux, aspiration, and life-
threatening pneumonitis. Thus, it would not be totally wrong to perform a RSI. Does this mean
that there are never any absolutely right or wrong answers? Yes, when it comes to major
problems that are both significant and severe. (The first lesson taught to junior oral Board
examiners: there is no one best answer. The seond: always ask Why.)

Therefore, when several simultaneous severe problems are in conflict, the best plan is one that
not only prioritizes the one you feel is most severe, but also accounts for the other problem(s).
For example, in Case 4, if the CAD were deemed the most severe problem, one could not only
perform a slow controlled induction, one could also: (1) Administer aspiration prophylaxis, (2)
Mask ventilate with cricoid pressure, and (3) Keep the patient in Trendelenberg position (to
prevent aspiration of any regurgitated gastric contents). On the other hand, if GER were deemed
the most severe problem, one could not only perform a RSI, one could also (1) Administer a prior
dose of fentanyl and lidocaine to blunt the hemodynamic response to laryngoscopy and
intubation, (2) Minimize the time of laryngoscopy, (3) Monitor BP with an AL, and (4) Have
rapidly-acting agents for hemodynamic control (e.g., esmolol, SNP, and propofol) readily
available. Since there are often no absolutely right or wrong answers on the orals, depending on
the circumstances, many different plans may be acceptable - as long as they show an attempt to
account for all problem(s). Remember: when choosing between diametrically opposed severe
and conflicting goals/plans, always try to account for the other severe problem(s).
21 The Essential Oral Board Review

What if more than 3 severe problems exist, as in the case of a patient with GER, a difficult airway,
and large unruptured intracranial aneurysm?

Case 5: GER, Difficult Airway, and Unruptured Intracranial Aneurysm


Problem GER Difficult Airway Large unruptured
intracranial aneurysm
Intubation Goal: Minimize aspiration Maintain spontaneous Avoid HTN
risk ventilation
Possible Plan for (1) RSI or (1) Awake intubation (1) Slow controlled induction
Intubation: (2) Awake intubation or
(2) GA with
spontaneous
ventilation

Think carefully. What was your first response? If you said, "I don't know", you would be halfway
correct. While it is tempting to automatically prioritize the difficult airway or unruptured aneurysm,
the correct answer is, "It depends." By now, you must know on what. Severity!

Case 5a: Mild GER, Class 2 Difficult Airway, and Unruptured Intracranial Aneurysm
Problem Mild GER Class 2 Difficult Large unruptured
Airway intracranial aneurysm
Intubation Goal: Minimize aspiration Maintain spontaneous Avoid HTN
risk ventilation
Possible Plan for (1) RSI or (1) Awake intubation (1) Slow controlled induction
Intubation: (2) Awake intubation or
(2) GA with
spontaneous
ventilation

Now, with severities of GER and difficult airway defined, the problem is simpler to solve. (Again,
this is what Preoperative Evaluation is all about.) Assuming the class 2 airway were one thought
to be easily intubated (sometimes a Judgment difficult) and there were no other major coexisting
problems (e.g., morbid obesity, CHF with pulmonary edema, etc.), the unruptured aneurysm
remains the only major problem. It therefore receives priority. Plan A: Slow controlled induction.

As a final variation, what if all three problems were difficult?

Case 5b: Severe G ER, Class 4 Difficult Airway, and Unruptured Intracranial Aneurysm
Problem Severe GER Class 4 Difficult Large unruptured
Airway intracranial aneurysm
Intubation Goal: Minimize aspiration Maintain spontaneous Avoid HTN
risk ventilation
Possible Plan for (1) RSI or (1) Awake intubation (1) Slow controlled induction
Intubation: (2) Awake intubation or
(2) GA with
spontaneous
ventilation

As mentioned above, when severe problems are in conflict, it is time to (1) Perform a risk/benefit,
pros/cons analysis to be performed, (2) Realize that there is no one best answer, and (3) Find
plan that can account for all problems. In performing a risk/benefit, pros/cons analysis (i.e.,
asking which problem is most severe, most life-threatening, and/or most likely to cause harm if
ignored), it becomes clear that mismanaging any of the three problems could lead to life-threating
consequences (aspiration, loss of airway, and aneurysm rupture). A RSI may reduce the risk of
aspiration, but not loss of airway or aneurysm rupture. An awake intubation may reduce the risks
of aspiration and loss of airway, but not aneurysm rupture. GA with spontaneous ventilation may
reduce the risk of loss of airway, but not aspiration. A slow controlled induction may minimize the
22 The Essential Oral Board Review

risk of aneurysm rupture, but not aspiration or loss of airway. So there is no perfect plan. But
can we find one that minimizes risk and maximizes benefit, one that has more pros than cons?

Plan A: Here is one possibility: For GA, a truly difficult airway is often thought to take priority
over all other problems because severe hypoxia is immediately fatal. (With aspiration and an
aneurysm rupture, death is often more remote or delayed.) Therefore, in Case 5b, plan A might
be: Awake fiberoptic intubation. To account for the other two problems (GER, aneurysm), one
could also: (1) Administer aspiration prophylaxis, (2) Mask ventilate with cricoid pressure, and (3)
Keep the patient in Trendelenberg position (...to minimize the chances of aspiration, and for the
aneurysm...) (4) Administer a judicious dose of fentanyl, lidocaine, midazolam, and/or droperidol
to blunt the hemodynamic response to intubation, (5) Perform transtracheal, superior laryngeal,
glossophyargeal nerve blocks; administer aerosolized lidocaine by mask; and apply lidocaine jelly
to the tongue/nares (6) Monitor BP with an AL, and (7) Have rapidly-acting agents for
hemodynamic control (e.g., esmolol, SNP, and propofol) readily available.

Plan B: What if the patient were vomiting feculent material? Prioritizing aspiration, here is
another possibility, plan B: RSI. To account for the other problems (airway, aneurysm): (1) Have
emergency airway equipment available, including different blades, different sized ETTs, LMA,
combitube, jet-ventilator, surgeon with a scalpel in hand + (steps 4-7 from Plan A).

Plan C: And if the surgeon said the aneurysm were so large and unstable that if it ruptured, the
patient would certainly either die or have a poor neurologic outcome, plan C, prioritizing
aneurysm rupture, might consist of: Inhalation induction with sevoflurane. By maintaining
spontaneous ventilation, the inhalation induction accounts for the difficult airway. And to account
for aspiration, one could (1) Administer aspiration prophylaxis) and (2) Keep the patient in
Trendelenberg position.

If neither plan B or C seem as attractive as plan A, you would be showing good Judgement. A
RSI, even with an LMA and surgical backup, is generally frowned upon with a truly difficult airway
(e.g., class 4) and risks of both aspiration and loss of airway are not adequately reduced by an
inhalation induction. My first plan would, therefore, be plan A.

Step 8 Prepare backup plans

Just as a patient's condition can suddenly change in everyday practice, so do examiners frequently
change disease severity during an exam (e.g., "What if in addition to the AAA, the patient had a
difficult airway?") As previously discussed, such questions are clearly intended to test a candidate's
Adaptability. In weighing priorities by comparing different goals/severities/plans above, you have
may have already started to develop backup plans. During the exam (as well as when you study or
administer anesthesia), it is useful to ask similar "what if something didn't work" questions (e.g.,
"What if there were no ketamine?" "What if the IV / AL / CVP / PAC / TEE / EEG couldn't be
inserted?" "What if the patient refused an awake fiberoptic intubation?")

Step 9 Anticipate complications

There are many reasons experienced clinicians have fewer complications. Among the most
important are: (1) They know what bad things could happen and (2) They take measures to prevent
(and are able to readily treat) them. In other words, they are anticipate. When you dissect a case, if
you anticipate the likely complications, you will have a better idea what questions will be asked (e.g.,
the patient undergoing a CEA will have intraoperative cerebral ischemia, postoperative HTN, and
23 The Essential Oral Board Review

preoperative myocardial ischemia). When you do the same in clinical care, you will have fewer
complications.
********************************************************

Steps 5-9 above dealing with intra-and postoperative care can be summarized as:

1. Establish goals for each problem


2. Determine plans for each goal
3. Make plans for each problem/goal
4. Look for conflicts between plans and assign priorities
based on severity of disease (as determined by H/P)
5. Attempt to account for all major problems in making the final plan
6. Devise backup plans
7. Anticipate complications based on the identified problems
*******************************************************

Step 10 Reread the case

It is always interesting, but perhaps not surprising, how many things can be missed with the first
reading of a case. Candidates are nervous, pressured, and eager to begin dissection.
Consequently, important facts and details are frequently overlooked or misread. (Believe it or
not, one of the most common is forgetting to account for the difficult airway.) Thus, if you find you
have additional time after your initial dissection, reread the case. Similarly, do not be afraid to
review the case for details you may have missed. Overlooking details has become an especially
common problem since the introduction of Session 1 cases, which consists of almost a full page
of information. Learn to read carefully, then reread as time permits.
24 The Essential Oral Board Review

Case Dissection in the Real World


When one first starts learning anesthesia, writing down plans helps create a feeling of
security - it forms a reassuring straight line between the various points in the case. But as one
learns more, it becomes impossible to write down everything. It is also unnecessary because
administering anesthesia becomes routine. In addition, perioperative care of sick patients is not a
simple straight line; it is more like a web - a complex, intricate pattern of events and relationships,
someiimes interdependent and overlapping, and sometimes with no connecting threads. A
competent clinician may be able to take care of such cases by virture of extensive his/her
experience. But completely planning out such a web on a piece of paper is extremely hard to do.

In the same way, candidates first preparing for the oral Boards are naturally drawn to any
technique that offers a simple, straight-line approach to stem question dissection. In fact, it is not a
bad idea for beginning candidates to write down as much as possible. But, as in clinical care, it soon
becomes apparent that trying to write down everything is impossible. It is also unnecessary since
topics that are truly familiar can usually be discussed extemporaneously. As in real life, cases on the
oral Board are a complex network of problems, concerns, and controversies, not straightforward
linear issues. It may be possible, by virture of experience and preparation, to handle such a web
during an exam. But it is extremely hard to capture its intricacies on paper.

Ideally, by the time you take the orals, you should be so familiar with the issues, formal
dissection is a mere formality (and it would not matter that you forgot to write something down). In
reality, most candidates are still at least somewhat unfamiliar with several issues. Writing down
details is as necessary as it is comforting. And what you need to write down depends on your
Knowledge, Judgment, and individual thinking/learning style. As alluded to previously, there is no
single best technique for every candidate to dissect every stem question because everyone's
needs are different. The best way is whatever consistently works for you.

Whatever approach you choose (e.g., numbered problem lists, flow charts, and spread
sheets), the most important items to include are the essential features of stem question dissection
described above: (1) A problem list, (2) Goals, (3) Plans, (4) Priorities, (5) Backup plans, and (6)
Anticipated complications - not necessarily in that order.
25 The Essential Oral Board Review

Sample Exam

The following stem question was based on a real case. As your read through the sample dissection,
remember that whatever is written reflects my Knowledge, Judgment, Adaptabiltiy, and bias.
They are not absolute answers.

A 37 year old, 5'6", 66 kg female is scheduled for an emergency evacuation of a


hematoma in her right knee. She had an aortofemoral bypass graft declotted one
week ago at that site. Her past medical history is remarkable for renal artery
stenosis, coronary artery bypass grafting done one year ago, and intravenous drug
use. Her medication include captopril, propranolol, Coumadin, and thiazide. HR 62,
BP 276/172, R 28, Hct 26, INR 1.4, ECG shows a left bundle-branch block and Q
waves in lead III.

Since this is a Session 2 case, preoperative evaluation and intraoperative management will be
discussed.

Step 1 Read the Case Completely

Do this now, if you have not already done so. Note that the case is stated as an emergency. It is
also a Session 2 test, therefore, only Preoperative Evaluation and Intraoperative Management will be
discussed.

Step 2
Identify the Major Problems

Highlighting the key words in the case:

A 37 year old, 5'6", 66 kg female is scheduled for an emergency evacuation of a


hematoma in her right knee. She had an aortofemoral bypass graft declotted one
week ago at that site. Her past medical history is remarkable for renal artery
stenosis, coronary artery bypass grafting done one year ago, and intravenous
drug use. Her medications include captopril, propranolol, Coumadin, and
hydrocholorthiazide. HR 62, BP 276/172, R 28, Hct 26, INR 1.4, ECG shows a left
bundle-branch block and Q waves in lead III.

Grouping these key words by systems yields a problem list like this:

1. Cardiovascular (CAD, S/P Ml, HTN, PVD, LBBB)


2. Hematologic (elevated INR possibly from Coumadin)
3. Renal -RAS
4. Intravenous Drug Abuse
5. Medications (captopril, propranolol, HCTZ)
6. NPO status
26 The Essential Oral Board Review

Preoperative Evaluation
Step 3 Determine Nature and Severity of Major Problems (H/P, labs, consults)

Step 4 Determine if Problems are Optimized (Optimize if Necessary/Possible)

1. Cardiac
Preoperatively, the patient has a history of CAD, Ml, PVD (RAS), LBBB, and is acutely, critically
hypertensive.

a. HTN: The most concerning among these at the moment is the HTN. If it were real, I would
want to know the patient's usual BP, associated symptoms/signs (myocardial ischemia, CHF,
mental status changes), and possible causes (last dose of antihypertensives, last illlicit drug
use, e.g., cocaine, use). 276/172 is malignant HTN. It should be treated (to prevent cardiac
and neurologic complications) before induction, either by addressing any treatable cause (e.g.,
hypoxia, pain, anxiety) or with vasodilators/p-blockers. If the EF were low, high dose p-
blockers should be used with caution.

b. CAD: Little is mentioned about her CAD and Ml except that she had a CABG one year ago
and now has Q-waves by ECG. I would want to know her recent pattern of chest pain,
syncope, DOE, PND, palpitations, and exercise tolerance. I also want to know when she last
saw her private physician, that physician's assessment, and results of any recent cardiac
catheterization/stress study/evaluation.

c. LBBB: The patient's LBBB is worrisome not only for more severe CAD but the possibility that it
may develop into a high-grade block. A history of syncope or SOB may be helpful, as would
the results of any evaluation by his private cardiac physician.

d. PVD: The presence of RAS suggests she may have more diffuse peripheral vascular disease.
I would ask if PVD, especially carotid and aortic, were every considered in the patient's history.

2. Hematologic
The patient's PT of 1.4 is moderately elevated, the most likely cause for which is the warfarin. But
the history of IVDA makes hepatitis and liver dysfunction possible. To make sure, I would ask for
a history of bleeding and bruising, ask when the last dose was given, and obtain the PTT and pit
count. In addition, if the bleeding were significant, even greater anemia may become apparent
after hydration. Depending on the amount of bleeding, at least 2 units should be typed and
crossed, possibly with platelets and FFP. Also, the reason for the patient's use of warfarin should
be confirmed (H/O thrombotic problems?).

3. Renal
I would like to know whether the RAS is bilateral, its severity, and whether there is associated
renal dysfunction. The BUN/Cr and results of any previous assessement of renal function should
be made available. Also, I wonder whether the Hct of 26 is chronic (e.g., from CRF), or acute
(e.g., from bleeding). If the RAS were bilateral and significant, is captopril an appropriate
antihypertensive?

4. Intravenous Drug Abuse


I would like to know what drugs she takes, when she last took them, and whether are any
multisystem complications from drug use exist, (e.g., if cocaine were used, myocardial ischemia,
dysrhythmias, cardiomyopathy, endocarditis, seizures, cerebral infarction, intracrantial bleed,
asthma, pneumonitis, and pulmonary hemorrhage are possible. If narcotics were used, opitate
resistance should be expected.) I would also be concerned about infectious hepatitis/HIV status
27 The Essential Oral Board Review

and possible difficult IV access. A drug screen, hepatitis/HIV panels, and LFTs should be
obtained.

5. Medications
Captopril is an ACE inhibitor that can cause cough, angioedema, and worsening of renal function
with severe bilateral renal artery stenosis (a disorder in which GFR is maintained by angiotensin
2-mediated efferent arteriolar vasoconstriction). Propranolol is a nonselective" p-blocker that can
cause syncope, bradycardia, hypotension. Thiazide is a diuretic working at the distal tubules that
can cause orthostasis, hyponatremia, hypokalaemia, hypomagnesemia, and hyperchloremic
metabolic alkalosis. Given her hypertension, I wonder if the patient has been compliant with
and when she last took her medications. If the HCTZ had been taken recently, urine output
may not be a valid indicator of volume status. If the propranolol had been taken recently, p-
blocker withdrawal should be considered a cause for any unexplained tachycardia.

6. NPO Status
Whenever the last meal was given, the pain and axiety associated creates a persistent aspiration
risk, justifying prophylaxis (H2 blocker, reglan, bicitra).

Time permitting, there is always more one could think and write about, e.g., (1) HTN/CAD, (2) Renal
failure, (3) Chronic renal failure, (4) Liver failiure, (5) IVDA (e.g., heroin, LSD, ecstasy or
3,4,methylenediolxylmethapamphetamine, (6) Anemia, (7) Transfusion medicine, (8) Fluid and
electrolyte management, and (8) Whatever else you feel may be asked. The benefit of having
mastering these issues in advance of the exam is that it makes such time-consuming review less
necessary, leaving more time to concentrate on Judgment and Adaptability. In addition, it is always
prudent to list the additional labs you need to obtain (e.g., electrolytes, CBC, platelets, LFTs, CXR),
perhaps in a box. Finally, the patient's airway needs to be addressed.

Intraoperative Management

Step 5 Establish lntra-/Postop Goals

Step 6 Develop Plans (Based on Assumptions)

Step 7 Weigh Priorities -» Plan A (Risk/Benefits, Pros/Cons)

Step 8 Prepare Backup Plans

Step 9 Anticipate Complications

(In an emergency case, it is not uncommon for major problems to be incompletely explained and/or
optmized. Thus, you may have to proceed with induction and continue the investigation/optimization
intra-/postoperatively. Also, because of extensive interaction between problems, it may be difficult to
follow the above linear path in working up every problem. It is not uncommon to have to jump back
and forth between different problems and perioperative periods. It does not matter what path you
choose or how the information looks on paper as long as you are able to answer the questions
during the exam.)

1. Cardiac
a. HTN: (Temporarily putting aside issues such as cause of the HTN and other preoperative
questions), my primary intraoperative goal is to lower the BP to prevent cardiac and neurologic
complications, while avoiding levels of hypotension that could cause end-organ hypoperfusion.
Before induction, I would insert an AL and gradually lower the BP to at least 180-160/110-90.
Because it is unclear what BPs is absolutely best (a 20% reduction is probably not enough) or
28 The Essential Oral Board Review

what end-organs might be at risk (incomplete W/U with emergency case), these numbers are
somewhat arbitrary and not fixed. (Knowing baseline BPs would be helpful to determine
allowable perioperative BP limits.) My first choice of antihypertensives is SNP because it is a
reliable, potent, titrateable, inexpensive and arteriolar vasodilator (although other acceptable
choices include NTG and diltiazem; the patient is too bradycardic for esmolol). In addition to
the AL, I would also insert a CVP before induction to assess volume status (bleeding and HTN
may have contributed to intravascular volume depletion). Induction should be slow and
controlled to avoid precipitous declines in BP and unmasking hypovolemia. (I expect to see
hypotension on induction and would be prepared with IVF and vasoactive agents.)
Intraoperatively, I would attempt to maintain a SBP of 160-150/80-60, but the actual target BP
all depends on the ECG, surgical needs, and depth of anesthesia. Even with adequate
analgesia, it may become necessary to continue antihypertensives and AL monitoring in the
recovery period. Since the malignant levels of HTN could lead to CHF, pulmonary edema,
myocardial ischemia, intracranial bleeding (e.g., from a previously unrecognized aneurysm), I
expect these complications to manifest perioperatively, with or without concurrent HTN.

b. CAD: Assuming we had to proceed with surgery despite incomplete cardiac workup, I would at
least want to know the patient's exercise tolerance to assess contractility. This affects my
choice of drugs and whether more invasive/involved monitoring would be required (e.g., if she
had a 15% EF, I may more strongly consider placing a PAC or request intraoperative use of a
TEE by a qualified specialist). Assuming any impairment was moderate to severe (or if no
information on EF were provided), my goal is to minimize severe hypertenison, hypotension,
tachycardia, or bradycardia, all of which could exacerbate myocardial ischemia. (I expect all
could occur.) Consistent with the management of HTN above, I would perform a slow
controlled induction. (If I were unable to obtain these monitors, I would still proceed with my
slow induction plan and set the NIBP to the stat mode.) If the EF were very low and the case
prolonged, involving hemodynamic instability/fluid shifts, I may ask a qualified colleague to
insert and monitor a TEE intraoperatively. (Assuming the EF is not 15%), I would induce
anesthesia with a combination of etomidate, fentanyl, sevoflurane, and rocuronium, carefully
titrated to avoid hypotension. (If it were 15%, I may consider a slow controlled "cardiac"
induction with high dose narcotic and extubation may be less likely.) Vasoconstrictors and
inotropic agents should be available. There is no apparent reason to keep the patient
intubated at the end of the case, so if it proceeded smoothly, I would plan to extubate her then.
If her CAD were moderate to severe, p-blockers may need to be continued postoperatively to
reduce the chances of adverse cardiac events (e.g., Ml, CHF, ischemia).

c. LBBB: Monitoring of ischemia may be difficult with a LBBB, but I am not sure it justifies
placement of a PAC, which is an unreliable monitor for ischemia (and may, rarely, produce a
RBBB, leading to complete heart block). A high degree heart block should be considered for
any intraoperative bradycardia or hypotension. A transthoracic pacer should be available at all
times.

d. PVD: Since the case is an emergency, complete assessment of any PVD may be difficult.
Intraoperatively, I would avoid extreme hypotension due to the concern of unrecognized PVD
and the need for higher-than-normal perfusion pressures (e.g., to the brain, retina).

2. Hematologic
Most clinicians would treat a PT of 1.4 in an actively bleeding patient. Coagulopathy from
extensive surgery and transfusion requirements could develop intraoperatively, requirining blood,
platelets, FFP and cryoprecipitate.

3. Renal
Depending on the severity of the stenosis (still unknown are the BUN/Cr, severity of RAS, and
results of previous renal workup), maintenance of a higher renal perfusion pressure may be
necessary to assure adequate GFR (e.g., MAP 60-80). In addition to the typical prernal, renal,
and postrenal cause of oliguria, inadequate glomerular filtration pressure from the RAS (a
29 The Essential Oral Board Review

prerenal cause) may need to be considered. Adequate hydration status should be maintained (as
assessed be PE, l/Os, VS, CVP, and U/O). Although dopamine, mannitol, and furosemide are
commonly given to treat oliguria, they have not been shown to affect the perioperative incidence
of acute or chronic renal failure.

4. Intravenous Drug Abuse


The anesthetic implications of substance abuse depend on the substance in question. If cocaine
were taken intranasally, its effects may last for up to 6 hours and lead to intraoperative
hypertension, myocardial ischemia, dysrhythmias, hyperthermia, seizures, and stroke. If
narcotics were used chronically, it may make the patient tolerant to opiates (possibly worsening
the HTN). A cental line may solve the problem of difficult IV access. Blood and body fluid
precautions should be implemented as it is for all cases. Victims of a needle-stick injury should
have their tetanus and HBV vaccinations reviewed. Depending on the results and HIV/HBV lab
studies from patient and victim, the victim may need hepatitis B and/or tetanus immunoglobulins
and vaccinations. They should also be counselled and given the choice of receiving HIV
prophylaxis with triple therapy (e.g., Zidovudine, Lamivudine and Indinavir), preferably within an
hour of exposure.

At some point, the question of regional anestheisa may be raised. I would strongly consider it for
intra- and postop analgesia if the patient were resistant to narcotics (this may partly explain the
presenting HTN). RA also has the benefits of avoiding GA, providing hemodynamic instability,
and helping to control the HTN. However, the patient has an elevated PT, and albeit less than
1.5X control, it still presents concern for neuraxial bleeing. Of course, the PT will probably be
corrected with FFP for surgery. But the time it would take to obtain and administer FFP and place
the epidural may be unacceptable to the surgeon if the bleeding were active. (A tourniquet may
staunch the bleeding, but may also cause pain and worsen the HTN). Furthermore, any pain and
anxiety associated with epidural placement may exacerbate the already dangerous HTN. BP
control during neuraxial technique may be required by an assistant watching the AL and tritrating
SNP. Finally, hypovolemia and anemia may lead to precipitous hypotension with any single shot
technique. A high level of sympathectomy may exacerbate any hypovolemia and anemia
present, and predispose to intraoperative hypotension. So while not contraindicated, if any RA
were to be used, an epidural or continuous spinal, both slowly titrated to a lumbar level, would be
my techniques of choice. In fact, if postop pain control were the main goal, a single shot of
intrathecal Duramorph may be the simplest, least invasive, least harmful alternative. Any time RA
is performed, low dose local anesthetics should be used for postop infusions, frequent neuro
checks must be employed to detect any new onset deficits, a stat MRI/CT performed to R/O
neuraxial hematoma bleeding, and a surgical team must be available to operate on the
hematoma, ideally, within 8 hrs of symptom onset.

At the end of the case, mental status may be effected by baseline condition, residual drug effect,
hypertensive stroke, hypoperfusion (e.g., BP too low in face of cerebral vascular disease).

5. Medications
Patients' medications are usually withheld during surgery. If unexplained tachycardia
developed and her next dose of propranolol were due, a p-1 selective p-blocker could be given.
P-blockers could also be given if the CAD were moderate to severe. If the surgery were
uneventful and the patient extubated at the end of the case, her usual medications could be
started postoperatively. If the patient had severe RAS, I would first make sure the captopril
were not ordered in error.

6. NPO Status
Even thought the patient could have a full stomach, unless she were vomting feculent material, I
would prefer a slow, controlled induction over a RSI or awake intubation due to the concern of
precipitous HTN or hypotension on induction. Cricoid pressure, gentle mask ventilation, and
Trendelenberg position can be used to minimize the chances of regurgitation/aspiration. An NG
30 The Essential Oral Board Review

or OG tube should be placed during the case and her stomach emptied as much as possible.
Even so, she remains an aspiration risk and should be extubated awake if possible.

Of course, no one thinks or writes in paragraph form on the exam. We think and write in sentence
fragments, which can be written down in outline form or organized into a spreadsheet. Personally, I
prefer the spreadsheet approach because having to look at each box makes me more conscientious
about addressing all aspects of perioperative care. Others find it less useful because it is difficult to
read during an exam. (My usual response: when talking to an examiner, you should not be reading
anyway!)

Labs:
CBC, pit, PTT CV Heme IVDA Kidney Meds NPO
Chem 7, LFTs a. HTN - severe if a. INR 1.4 Which? Severity Captopril - Asp.
CXR real! Baseline?, -cause RAS?
v'Drug cough, prophyl
Hepatitis, HIV/drug Cause?, Sx? Rx warfarin? screen, BUN/Cr, ?good for
panel to prevent Rx: FFP, others? bilat. RAS.
hepatitis/HIV
Any cardiac, renal MI/CVA: AL, ^CBC,
studies panel. may Propron -
SNP. Avoid 44 pit, PTT, s IV access. need needed for
BP, watch vol. LFT. If narc use, t B P. CAD?
b. CAD/MI - b. Hct 26- consider ->vol, HCTZ-
Severity? Exerc. could be epidural or IT No drug obligate
Preop Problem/ tol? EF? Rx: CV 4er with Duramorph prevents diuresis?
Severity stability, rx HTN!! severe ARF,
Continue BBs? bleed, CRF.
c. LBBB - New? rehydrate.
Sx? Pacer! T/C 2U.
d. PVD - involved?
IBP, avoid HTN, Prepare Type; watch TBP if Remember Prevent
achieve CV stability, for large hyperthermia severe Possible asp.
watch unmasked blood dysrhythmias RAS BB
Goal hypovol. Arbitrarily loss. myocardial withdrawal.
before induction: ischemia
180-160/110-90. with cocaine
If HTN severe, AL + Freq H/H U/O RSI or
CVP, I BP before awake
Plans: induction, RBBB
Monitor with PAC. TEE if I
EF, big case.
SCI - etomidate, RA if narc Defer to
Plans: fent, sevo, roc. resist, PT & CV
Induction surg OK. prioriies

CV > NPO risk because [stroke, Ml, and CHF] > [aspiration] (less likely). Account for
Priorities asp, with prophylaxis, eric, pr., gentle mask vent., and T-berg.
Balanced: fentanyl,
Intraop Plans: sevoflurane,
Mainten rocuronium.
No monitors: still
SCI; very low EF:
Backup high dose "cardiac"
Plans induction and
maintenance.
HTN, hypoTN, Bleeding, Cocaine: Oliguria: TH:need Extubate
brady, tachyC, CVA, possible myocardial check more BB awake
Anticip'd CHF, Ml DIC ischemia, BP,
Cx dysrhthm, CVP,
seizure, perf
stroke, t°C, pressure
withdrawal

V
.
3 1 The Essential Oral Board Review

Memorized Responses

Rote memorization is not an essential Consultant attribute. One wonders, then, why there is
so much emphasis on memorization from "experts" in other courses. The obvious answer: They do
not understand the oral Boards.

There are several real dangers in advocating a memorization approach to the orals. First, it
gives examiners the impression that you are reading a script or a list, cannot think on your own, and
cannot make your own Judgments or Adapt. Examiners may feel that this mindless, robotic way of
thinking and speaking is how you appear to others. (Is this also the way you practice anesthesia?)
Second, by frantically trying to dig up memorized responses when you should be engaged in
problem-solving and conversation, you give failing (or fail to give) meaningful responses. Canned,
generic responses, which you may not even retrieve in time, do not really address specific questions.
Third, trying to memorize an exact answer to every oral Board question is folly- not because exact
answers are too numerous, but because exact answers do not even exist! It is infinitely more
preferable to learn how to think on your feet and hold a conversation. Finally, a memorization
approach misrepresents and trivializes the oral Board process, which is only partly about
memorization. The oral Board exam is mainly about the way you think, solve problems, respond to
new situations, organize your thoughts, and commmunicate with others. By spending your time
memorizing speils, you miss out on a golden opportunity to become a perioperative Consultant.

Nevertheless, rote memorization does have a limited place. Just as clinical treatment
algorithms serve as useful guidelines for patient management, so can memorization of certain facts,
concepts, and approaches guide you towards the an acceptable test response. This is not the same
as memorizing the entire answer. Instead, it is laying down a framework on which you can organize
thoughts and phrase answers. Thus, if memorization is to be used, it is only with the ultimate goal of
facilitating Judgment, Adaptability, and Communication.

The next section contains several answers that you will be tempted to memorize. Please do
- and remember that your memorized response will probably never be used verbatim during a real
exam. The Sample Responses can be cited verbatim when problem is generic. Verbatim responses
may also be useful, when, after exhausting all possible specific diagnoses, the problem persists
(perhaps you have missed something). However, your actual Board question will be based on a
specific case scenario, the details of which often make parts of the memorized response incorrect or
incomplete. For example, you would not "check the pulse oximeter and ETC02" if a person passed
out and turned blue at a restaurant - most restaurants do not carry pulse oximeters or capnograms.
Similarly, if a patient came into the emergency room had a knife sticking out of his head, "intracranial
foreign body" would be high in your list of reasons for altered mental status, even though it was not
on your memorized list. As you learn to apply the algorthims, you will learn to quickly add and
subtract items that either do or do not belong, respectively. For better, not for worse, memorization
does not relieve you of the need to think.
32 The Essential Oral Board Review

Sample Responses to Adverse Events

1. Hypoxia
a. First I, would check the patient's pulse oximeter and color to make sure the hypoxia was real.
b. Then, I would look at the presence and pattern of expired C02 to make sure the patient was
being ventilated and no airway obstruction was occurring.
c. Next, I would hand ventilate with 100% 02, listen for bilateral breath sounds, look for symmetric
chest wall excursion, and check the patient's peak airway pressure and tidal volume.
d. Depending on what I found, I would consider suctioning the ETT, looking down the ETT with a
fiberoptic bronchoscope, and/or obtaining a CXR.

2. Hypotension
a. First, I would confirm the BP value by palpating the pulse at 2 separate sites and repeat the
NIBP reading.
b. At the same time, I would look at his other vital signs, including HR, saturation, ETC02, and
temperature. Depending on my findings, I may consider chest auscultation and
examination.
c. Then, I would look for a specific cause.
• I would look at the ECG for heart rate rhythm, and ST segment changes.
• I would examine the surgical field for signs of bleeding or IVC compression.
• All drugs given to the patient and relevant history should be reviewed.
• If present, invasive monitors should be checked for filling pressures and CO.
d. Critically low BPs should be treated acutely with epinephrine or ephedrine if there were
bradycardia and phenylephrine or atropine if there is tachycardia.
e. An initial fluid bolus may also be indicated unless it is strongly suspected that fluid will
worsen CHF.
f. CPR is necessary for life-threatening hypotension.

3. Hypertension:
a. First, I would first make certain the value was real by repeating a noninvasive reading and
flushing the AL, if one were present.
b. Then, I would make certain there was no change in the patient's oxygenation and
ventilation by looking at the pulse oximeter, patient's color, and capnogram.
c. Next, I would determine the heart rate to determine whether there was tachycardia,
bradycardia, or dysrhythmias.
d. If the hypertension were critically high, I could initiate treatment with a drug such as
nitroglycerine, nitroprusside, labetolol, nifedipine, or nicardipine. However, I prefer to treat
the exact cause, the possibilities for which include hypoxia, hypercarbia, pain, fluid
overload or unintended pressor administration.
e. The need for acute treatment depends on whether the BP is a baseline condition or
causing any harm (such as ischemia, dysrhythmias or bleeding).

4. Hypercarbia
Hypercarbia is due to an increase in C02 production or a decrease in C02 elimination.
a. First, I would check the other vital signs, including the pulse oximeter, HR, BP, and
temperature, to make sure the condition was not an emergency.
b. Next, I would hand ventilate with 100% 02, listen for bilateral breath sounds, look for
symmetric chest wall excursion, and check the peak airway pressure and tidal volume.
c. Causes of increased production include fever, shivering, MH, neurolept malignant
syndrome, seizures, thyroid storm, high carbohydrate diet - all hypermetabolic conditions.
d. Causes of decreased elimination include improper ventilator settings, leaks or obstruction
in the breathing system, rebreathing in the circuit, changes in lung compliance or function
(e.g., bronchospasm, mucous plugs, pneumonia, aspiration, right mainstem intubation, and
a patient fighting the ventilator).
e. After hyperventilation, other specific therapy depends on making an exact diagnosis.
33 The Essential Oral Board Review

5. Tachycardia
a. First, I would palpate the pulse to determine if the tachycardia was real and look at the
ECG to determine the underlying rhythm. I would also determine its relative significance by
determining the baseline HR.
b. Next, I would make certain there was no change in the patient's oxygenation and ventilation
by looking at the pulse oximeter, patient's color, and capnogram.
c. Tachycardia can be due to hypoxia, hypercarbia, pain, anxiety, hypovolemia, anemia,
fever, endogenous catecholamines, exogenous sympathomimetics, anticholinergics,
dysrhythmias, or pacemaker malfunction (if present).
d. If immediate therapy were required (as with worsening myocardial ischemia), but the exact
cause still unknown, I may consider administering esmolol, a short-acting beta-blocker.

6. Bradycardia
a. First, I would look at the ECG to determine the underlying rhythm and palpate the pulse to
determine if the bradycardia was real. I would also determine its relative significance by
determining the baseline HR.
b. Next, I would make certain there was no change in the patient's oxygenation and ventilation
by looking at the pulse oximeter, patient's color, and capnogram.
c. Bradycardia can be due to hypoxia, vagal or parasympathetic reflexes (e.g., from
hypertension), drug effect (e.g., acetylcholinesterases, beta-blockers), pacemaker failure,
or a baseline condition (e.g. athlete, acquired, or congenital heart block).
d. If immediate therapy were required (as with hypotension) but the exact cause still unknown,
I would consider using atropine, ephedrine, epinephrine, isoproterenol, followed by
transcutaneous or intravenous pacing.

7. Delayed Emergence
Delayed emergence can be due to problems with ABCs, medications, an adverse neurologic
event, or metabolic conditions.
a. First, addressing the ABCs, I would make certain the patient's oxygenation, ventilation, and
vital signs were acceptable by looking at the patient's color, pulse oximeter, HR, BP,
temperature, and capnogram (hypocarbia removes the drive to breathe and hypercarbia
can cause C02 narcosis).
b. Then, I would review the patient's exposure to medications, including neuromuscular
blockers (twitch stimulator), residual inhalational agent (sampling exhaled gases),
intraoperative drugs (type and dose of narcotics, benzodiazepines, and ones continuously
infused), and premedications (like scopolamine and droperidol).
c. Next, I would examine his pupils and review the chart for conditions that could affect mental
status, such as seizures, or a stroke (embolic, ischemic, hemorrhagic, or thrombotic).
d. If no diagnosis were apparent, I would consider metabolic causes and consider obtaining
an ABG, electrolytes, glucose,
e. If all else failed, I may consider a head CT and/or a neurologic consult.
f. In the mean time, the patient may need to have his airway controlled and protected, and
ventilation controlled or assisted.

8. Oliguria
The causes of oliguria can classified as prerenal, renal, or postrenal.
a. First, I would quickly rule out postrenal causes by checking the Foley catheter, if one is in
place, and palpating the bladder.
b. Then, I would evaluate prerenal causes by assessing the possibility of hypovolemia,
vasoconstriction, or mechanical restriction of renal blood flow. Vital signs and any
hemodynamic or respiratory parameters should be used to help investigate these
possibilities. If aortic surgery had been performed, the possibility of emboli should be
considered, even though no specific therapy may exist.
34 The Essential Oral Board Review

c. Renal causes include intrinsic renal injury from ischemia, toxins, or mechanical damage. I
would review the anesthetic record and history for hypotensive episodes, antibiotic or dye
exposures, transfusion reactions, possible muscle injury, and blunt or penetrating trauma.
d. Laboratory studies are usually not immediately helpful. Inspection of the urine may
suggest the presence of blood, heme, or myoglobin. A BUN, Cr, and fractional excretion of
Na+ are not helpful in the setting of acute injury since changes in these values take hours to
develop. If renal vein or artery thrombosis are considered, a renal ultrasound and urology
consult may be needed.
e. If immediate therapy were required but the exact cause still unknown, the patient's volume
status and hemodynamic stability would should be assured. I would consider administering
a fluid challenge. Mannitol, furosemide, and dopamine may also be given, but without
proof that they can prevent renal failure from occurring. They may, however, prevent total
anuria, decreasing the need for later dialysis.

9. Jaundice
The causes of jaundice can be classified as prehepatic, hepatic, or posthepatic.
a. Prehepatic causes are due to the increased production of bilirubin from a hemolysis or
absorption of a hematoma.
b. Hepatic causes are due to hepatic injury from ischemia; hepatotoxic drugs including
antibiotics (tetracycline, isoniazid, rifampin, sulfonamides), phenothiazines, alpha-
methyldopa, salicylates, acetaminophen, anabolic steroids, oral contraceptives, and
alcohol; intrinsic disorders (Gilbert's is most common, Dubin-Johnson produces a
conjugated hyperbilirubinemia, Crigler-Najjar is fatal in childhood); infections; hepatic
congestion; and rarely halothane hepatitis.
c. Posthepatic causes are due to mechanical obstruction, such as a stone, stricture, or
atresia.
d. The most useful way to manage jaundice is to obtain a thorough history and physical exam,
concentrating on perioperative events (such as hypotension or recent transfusions), recent
medications, and physical findings (such as hepatomegaly or signs of coagulopathy).
e. Important labs may include LFTs (including conjugated and unconjugated bilirubin),
alkaline phosphatase (for obstruction), a H/H (for hemolysis), and possibly abdominal
ultrasonography.

10. Nausea
Nausea occurs from a variety of stimuli acting on the chemotrigger zone in the medulla.
a. The most important causes to rule out and treat are hypoxia and hypotension.
b. Other important and readily treatable causes include pain and anxiety, narcotics,
movement, and vagal stimulation (e.g., visceral traction).
c. Acute treatment consists of eliminating specific causes. Effective medications include
butyrophenones (droperidol), phenothiazines (promethazine, perchloperazine), selective
5-HT3 antagonists (ondansetron), and even propofol.

11. Routine Monitoring and Induction


a. First, I would place monitors on the patient, including an ECG, NIBP, and pulse oximeter.
b. Then, I would preoxygenate the patient with 100% 02. Small amounts of fentanyl and
midazolam could be titrated at this time to treat anxiety and facilitate induction.
c. Next, I would then administer an induction dose of propofol, test my ability to mask
ventilate, administer rocuronium and additional fentanyl, and deepen anesthesia with
sevoflurane.
d. Once an adequate level of anesthesia and neuromuscular was achieved, I would perform
direct laryngoscopy and intubate the trachea. Proper endotracheal positioning would then
be confirmed by auscultation and capnography.
35 The Essential Oral Board Review

Differential Diagnoses

1. Hypoxia
a. Low inspired 02 concentration
b. Hypoventilation
Central or obstructive causes with spontaneous ventilation
Mechanical causes with controlled or spontaneous ventilation
Shunt-V/Q mismatch
Atelectasis
Mucous plugs
Pneumo-/hemo-/chylothorax
Pleural effusion
Pulmonary contusion
Endobronchial intubation
Pulmonary edema
Pneumonia/pneumonitis
Aspiration
Bronchospasm
Pulmonary embolism
Inhibition of hypoxic pulmonary vasoconstriction
Decreased MV02
Intracardiac shunt
Pulmonary arterio-venous fistulae
During one-lung ventilation
d. Increased diffusion barrier
e. Baseline condition

Hypercarbia
a. Increased production
Fever
Excessive carbohydrate intake
Malignant hyperthermia
Neurolept malignant syndrome
Thyroid storm
Bicarbonate administration
Release of tourniquet or aortic crossclamp
b. Hypoventilation
• Significant dead space ventilation
Large pulmonary embolus
Significant hypotension
• Not usually one lung ventilation
c. Rebreathing
• Incompetent inspiratory/expiratory check valve
• Exhausted soda lime
• Inadequate flow with Mapleson system
36 The Essential Oral Board Review

3. Wheezing
a. Lower Airway
Kinked tube
Mucous plug
Herniated cuff
Foreign body
Endobronchial intubation
Pneumothorax
Bronchospasm
Pulmonary embolism
Cardiogenic
Anaphylaxis
Aspiration
Carcinoid
Baseline condition
Upper Airway
Laryngeal edema
laryngospasm
Laryngomalacia
Foreign body
Vocal cord paralysis
Infection
Tumor
Polyps
Baseline condition

Hypotension
a. Hypoxia
b. Hypercarbia (very late)
c. dysrhythmia
Bradycardia
Tachycardia
Nonsinus rhythm
Asystole
Pacemaker failure
d. Decreased afterload
Vasodilation
Hypovolemic shock
Septic shock
Anaphylactic shock
Neurogenic shock (high spinal or cord injury)
Decreased preload
All types of shock above
Aortic crossclamp or unclamping
Pneumothorax
Embolism (pulmonary, air, fat, amniotic fluid)
IVC occlusion
Tamponade (high airway pressures)
37 The Essential Oral Board Review

Impaired myocardial function


Ischemia
Acidosis
Hypocalcemia
Cardiomyopathy
Electrolyte disorder (hypocalcemia)
Valvular heart disease
Congenital heart disease
Vegetation or myxoma
I. Carcinoid crisis
m. Addisonian crisis
n. Drug effect
o. Erroneous value
p. Baseline condition

Hypertension
a. Hypoxia
b. Hypercarbia
c. Inadequate anesthesia
d. Pain
e. Anxiety
f. Drug withdrawal
g. Increased ICP
h. Hypervolemia
i. Bladder distention
j. Pheochromocytoma
k Thyroid storm
I. Malignant hyperthermia
m. Carcinoid
n. Drug effect
• Sympathomimetic (e.g., cocaine)
• MAOI + Demerol
• Pheochromocytoma + droperidol
o. Erroneous value
p. Baseline condition

6. Tachycardia
a. Hypoxia
b. Hypercarbia
c. Inadequate anesthesia
d. Pain
e. Anxiety
f. Drug withdrawal
• Clonidine, ETOH, etc.
g. dysrhythmia
• SVT, AF, A flut, VT, PAT
• Electrolyte
h. Hypovolemia or hypotension
i. Fever
j. Malignant hyperthermia
k. Pheochromocytoma
I. Thyroid storm
m. Carcinoid
n. Myocardial ischemia
o. Bladder dissension
38 The Essential Oral Board Review

p. Drug effect
• MAOI + Demerol ^
• Pheochromocytoma + droperidol <*
• Pancuronium
q. Erroneous value ^
• Counting T waves m.
r. Baseline condition

Bradycardia m
a. Hypoxia <
b. Hypercarbia (late) m
c. Anesthetic overdose ^
d. dysrhythmia v
. AV B m
• Sick sinus syndrome
• Pacer malfunction
• Electrolyte
e. Myocardial infarction (especially with IWMI)
f. High spinal or spinal shock
g. Va g a l r e fl e x m
Hypertension (autonomic h y p e r r e fl e x i a ) N<-
Oculocardiac T
Visceral traction m
Right atrial dissension v
Increased ICP *
h. Bladder dissension m
i. Hypothermia V
j. Suctioning of airway (especially with pediatrics) **
k. Drug effect ^
Opiates s^
Beta blocker ^
Calcium channel blocker
Potent inhalational agents
Anticholinesterase inhibitor
Succinylcholine
I. Erroneous value
m. Baseline condition

Altered Mental Status/Delayed Emergence


a. ABCs
• Hypoxia
• Hypercarbia (C02 narcosis)
• Hypocarbia (insufficient C02 stimulus)
b. Medication effect
• Premedications (sedatives, scopolamine, droperidol, benzodiazepines)
• Central anticholinergic syndrome (atropine, scopolamine, organophosphates, TCA)
• Neuromuscular blocker
• Anesthetic (inhaled, intravenous)
• Usual medications (narcotics, sedatives, tranquilizers, lithium, reserpine, clonidine,
alpha-methyldopa, steroids, amphetamines, etc.)
• Substance abuse (alcohol, cocaine, LSD, heroin, etc.)
c. Endocrinologic/metabolic
• Hyponatremia
• Hypocalcemia
• Hypoglycemia
• Hypothermia
39 The Essential Oral Board Review

DKA
Hepatic encephalopathy (chronic problem)
Renal encephalopathy (chronic problem)
Hypothyroidism (chronic problem)
Addison's Disease (chronic problem)
Cushing's Disease (chronic problem)

Neurologic
• Ictal or post-ictal state
• CVA (ischemic, thrombotic, embolic, hemorrhagic)
• Cerebral edema
Baseline condition

9. Agitation
a. ABCs (see item 8 above)
b. Medication effect (see item 8 above)
• Withdrawal (alcohol, narcotics)
c. Pain, anxiety, discomfort, disorientation, distended bladder
d. Movement disorder (Parkinsonism, metoclopramide, droperidol)
e. Baseline condition

10. Oliguria
a. Prerenal
• Hypovolemia
• Hypotension
• Hypoperfusion
• Renal vein or artery thrombosis
b. Renal
• ATN (ischemia; toxin - dye, abx, myoglobin, hemoglobin)
• Intrinsic disease (vascular, glomerular, thromboembolism, interstitial nephritis)
c. Postrenal
• Obstructed catheter
• Urethral or uretal obstruction
d. Baseline condition

11. Nausea/Vomiting
a. Pain
b. Hypoxia
c. Hypotension
d. Vagal
e. Drug
• Anesthetic agents (narcotics, inhalation agents, etomidate)
■ Chemotherapy
• Other
f. Surgery
g. Anxiety
h. Obstruction
i. Infection
j. Baseline condition
4U The Essential Oral Board Review ^

/*$
Consultant Phrases vfffS
v.

Even though relying on memorized answers is not to be recommended, certain patterns of ^


speech (reflecting certain patterns of thought) begin to emerge with repeated mock oral practice. *v.
The following phrases are among the most common. As you study them, bear in mind that they are **
not meant to represent gimmicks or shortcuts. Quite the opposite, when used properly, they reflect ^
the essential attributes of a Consultant described previously. As you become more accustomed to (*
their use, your responses should become less short and choppy and more continuous and smooth- /%
flowing. Eventually, different phrases will blend together as seamless, effortless explanations, as ^
they may seem to do in the following examples. Q

1. Always State Why C


When: Any question, any time. £*
H o w : B e f o r e / d u r i n g a r e s p o n s e , p r o v i d e y o u r r a t i o n a l e a s p a r t o f y o u r a n s w e r. \
Example: "I would perform an awake intubation because the patient has a difficult airway and a C
full stomach..." /«

2. Offer Factual Information (^


When: Any question, any time. /*
H o w : B e f o r e / d u r i n g a r e s p o n s e , p r o v i d e f a c t u a l i n f o r m a t i o n a s p a r t o f y o u r a n s w e r. ^
Example: "Because patients with severe HTN are often hypovolemic, I would hydrate the Q
patient before proceeding..." j*.

3. State Yo u r Goals ( *■
When: Any question, any time. ^»
H o w : S t a t e y o u r g o a l s a s p a r t o f y o u r a n s w e r, e s p e c i a l l y b e f o r e y o u r p l a n . v
Example: "My goals for induction are to prevent bronchospasm in this asthmatic patient. (*
Therefore, I would...." ^

4. Provide Perspective /«
When: Any question, any time. ^
H o w : B e f o r e / d u r i n g a r e s p o n s e , s t a t e y o u r p e r s p e c t i v e a s p a r t o f y o u r a n s w e r. C
Example: "This is a bleeding patient who is critically ill, for whom surgery is life-saving and ^
should proceed immediately..."

5. Making Choices
When: Asked to make a choice.
How: State whether that alternative would be acceptable or not, and, most importantly,
why.
Example 1: "Propofol is not my first choice for an induction agent because it is more likely to
cause hypotension, an important consideration in this hypovolemic, elderly patient.
That's why I chose etomidate, which causes less cardiovascular depression ..."
Example 2: "Propofol would be acceptable, but because the patient is elderly and possibly 4>-
hypovolemic, I would use a low dose titrated to effect..." £

6. R i s k / B e n e fi t s \
When: Any question, any time, especially if asked to defend a decision. /"
How: State the risks and benefits, including which is more important, defending your plan. \
Example: "The risk of performing a rapid sequence induction is losing control of the airway if C
intubation were unsuccessful. The benefit is minimizing the risk of aspiration. /*
Because it does not seem like it would be difficult to intubate the patient's airway, but \
the patient is actively vomiting, a rapid sequence induction is my first plan for t
induction..." s*
41 The Essential Oral Board Review

7. Paraphrasing with Perspective


When: Asked to respond to a set of conditions.
How: Repeat the conditions with perspective.
Example: "A BP of 80/40 is low, but critically low in this patient with elevated ICP due to the
concern of inadequate cerebral perfusion pressure, defined as MAP minus ICP...

8. Resolving Ambiguity
When: The situation is ambiguous because of inadequate information
How: Clarify the situation using "It depends" and "If...then" statements.
Example: "Whether or not I would proceed with surgery in patient whose morning glucose level
was 250 mg/dl depends of urgency of the case. If the case were an emergency,
then I would proceed. If, however, the case were elective, then I would delay..."
(continuing...)
ulf\ had to proceed, I would administer 10 units of insulin and repeat a blood sugar
reading in half-an-hour..."

9. Lists
When: You have several items to cover in succession.
How: Use "First...Second..Third...Finally"
Example: "I have several preoperative concerns. First, the patient's blood pressure is 220/110
and should be under better control. Second, his HR is 140, suggesting his beta-
blocker dose may be inadequate, or a concomitant problem may be present. Third,
he recently discontinued his clonidine, which may help explain the tachycardia and
hypertension. Finally, he reports new onset chest pain, suggesting that we should
delay surgery to rule-out myocardial ischemia or infarction..."

10. Best Guess


When: You're not sure, but feel you need to offer some explanation.
How: Admit your uncertainty, and continue with your best guess.
Example: "I'm not sure what aprotinin does, but I think it inhibits fibrinolysis and platelet
activation ..."

11. Retraction,
When: When it becomes clear that you committed a gross error in patient management and
that error is so egregious that you feel you might fail if it were left uncorrected or
unaddressed.
How: When the situation arises and only if appropriate (Judgment), state your realization
that your previous plan was in error, and follow-up with why, how you would handle
the complication, and what choice would have been preferable.
Example: "Before continuing, I would like to modify my response in which I said I would use
succinylcholine for induction. That was in error because giving a depolarizing agent
to a quadriplegic patient could cause a hyperkalemic arrest. If it occurred, I would
administer calcium, administer insulin and glucose, hyperventilate, consider giving
bicarbonate, and prepare for CPR. A better choice would have been rocuronium,
which works almost as fast, but is a nondepolarizing agent..."
Example: "You'll have to excuse me. I would like to rephrase my previous response. It should
have been obvious to me that performing awake laryngoscopy on a child with
epiglottitis was a poor choice because it can cause laryngospasm. I should have
performed an inhalation induction. I realize that the patient has a full stomach. But
the goals of maintaining spontaneous ventilation and avoiding airway stimulation in
this awake patient takes priority over the risk of aspiration. Since laryngospasm has
now occurred and the patient is already cyanotic, I would administer succinylcholine,
perform direct laryngoscopy, and try to intubate while the surgeon prepared for an
emergency tracheostomy.."
42 The Essential Oral Board Review

12. Prototypical Response


When: Anytime you have to state your plan for a complex case with conflicting goals for
which a risk/benefits analysis is necessary.
How: State what you would do, why you would do it, the other alternatives, why you didn't
choose them, the problems with your choice, and how you are prepared to deal with
them.
Example: "For this patient with an unruptured abdominal aortic aneurysm, I would choose a
slow controlled induction with fentanyl and isoflurane titrated over several minutes. I
chose this technique because my primary goal is to achieve a deep level of s®^
anesthesia prior to intubation, thereby avoiding hypertension and, hopefully,
aneurysm rupture. I realize that since the patient has reflux symptoms, I could have
performed a rapid sequence induction. But the symptoms were mild, the patient was
given aspiration prophylaxis, and I plan to apply cricoid pressure and ventilate with
low peak airway pressures during induction. Should vomiting occur, I would place *m*
the patient in Trendelenberg's position, suction the oral pharynx, intubate the
trachea, suction the trachea, then look down the trachea with a fiberoptic
bronchoscope. On the other hand, should hypertension occur during laryngoscopy, I
would stop and deepen the level of anesthesia before proceeding with intubation. If
a dangerous surge in BP occurred, as detected on an arterial line, I would give a 40-
50 mc bolus of nitroprusside.

<"f%

<r§\

^ts

%'tSjfc

ft
43 The Essential Oral Board Review

Examiner Strategies

For many candidates, especially those taking the exam for the first time, it is not the questions
themselves but the way they are asked that is so surprising and difficult. Examiners are interested in
how you function in everyday practice. Consequently, they may ask questions indirectly in the form
of provocative case scenarios, as they occur in real life. The following discussion illustrates some of
the ways such questions may appear. They are presented not to trivialize or oversimplify the Board
examination process, but to help candidates better understand what to expect from the exam. After
reviewing and reflecting upon this section, the candidate should be less surprised and distressed by
the way questions may appear. He/she may even wish to develop his/her own specific strategies to
deal with certain types of questions.

1. Distractors
Description: Asking questions seemingly unrelated or only peripherally related to the case.
Advice: Answer the question as best you can; do not lose focus on the case. You are
unlikely to fail if this is the only question you miss.
Example: In a case involving pneumonectomy, the first question is, "What is the shunt
equation?"
Response: If you know it, 'The quantity pulmonary capillary 02 content minus arteriolar 02
content divided by the quantity 02 content minus mixed venous 02 content." If you
don't know it, "I don't recall."

2. Minutiae
Description: Asking for extremely detailed information
Advice: Answer the question as best you can; do not lose focus on the case. You are
unlikely to fail if this is the only question you miss. Sometimes, examiners ask
detailed questions when you've gotten everything else right.
Example: "What is the P02 of the mitochondria?"
Response: "If you know it, "1-2 mm Hg." If you don't, "I don't know."

3. Forced Response
Description: Insisting or strongly suggesting you should proceed when it would be unwise or
you are not prepared to do so; any ambiguous situation; or asking for the best
technique.
Advice: State your concerns, then answer the question with an "If...then" statement. There
are no best techniques, only ones with advantages and disadvantages.
Example 1: As the first question in a session 2 case (no prior preop evaluation), you are asked,
"Your patient is on the OR table. How would you induce anesthesia?"
Response 1: "This patient scheduled for a total knee replacement and recently had warfarin
discontinued. Ideally, I would have liked to check the PT/PTT and if normal,
discussed the possibility of an epidural beforehand. But if she has already agreed
to GA and her preoperative condition is optimized, then I could proceed with a slow
controlled induction using fentanyl, midazolam, thiopental, vecuronium, and
isoflurane."
Example 2: You are asked as an additional topic question, "Do you think morbidly obese
patients should be done as outpatients?"
Response 2: "It depends on the presence of comorbid disease, type of case, and arrangements
for home care. If she had no sleep apnea or other major comorbidity, the case
were peripheral, she had assistance at home, and lived in the vicinity of a hospital,
outpatient surgery may be an option. If some of these conditions are not met,
perhaps it would be safer to admit her for 23 hour observation."
Example 3: "What is the best drug for induction?"
Response 3: "There is no best drug for induction. But because of this patient's asthma, I would
chose ketamine for its sympathomimetic bronchodilating properties."
44 The Essential Oral Board Review

4. Turning the Tables


Description: Asking why your initial answer is wrong or a poor choice.
Advice: Answer the question. You should realize that every choice has a pro and a con, a
benefit as well as a risk.
Example: After you respond that you would use remifentanil, you are asked "Why is
remifentanil such a bad drug?"
Response: "I wouldn't call it a bad drug. I chose it for this bronchoscopy case because it has a
fast onset and short duration. However, it has several disadvantages, including
chest wall rigidity, the need for administration as an infusion, and no prolonged
effect for postop pain."

5. Asking Why
Description: The examiner asks you "Why do you want to know?" in response to a question
you asked.
Advice: Never ask a question, except to repeat something you did not hear or understand.
Asking makes it seem like you're trying to evade the question. If you have a
question, use "If...then" or "Assuming that..." statements.
Example: If you are asked to induce anesthesia in a patient with known hypertension, but for
whom the preoperative BP was unknown.
Response: Instead of asking,"What is the BP?" state, "Assuming the BP were under good
control, I would proceed with..."

Imaginary Persons
Description: Asking what you would say to imaginary persons (patient, family member, medical
student, nurse, surgeon, hospital administrator, or colleague) or how you would
respond to their recommendations.
Advice: A person's status should not affect how you treat him/her or interpret his/her
comments, which may be valid or invalid. All explanations should be given as
though you were speaking with an intelligent medical student.
Example: 'The surgeon says you shouldn't place an epidural. How would you respond?"
Response: "I would ask his reasons why. Perhaps he knows of some contraindication of
which I am unaware. If not, I would explain the advantages of an epidural,
including the ability to avoid intubation, decreases in intraoperative blood loss, and
superior postoperative pain control."

7. Numbers Game
Description: Asking you to set a numerical threshold for treatment for action or varying a
numerical value until your plans are altered, then asking why.
Advice: Although normal ranges for clinical parameters exist, there are usually no absolute
standards of care establishing cutoffs for clinical action. The significance of a
particular numerical value depends on its cause, symptomatology, chronicity, and
need to proceed with surgery. Having said this, a decision must often still be made
based on a number, the cutoff for which may be based on a commonly cited value.
Example: "Would you cancel the case with a K* of 3.5? 3.3? 3.0? 2.5?"
Response: 'There is no absolute cutoff for canceling the case. It depends on the cause and
chronicity of the hypokalemia, associated symptomatology, and whether or not the
surgery was urgent. Moderate levels of chronic hypokalemia that have been
previously well tolerated probably do not increase the risk for intraoperative
dysrhythmias. However, I would seriously consider postponing surgery if the level
was below 2.5. Because that value is somewhat arbitrary, it is not a rigid cutoff.
It's just that with lower values, I become more concerned about dysrhythmias..."
45 The Essential Oral Board Review

8. Displays of Intimidation
Description: Intimidating displays of anger, boredom, condescension, collusion, derision,
disgust, frustration, or rudeness.
Advice: Become oblivious to presumed examiner affect. Remember, you are being
interviewed for a job as a Consultant. Part of a Consultant's description of duties is
to behave professionally, even in the face of adversity. Besides, your response to
stress may actually be what is being evaluated. Conversely, you may be
misreading or overreacting to the examiner. Never laugh, snicker, or make light of
the situation.
Example: "I can't believe you would actually do that, doctor!" or "That was a stupid answer!"
Response: Calmly, without expression, "That was my best first response. There are pros and
cons with every plan. Please allow me to review the advantages and
disadvantages of mine.

9. Probing Deeper
Description: Asking, "What else?"
Advice: Usually, it means your answer is incomplete. Reassess the situation, make sure
you understand the situation correctly, make sure you mentioned everything you
thought you did, and try invoking an answer from the Sample Responses to
Adverse Events.
Example: After you said you would check the capnogram, hand ventilate, listen to breath
sounds, and inspect the airway pressures and tidal volumes in the management of
a cyanotic patient, you are asked, "What else?"
Response: "This is a cyanotic patient in a new operating room. Assuming everything that I
mentioned was in order, I would also check the Fi02 to rule out a problem with 02
supply."

10. Testing Resolve


Description: Asking, "Are you sure?"
Advice: Ask yourself, "Am I sure?" If you are, say so. If you're not, admit it. Either way,
explain as much as you can, using previously discussed Consultant skills.
Example: "Are you sure 50/30 is a normal blood pressure for a premature neonate?"
Response: "I'm fairly certain, but to make sure, I would look it up or consult with a colleague
before proceeding. It's important because if it was low, fluids or pressors may
have to be given."
46 The Essential Oral Board Review

Candidate Strategies

How you think makes all the difference. Objectively review the following to see which ones might
make a difference for you.

1. Monitor Body Language


a. Notice how people dress, sit, and position themselves. What earns your respect? What do
you find disrespectful?
b. Sit up straight. Plant your feet on the floor. Fold your hands in your lap. Practice this posture.
Can you sustain it for 35 minutes in your format attire? (It can be quite uncomfortable.)
c. Look people in the eye when you talk. If you can't, look at their glasses, nose, or hairline.
Chances are, they won't notice, and it's better than staring at the paper or your feet.
d. Avoid taking notes when you talk, if possible. (After all, you don't usually take notes when
conversing with surgeons.)
e. Avoid distracting movements (shaking, rocking, pen clicking, etc.).
f. Practice appearing and being calm and composed. It takes practice, but can be learned.

2. Control Verbal Language


a. If trying to sound eloquent makes you nervous or tongue-tied, talk normally, and be
conversational.
b. If you're at loss for words or ideas, learn to either say "I don't know" with a straight face or think
aloud logically. Never babble. Avoid "uhms" and "ahs."
c. Answer the question asked, not the one you think was asked. This means you must listen to
the question carefully.

3. Expect the Worse


a. Bad things happen - even to good people, with the most carefully laid plans, with the best of
intentions.
b. Expect complications. The oral Board exam is all about complications, so don't take it
personally or presume that you made a mistake when they occur. They're an inherent part of
the test.
c. Expect the unexpected. Don't be (or let it be known that your are) surprised by anything. This
includes all types of distracting, picky, probing, offbeat, and nonsensical questions. It also
includes any type of intimidating or outlandish behavior.
d. Expect to get questions wrong. If you don't have to say "I don't know" at least once during the
exam, you've probably missed something. When you do miss a question (or 2 or 3), forget
about it and move on. Belaboring the past impairs your ability to deal with the next question.
The same principle hold true in moving from session 1 to session 2.

4. Set the Pace


a. If examiners seem rushed with their questioning, you may be more deliberate and insert a
thoughtful pause when needed. You may a take a brief time to think before you speak
b. But if you take too long, the examiners will move on, and the question will be counted as
wrong. You may fail from this reason alone.

5. Define the Situation


a. When asked to respond to a new situation or changing condition, start by defining or redefining
the situation and stating its significance.
b. When things go awry, you are confused, or you can't find the answer, going back to basics.
Restate your understanding of the situation. Review your goals. Check the vital signs,
monitors, and ABCs.
4 / The Essential Oral Board Review

6. Be Ordinary
a. You've trained, studied, practiced, and prepared thoroughly. You've taken care of patients,
both healthy and sick, without complications. Now on the exam, just do what you would in
your everyday practice (assuming your practice is competent and sound).
b. Don't do anything extraordinary or heroic, just because you recently read about it or just to
impress the examiners (like indiscriminately insert a TEE or try an exotic drug or technique).
There has to be a valid reason (as there sometimes is for inserting a TEE).
c. Remember, just because you can't or refuse to do something doesn't mean you're not
responsible for being able to discuss it.

7. Take Control
a. You should realize that you can never really control the exam; only examiners have ultimate
control. But that doesn't mean you shouldn't try to direct the course, pace, and tone of the
exam.
b. The way you try to "take control" is by talking more, not less, using Consultant skills.
c. When you are able to talk at length about a familiar topic, applying knowledge, specifying
goals, assigning priorities, weighing risks and benefits, identifying and resolving conflicts,
anticipating complications, and delineating backup plans, people will stop and listen.
d. When people stop and listen, they can't ask questions. (Some would call this control.)

8. Talk More, Not Less


a. The best candidates neither fear discussing anesthesia nor loath being asked questions about
it. They speak with ease and seem eager to share their knowledge. It has been said
examiners can tell whether someone is going to pass or fail in the first 5 minutes of an exam.
If so, it is usually those who speak with alacrity that make the best early impressions.
Unbelievable though it may seem, some candidates actually enjoy the exam.
b. On the other hand, if you don't like to talk, it shows. If you can't wait until it's all over, it also
shows. If you are shy and reticent, you invite interrogation. All these forms of weakness are
highly provocative.
c. It takes hard work for examiners to have to extract information from a candidate who's
withdrawn and reluctant to speak, even if they're on your side. Even the most patient
examiner can become frustrated, explaining the impatience sometimes seen on their part.

9. Be Pleasant
a. If you are hostile, defensive, argumentative, or brittle, you can fail no matter how smart you
are. This is because you may appear inflexible, communicate poorly, or give the impression
that you would represent the ABA poorly.
b. Allow being cutoff. Do not argue with the examiner. Do not blame in any way or use
inflammatory language in front of the examiner, such as, "You didn't make it clear to me." (In
other words, leave your ego behind.)

10. Use Positive Imagery


a. When handling a perioperative problem, imagine yourself dealing with the event in the preop
area, OR, or PACU. Take care of things just as you would in real life.
b. Imagine the examiners, not as adversaries, but as wanting to help. Imagine they are honest,
straightforward, and just want to know what you know (but are unable to give positive
feedback).
c. Imagine yourself talking to a medical student - someone who has less knowledge than you,
not some professor who knows 10 times as much.
d. Maintain a positive self image - it becomes you.
4b The Essential Oral Board Review

Reasons for Failure

The ABA lists six deficiencies exhibited by candidates during the oral examination that may
account for their failure. All have been addressed in previous sections, but are mentioned again
here to reinforce the writings of the ABA. Even though several additional reasons for failure may
exist (e.g., hostility, excessive nervousness, failure to understand the questions, poor communication
skills, etc.), the list below should suffice since almost all others can be grouped in one or several of
its categories.

1. Superficial knowledge
a. The oral exam in not a test of cognitive information or recall.
b. However, if the candidate's amount or ability to recall information is so inadequate that
subsequent discussion of anesthetic management cannot proceed in a meaningful manner,
failure may result.

2. Inability to Apply Knowledge


a. If the candidate's information base is adequate but isolated from clinical relevance, failure may
result.
b. Such may be the case if the knowledge cannot be utilized to analyze or manage clinical
problems (e.g., knowing normal PFT values, but failing to use abnormal results to alter
anesthetic technique, anticipated complications, or postoperative care.)

3. Inability to Adapt
a. If the candidate is able to handle routine anesthesia care (e.g., one-lung ventilation during a
thoracotomy) but is unable to efficiently and effectively adjust to a complication or unusual
response in a normal patient (e.g., hypoxemia during the same case), failure may result.

4. Inability to Express Ideas or Defend a Viewpoints in a Convincing Manner


a. There may be many ways to interpret and act on data or manage anesthesia in particular
situations.
b. However, if the candidate cannot express his/her point of view in a coordinated, rational
manner, failure may result.

5. Faulty Judgment
a. There are often several approaches to patient management.
b. However, if the candidate selects ones that are clearly dangerous, failure may result.

6. Excessively Slow and Deliberate Answers


a. If the candidate responds so slowly that an adequate exam cannot be given, failure may result.
49 The Essential Oral Board Review

Useful Studying Tips

1. Assess Your Specific Needs and Practices


a. Everyone is different. Identify your specific weak points and goals. They should become
apparent after the course.
b. Determine what you need to do to overcome those weak points and achieve those goals. If
your problem is speaking, consider speaking classes. If your problem is knowledge, study. If
your problem is fear, practice until you are completely desensitized and taking an oral exam is
like your everyday practice sessions.

2. Dedicate Time for Studying


a. Assess you daily routine. Eliminate unnecessary distractions. Make a schedule and stick to it
- no excuses! This may involve stopping off at the library everyday before coming home or
unplugging the TV. Remember, the sacrifice lasts only a few days; the reward lasts a lifetime.
b. One hour a day is often not enough. You have to not only read and understand, you have to
explain. As the test approaches, additional time should be dedicated to speaking aloud and
taking mock orals.

3. Do Not Waste Time


a. Procrastination is like an addiction - the more you do it, the worse it gets. The most important
thing you can do is to start your preparation now - this very moment - and break the vicious
cycle of delay. Every time you think about doing something later, you miss the chance to get it
done now. So the next time you think of a question or recognize a deficiency, look it up, seek
help, and remedy the problem immediately. Wait until tomorrow, and it won't get done.
b. Time is too precious to waste. Become efficient. If you studied a topic one night, but find
you're forgetting it the next day, plan to review. Make note cards. Carry these cards, or our
textbook, with your everywhere. If you have to rehearse an explanation, do so wherever you
can.

4. Study Actively, Not Passively


a. Emphasize explanation, not familiarity. After covering a chapter, ask yourself, "Can I explain
the major concepts to a medical student?" If not, you've wasted your time.
b. Quiz yourself as you read. Ask yourself, "How can I use what I just read to make a clinical
decision, justify a course of action, or solve a difficult problem?" Pretend you were an examiner
trying to devise an interesting question. (This is the best way to understand what kind of
questions are likely to be asked.)

5. Practice at Work
a. Challenge everything you do on a daily basis. Am I setting up the machine correctly? Why
am I using propofol instead of thiopental? What if the patient had a class 4 airway? Is there
another way? What problems might arise with my technique? How am I prepared to deal with
them? What if I gave phenylephrine instead of pancuronium? How would I treat hypotension
and hypoxia if they occurred? What if the patient's pacemaker fails? What if the peak airway
pressures suddenly begins to rise? What if the bypass machine fails? What if the line
isolation monitor goes off? How could I make my outpatient surgicenter more efficient?
b. If you fear public speaking and an opportunity to speak in public arise (e.g., lectures, in-
services), take advantage of it.
c. Begin to speak more to your colleagues, coworkers, and patients as a Consultant.

6. Practice With a Partner


a. Consider obtaining a study partner, even if it's on the phone. Partners may be used to clarify
confusion, practice questioning, and offer support. Even if you end up doing all the teaching
and begin to think the partnership is becoming one-sided, recall that teaching is what
Consultants do. You may be getting more out of it than you think.
50 The Essential Oral Board Review

b. Take as many mock oral exams as you can. Call on trusted colleagues. Return to your
residency program for a few days and have former attendings administer exams. (It doesn't
matter whether you liked them as a resident. The service they're providing is one in which
friendship and well-intended sympathy may get in the way.) We recommend you take at least
10-15 complete mock orals before the actual exam; depending on your needs, you may need
at least 20-30. The more, the better.

7. Do the Daily Verbal Workout (DVW)


a. The DVW is talking at length about anesthesia - you plans, you goals, your priorities,
pathophysiology, essentially any quality topic of discussion. It lasts one hour, and can be done
by yourself, in front of a mirror, on video or audio tape, to a friend, or with a colleague. A mock
oral exam counts.
b. In the months preceding the exam, you should begin speaking, gradually increasing the
frequency and duration of your discussions. As the Exam approaches, you should perform
several DVW per week. In the month before the exam, you should perform the DVW at once
a day.
c. The DVW is not meant to replace studying, which must continue independent of the DVW.

8. Use the Course Material


a. Follow the instructions in the preface of the text on how to quiz yourself. Unlike any general
textbook, The Essential Oral Board Review is meant to be a home tutor. To make the most of
it, pick a Basic Concepts section to study. Next, read an accompanying question from
Consultant Q/A section. Keep the "answer" completely covered. Verbalize your response fully
and completely - resist the temptation to peek at the answer. Only after fully completing your
answer may you read the text response. Repeat the process for the same question until you
can respond without hesitation. You are not only mastering information, you are rehearsing
your Consultant Communication skills.
b. Have friends, family, and/or colleagues ask you the questions in the book. They need not be
anesthesiologists, just able to read. (The more interrogating they can be, the better. Ask them
to interrupt frequently with, "Are you sure?", "Why?", and "What do you mean?")

9. Plan Your Preparation as You Would for a Difficult Case


a. With a difficult anesthetic case, you would thoroughly assess the patient's baseline condition,
review the surgery and skills of the surgeon, set up your equipment, monitor the progress of
the case, ensure the availability of help, and devise back up plans. Why should you treat your
preparation for the exam any differently?
b. Thoroughly plan your study time and location, have help available, monitor your progress with
exams and questions, and devise back up plans for study in case you become busy. Most
importantly, do not panic.

10. Think Positive


a. Do not be discouraged by past failures, missed deadlines, the seemingly endless sea of
information you must cover, the difficulty you are having with comprehension and recall, or
the apparent ease with which others seem to master the material. Everyone learns at their
own pace, and everyone can pass.
b. Every minute of study is another topic better covered, every topic covered makes you a
better clinician and Consultant, every step taken towards becoming a Consultant is a step
towards earning Board Certification.
c. Just learn to take one step at a time - beginning now!

No goal of any significance is ever achieved without significant sacrifice.


J I The Essential Oral Board Review

Mock Oral Exams

The rest of the workbook contains mock oral exams used in the course. Note that each
case is presented as Session 1 or Session 2, corresponding to the two types of exams presented
in the real Board exam. After the course, try taking these tests at home in their entirety. Although
the "answers" are not provided as they are in the text, you may find the search for responses the
most useful exercise of all.
52 The Essential Oral Board Review

Cardiovascular I (Session 1)

A 52 year old male with a gunshot wound to the chest presents for emergency thoracotomy.

HPI: Pt claims he had just walked out of a bar, where he had been drinking with his friends
and using cocaine, when he was shot in the chest. Two 14 gauge IVs were started by
the paramedics, who then administered 2 liters of normal saline. During transport to the
emergency room, his systolic BP was approximately 90 mm Hg. The pt seemed
intoxicated but never lost consciousness.
v
PMI: Intravenous drug abuse and hepatitis several years ago. He has had multiple gunshot
wounds to the chest and abdomen. He claim to have had limited exercising ability and
chest pain several times a day.

PE: P 60, BP 60/40, R 30, T 35.4°C. Healed median sternotomy and abdominal incision
scars is seen. Chest auscultation reveals distant heart tones. He is confused and
complaining of chest pain. His airway appears normal.

CXR: Wide mediastinum and cardiomegaly.

ECG: Sinus bradycardia, ST segment depression in leads V2-6.

Labs: Hgb 12 gm/dl, electrolytes are normal, platelets 150 K, and INR 1.4. LFTs are pending.
53 The Essential Oral Board Review

Cardiovascular I (Session 2)

A 52 year old male with a gunshot wound to the chest presents for emergency thoracotomy. He
is confused and complaining of chest pain. A healed median sternotomy scar is seen. P 60, BP
60/40, R 30, T 35.4°C. His CXR shows a wide mediastinum.
54 The Essential Oral Board Review

Cardiovascular I (Questions)

A 52 year old male with a gunshot wound to the chest presents for emergency thoracotomy. He
is confused and complaining of chest pain. A healed median sternotomy scar is seen. P 60, BP
60/40, R 30, T 35.4°C. His CXR shows a wide mediastinum.

A. Preoperative Evaluation
1. ACLS: What are your priorities in managing this patient? Do you require an ECG?
2. Hypotension: How to Rx? How do you know what specific treatment to use? What is
the significance of his HR? Should you give atropine? Pacemaker?
3. Prior CABG: Significance of scar? How would history of prior CABG affect your
management?
4. Labs: What labs do you require?
B. Intraoperative Management
1. Monitor: Which monitors? CVP or PAC? Why/why not?
2. Tamponade: The CVP is 30. Significance? DDx? How would you approach the
problem? Is there a pathognomonic finding for tamponade? The blood pressure
decreases >10 mm Hg with spontaneous inspiration. What is this called? Mechanism?
What does the CVP look like with tamponade?
3. Induction: What are the principles of management of a patient with tamponade? What
drugs would you use for induction? Ketamine vs. etomidate?
4. Maintenance: Which drugs would you give for maintenance? Are you concerned about
recall? How does scopolamine work?
5. Tachycardia: After intubation, the HR increases to 120. Your response?
6. Ischemia: Which organs are at greatest risk for ischemia? Are you concerned about
myocardial ischemia? What makes the heart the greatest organ at risk? What can you do
to prevent it?
C. Postoperative Care
1. Angina: The patient complains of chest pain. What would you do? How could you tell
whether it was cardiac in origin? Would you Rx with NTG?
2. Bradycardia: The HR is still 60. What would you do? At what point would you call for a
cardiology consult?
56 The Essential Oral Board Review

Cardiovascular II (Session 1)

A 55 year old 80 kg, 57" male is scheduled for a right total knee replacement.

HPI: Pt is a former football player with a history of degenerative joint disease of the lumbar
spine and knees.

PMI: Hypertension and insulin-dependent diabetes mellitus. He had a Ml 8 months ago,


smokes heavily, and complains of several episodes of chest pain occurring every week.
Both feet have been numb for several years. He takes propranolol, enalipril, aspirin,
NPH 40 q AM, 20 q PM, and nitroglycerine PRN.

PE: P 94, BP 150/86, R 20, T 36°C. Only part of his uvula can be seen on airway exam. He
has an 8 cm thyromental distance and slightly reduced range of motion of his neck. The
rest of his exam is unremarkable.

CXR: No apparent disease.

ECG: NSR with RBBB.

Labs: Hct 35. PT/PTT are normal. Electrolytes are remarkable for a BUN of 25 and creatinine
of 1.8.
57 The Essential Oral Board Review

Cardiovascular II (Session 2)

A 55 year old 80 kg male is scheduled for a right total knee replacement. He has hypertension
and diabetes mellitus, had a Ml 8 months ago, smokes heavily, and complains of several
episodes of chest pain occurring every week. He takes propranolol, enalipril, aspirin, NPH 40 q
AM, 20 q PM, and nitroglycerine PRN. P 94, BP 150/86, R 20, T 36°C, Hct 35, EKG - NSR with
RBBB.
58 The Essential Oral Board Review

Cardiovascular II (Questions)

A 55 year old 80 kg male is scheduled for a right total knee replacement. He has hypertension
and diabetes mellitus, had a Ml 8 months ago, smokes heavily, and complains of several
episodes of chest pain occurring every week. He takes propranolol, enalipril, aspirin, NPH 40 q
AM, 20 q PM, and nitroglycerine PRN. P 94, BP 150/86, R 20, T 36°C, Hct 35, EKG - NSR with
RBBB.

A. Preoperative Evaluation
1. Cardiac: What is the significance of his prior Ml? Of his RBBB? If he had an Ml, why are
no Q-waves found on EKG? How will you determine whether additional tests are
indicated? When would you need to see a persantine thallium test vs. a dobutamine echo
test? Why dobutamine, not dopamine echo?
2. Medications: Why is he taking enalipril? What does propranolol do? Is he adequately B-
blocked? Does nitroglycerine cause steal?
3. Diabetes: How does his diabetes affect your management? Does if affect your
monitoring technique?
B. Intraoperative Management
1. Monitors: What monitors would you select? Which EKG leads and why? What is the
principle behind the operation of the NIBP cuff? Does he need a PAC or a CVP? What
special risks are associated with placement of a PAC?
2. Anesthetic Choices: Which is better, regional or general? What are the
contraindications for regional? The patient insists on regional. Would you agree? Would
you order a bleeding time? What is the significance of a BT?
3. Intravascular Injection: During injection of your epidural catheter with 2% lidocaine, the
patient passes out and has a seizure. Your response? After intubation, the blood
pressure is 180/95. Your response? Could this be due to hypoxia? How? Would you
cancel surgery? The surgeon states it was just an intravascular injection, the local
anesthetic is now gone, the patient is already anesthetized, and wishes to proceed.
4. Anesthetic Choices: Which is better for his heart condition, propofol or fentanyl? Would
ketamine be inappropriate? Which inhalational agent is best? Does N20 cause
cardiotoxicity?
5. ST Segment Depression: ST segment depression occurs in lead 2. Is this unusual?
What would you do? What factors determine ischemia?
C. Postoperative Care
1. Extubation: Would you extubate this patient deep? Discuss the risks vs. benefits.
2. Ml: Does this patient need to be monitored in the ICU postoperatively? The surgeons ask
if you would like to rule out an Ml? Can this be done on the floor? On your way home 2
days later, you meet the patient's floor nurse in the elevator. She informs you that new Q-
waves were found in leads 2, 3, and aVF on postop day 2. What would you do? Why
should this patient now be in the ICU?
60 The Essential Oral Board Review

Cardiovascular, CPB, Monitors III (Session 1)

A 68 year old, 70 kg, 5'0" female is scheduled for repair of an abdominal aortic aneurysm.

HPI: Pt noted the onset of severe back pain 12 hours ago. Abdominal/thoracic ultrasound
reveals a 10 cm suprarenal aortic aneurysm.

PMI: History of hypertension and congestive heart failure. Echocardiogram revealed an


ejection fraction of 30%. Her medications include taking captopril, alpha-methyldopa,
and nitroglycerin PRN.

She has a 40 pack-years of smoking history.

PE: P 110, BP 130/85, R 24, T 36.7°C. Pt is moaning with severe back pain. Airway exam is
normal. Auscultation of the lungs reveals bibasilar crackles. Cardiac exam is
unremarkable. The abdomen is obese. There is no pedal edema. Peripheral pulses are
palpable.

CXR: Mild cardiomegaly, tortuous aorta.

ECG: Q waves in leads II, III, and aVF.

Labs: Hgb 12 gm/dl, platelets 250 K, PT/PTT and electrolytes are within normal limits. ABG
reveals pH 7.29, pC02 40, p02 78 on room air.

The surgeons want to perform surgery as soon as possible on what they are calling a leaking
aneurysm.
61 The Essential Oral Board Review

Cardiovascular III (Session 2)

A 68 year old 70 kg, 5'0" female with a 12 hour history of severe back pain presents for repair of a
leaking abdominal aortic aneurysm. She has a history of hypertension, congestive heart failure,
and 40 pack-years of smoking. She is taking captopril, alpha-methyldopa, and nitroglycerin PRN.
Her EKG shows Q waves in leads II, III, and aVF. P 110, BP 130/85, R 24, T 36.7°C, Hgb 12
gm/dl.
62 The Essential Oral Board Review

Cardiovascular III (Questions)

A 68 year old 70 kg, 5'0" female with a 12 hour history of severe back pain presents for repair of a
leaking abdominal aortic aneurysm. She has a history of hypertension, congestive heart failure,
and 40 pack-years of smoking. She is taking captopril, alpha-methyldopa, and nitroglycerin PRN.
Her EKG shows Q waves in leads II, III, and aVF. P 110, BP 130/85, R 24, T 36.7°C, Hg 12
gm/dl.

A. Preoperative Evaluation
1. Cardiovascular: Is it important to obtain a preoperative cardiac history? Would you
order an abdominal ultrasound to determine the size of the aneurysm? What are your
hemodynamic goals? Is this a good blood pressure for her condition?
2- Ischemic heart disease: The cardiologist says this patient is at an increased risk for
postoperative Ml based on Goldman's criteria? Would you agree?
3. Antihypertensives: How does Alpha-methyldopa work? Nitroglycerine? Should any of
them be continued intraoperatively?
B. Intraoperative Management
1. Monitors: Would you place an AL before induction? Why? A PAC? She cries out in
pain with placement of an AL. Would you continue?
2. Induction: Would you use an intrathecal or epidural catheter for this case? How would
you induce general anesthesia? What if she had a difficult airway?
3. Hypotension: Following intubation, 2% isoflurane is administered and the AL reads BP
50/25. Your response? After 50 mc of neosynephrine, the BP reads 0. Your response?
The surgeon begins immediate CPR. Would you give crystalloids, blood, or pressors?
4. Placement of the aortic X-clamp: After clamping the aorta, the BP increases to 170/90.
Is this acceptable? What do you think would happen to the CO? What would you do?
What if ST segment depression also occurred? What causes ST segment depression?
What is the J-point?
5. Spinal cord ischemia: What organs are at greatest risk for ischemia? Why? Can
anything be given to protect the spinal cord? (SCPP, drain CSF, intrathecal papaverine,
LA/LV/Ao to FA bypass). Would you give steroids? If an epidural catheter had been
placed, could it have been used to worsen or lessen spinal cord ischemia?
6. Release of the aortic X-clamp: What preparation would you make before release?
Would you administer bicarb? After clamping, the pressure decreases to 80/50. Why?
Your response? The ETC02 decreases from 35 to 25. Is this expected?
C. Postoperative Care
1. Renal failure: What complications would you expect? Postoperative urine output is 20
cc over 2 hrs. How would you manage? Would a serum BUN and creatinine help in your
management?
2. Paraplegia: The patient is unable to move her legs after extubation. The surgeon is
certain it was an unavoidable complication of aortic cross-clamping and hypotension. Do
you agree? What else might be the cause? How would you evaluate?
64 The Essential Oral Board Review

Cardiovascular IV (Session 1)

A 66 year old 60 kg, 5'3" female presents for skin grafting for a 96% total body surface area burn.

HPI: Pt was rescued from a fire of her trailer home, where she was found unconscious. She
was resuscitated with fluids, given 100% 02 by face mask, and is now awake but
confused.

PMI: Pt has severe mitral stenosis from a childhood case of rheumatic fever. She was
hospitalized for congestive heart failure four months ago. At that time, an
echocardiogram revealed a valve area of 1 cm2, left atrial enlargement, and an ejection
fraction of 70%. Her pulmonary artery pressures thought to be moderately elevated. She
decided to delay mitral valve surgery until several months later. She is taking digoxin,
lasix, and Coumadin.

PE: P 100 (irregularly irregular), BP 110/60, R 36, T 35.6°C. The airway in normal.

CXR: Mild cardiomegaly, left atrial enlargement. Moderate pulmonary edema.

ECG: Atrial fibrillation with RBBB.

Labs: Hct 31, K+ 4.5, PT/PTT 25/33. ABG on 4 liters/min face mask pH 7.31, pC02 49, p02 78.

The nurses bring the pt to the operating room from the ICU and connect your monitors for you.
65 The Essential Oral Board Review

Cardiovascular IV (Session 2)

A 66 year old 60 kg, 5'3" female with mitral stenosis presents skin grafting for a 96% total body
surface area burn. She was hospitalized for congestive heart failure four months ago. Her valve
area is 1 cm2. She is taking digoxin, lasix, and Coumadin P 100 (irregularly irregular), BP 110/60,
R 36, T 35.6°C, Hct 31, K+ 4.5, PT/PTT 25/33.
66 The Essential Oral Board Review

Cardiovascular IV (Questions)

A 66 year old 60 kg, 5'3" female with mitral stenosis presents skin grafting for a 96% total body
surface area burn. She was hospitalized for congestive heart failure four months ago. Her valve
area is 1 cm2. She is taking digoxin, lasix, and Coumadin P 100 (irregularly irreqular) BP 110/60
R36,T35.6°C, Hct 31, K 4.5, PT/PTT 25/33.

A. Preoperative Evaluation
1. Mitral stenosis: What are your concerns about this patient? What is the Gorlin
equation? (area = flow / sq root of pressure gradient) Is it important to know? Is this
severe MS? What are your hemodynamic goals for management? What is the usual
function of the LV with MS? What is the significance of the HR?
2. Burns: What is the difference between 2nd and 3rd degree burns? Which is more
painful? How do the coexistence of MS and a 96% TBA burn affect your management?
3. CO poisoning: The pulse oximeter reads 100%. Are you surprised by this value? Could
she have methemoglobinemia? Would you administer methylene blue as a diagnostic
test?
4. Coagulation: Would you discontinue her Coumadin before surgery? How does Coumadin
work? Do her PT/PTT suggest an adequate Coumadin effect? Do you need to see a
platelet count?
B. Intraoperative Management
1. Monitors: What monitors would you like? What would a PCWP show? Would you
expect to see a-waves? What does a V wave mean on the PCWP tracing? What PCWP
would you use to estimate LV filling pressures? What if you saw a large CV wave on the
CVP? Do you think it is justified to place a PAC with continuous SV02 monitoring
capabilities? After placement, the SV02 is 85%. Interpretation?
2. Pulse oximetry: The patient's fingers and toes are blackened from burns. Where could
you monitor pulse oximetry? What if you were unable to obtain a signal anywhere? How
often would you send blood gases?
3. Induction: What are your goals for induction? Would ketamine meet your goals? Is it
safe to give succinylcholine? Does it depend of the BSA? What is the mechanism of the
hyperkalemia?
4. Maintenance: What drugs would you use for maintenance? Why?
5. Fluids: The surgeon asks whether you would run her dry or wet? How do you know
how much is necessary to allow flow across the MV? How is % surface area
approximated? (head/arms = 9%, legs/front/back = 18%, neck/groin = 1%) How do you
calculate perioperative fluid requirements? (crystalloid = 4ml/k/%) Would you give
colloids? How much? (After 24 hrs, colloid 0.3-0.5 + 1.5 crystalloid + 2L D5W)
6. Tachycardia: The HR is 120. What are the pros and cons of using a p-blocker to treat
the rate?
C. Postoperative Care
1. Pulmonary edema: The nurse calls you for a PaC02 of 60. What would you do? Would
you administer NTG for pulmonary edema? The BP is 70/30 and the HR is 110. Would
you administer neosynephrine? Do you think the pulmonary edema will affect her work of
breathing?
2. Hyperkalemia: The K+ is 5.5. How would you like to lower it? The PIP is 45 and ETC02
47. Would you hyperventilate? Would you give lasix?
6o The Essential Oral Board Review

Cardiovascular V (Session 1)

A 46 year old 80 kg, 6'1" male is scheduled for a hemicolectomy for a small bowel obstruction.

HPI: Pt has had a week of abdominal pain and 3 days of nausea and vomiting. Abdominal
films reveals distended loops of bowels with air-fluid levels.

PMI: Hypertension for 7 years. He had an uncomplicated Ml 7 months ago. He now has a
pattern of angina at rest one to two times a week, which his internist claims has been
optimally treated with aspirin, propranolol, clonidine, and furosemide.

PE: P 88, BP 160/90, R 14, Hb 13, Temp 37°C. His airway exam is normal. His lungs are
clear to auscultation. His cardiac exam reveals a regular rate with no rubs, clicks, or
murmurs. His abdomen is distended and tender. There is no peripheral edema. His
pulses are strong and symmetric.

CXR: Normal cardiac silhouette.

ECG: Normal sinus rhythm with left axis deviation and LVH.

Labs: Hgb 16 gm/dl, K+ 3.3 mEq/L, C02 30 mEq/L. Platelets, other electrolytes, and PT/PTT
are within normal limits.
69 The Essential Oral Board Review

Cardiovascular V (Session 2)

A 46 year old 80 kg. 6'1" male is scheduled for a hemicolectomy for a small bowel obstruction.
Past medical history is significant for hypertension. He had an Ml 7 months ago and now has
angina at rest. His medications include aspirin, propranolol, clonidine, and furosemide. P 88, BP
160/90, R 14, Hb 13, Temp 37°C, Hgb 16 gm/dl, K+ 3.3 mEq/L.
70 The Essential Oral Board Review

Cardiovascular V (Questions)

A 46 year old 80 kg. 6'1" male is scheduled for a hemicolectomy for a small bowel obstruction.
Past medical history is significant for hypertension. He had an Ml 7 months ago and now has
angina at rest. His medications include aspirin, propranolol, clonidine, and furosemide. P 88, BP
160/90, R 14, Hb 13, Temp 37°C, Hgb 16 gm/dl, K+ 3.3 mEq/L.

A. Preoperative Evaluation
1. Cardiac Evaluation: Why does this patient have chest pain? What factors control
coronary blood flow? What do you want to know about his cardiac function? How would
you assess his anesthetic risk? What is a persantine thallium test? Is this the best test to
perform to assess risk? Is he ready for surgery? What do you think about his HR?
Should he receive more P-blocker before surgery? What questions would you ask his
cardiologist?
2. Management of aspirin: What effect does his aspirin have on coagulation? How would
you assess its effect? What is the significance of an elevated bleeding time? Would you
discontinue the aspirin?
3. Management of hypokalemia: What do you think about his K? Why is it low? Should it
be corrected? How would you decide?
B. Intraoperative Management
1. Monitors: What monitors are needed - AL, CVP vs. PAC? What additional information
can you obtain from a PAC? What is the significance of a preinduction PAWP of 22 mm
Hg? Would you cancel the case? How would you monitor for ischemia? What if you
were unable to insert an AL?
2. Induction: What drugs would you use for induction? What are your goals? Is thiopental
safe? Ketamine? How does the history of hypertension affect your technique? What are
the risks of hypertension?
3. Maintenance: What drugs would you administer for maintenance? Why? Is a high dose
narcotic technique better than an inhalational technique? Which inhalational agent is
best? Does it matter which muscle relaxant you use?
4. Management of hypertension: Following intubation, the BP rises to 180/90.
Significance? Causes? Rx?
5. Management of hypotension: 1 hour into the case, the blood pressure gradually
decreases over 10 minutes to 70/40. Significance? Causes? Rx?
6. Extubation: What are your extubation criteria?
C. Postoperative Care
1. Delayed emergence: After 1 hour in the PACU, the nurse call you stating that the patient
still does not wake up and asks if you think pyridostigmine would be a good idea. Your
response?
2. Pain control: What is you plan for pain management? Would you place an epidural?
(anticipation) What are the criteria for and contraindications against its insertion.
3. Hypertension: BP 180/98. DDx? Rx? What class of drug is clonidine? Would alpha-
methyldopa be an adequate substitute?
72 The Essential Oral Board Review

Cardiovascular VI (Session 1)

A 60 year old 50 kg, 5'4" female presents for repair of a fractured left hip.

HPI: Pt lives alone and was found 2 days after she had fallen. She was found awake but ^
d i s o r i e n t e d , a n d w a s a d m i t t e d d i r e c t l y t o t h e h o s p i t a l fl o o r . ^

PMI: The pt has been seen in clinic for Raynaud's phenomena several times over the past 10 ^
years. A chart review reveals no cardiac complaints. Her lifestyle is sedentary. During ^
the first day of her present hospitalization, an echocardiogram was ordered to evaluate a Q
3/6 systolic ejection murmur discovered on exam. It revealed moderate aortic stenosis ^
w i t h p r e s e r v e d LV f u n c t i o n . S h e h a s a 4 0 p a c k - y e a r s m o k i n g h i s t o r y. < c

She is receiving no medications except IM meperidine for pain. ~

PE: P 70, BP 150/80, R 20, Temp 36.4°C. A 3/6 systolic ejection murmur is audible over the /**
right upper sternal border. Her left hip is bruised and extremely tender to palpation. ^

CXR: Normal. n

ECG: NSR, LVH.

Labs: Hct 25, Na+151 mEq/L, BUN/Cr 22/1.5. Other electrolytes are normal.

The surgeon's feel they need to operate today due to their stated concerns of fat embolism and
deep venous thrombosis. V.
73 The Essential Oral Board Review

Cardiovascular VI (Session 2)

A 60 year old 50 kg, 5'4" female presents for repair of a fractured left hip. She has a 40 pack-
year smoking history, Raynaud's phenomena, an a 3/6 systolic ejection murmur patient over the
right upper sternal border. She has no cardiac complaints, but is sedentary. P 70, BP 150/80, R
20, Temp 36.4°C, Hct 25. The ECG shows LVH.
74 The Essential Oral Board Review

Cardiovascular VI (Questions)

A 60 year old 50 kg, 5'4" female presents for repair of a fractured left hip. She has a 40 pack-
year smoking history, Raynaud's phenomena, an a 3/6 systolic ejection murmur patient over the
right upper sternal border. She has no cardiac complaints, but is sedentary. P 70, BP 150/80, R
20, Temp 36.4°C, Hct 25. The ECG shows LVH.

A. Preoperative Evaluation
1. Cardiac evaluation: Is she ready for surgery? What disorders are you concerned
about? What history would your try to elicit? (CP, syncope, DOE) Is this a common
presentation for AS?
2. Aortic stenosis: What is the essential pathophysiologic problem? What are your
hemodynamic goals for managing AS? What gradient would you consider significant?
Can the echocardiogram be used to estimate the transvalvular pressure gradient?
3. Anemia: What is the effect of anemia on AS?
4. Raynaud's: What is Raynaud's phenomenon? Significance? How might it be related to
her AS? To her anemia? (SLE, scleroderma, PPH)
B. Intraoperative Management
1. Monitoring: Does this patient need an AL? Where would you place it? Would you
perform an Allen's test?
2. Anesthetic technique: Is she a candidate for a spinal? An epidural? Epidural opiates?
Explain in terms of myocardial supply and demand. Could GA cause problems? Explain
in terms of LaPlace's Law.
3. Hypotension: The BP after induction is 120/70. Rx?
4. Fluids and transfusion: What are your goals for fluids? Are your transfusion thresholds
any different for this patient compared to a healthy ASA I patient?
5. Atrial fibrillation: The HR is 140. Rx? Would your therapy change if atrial fibrillation
were present?
C. Postoperative Care
1. Extremity ischemia: The hand on the side of the radial artery seems dusky. What
would you do?
2. ST changes: ST segment depression is seen in leads II, III, and aVF. What is your
differential? What is the J-point? What would you do? What does ST segment
depression mean? Why is the subendocardium at risk?
3. Mental status change: To what would you attribute her initial disorientation?

V.
76 The Essential Oral Board Review

Cardiovascular VII (Session 1)

An 18 year old, 60 kg, Ht 6'6", male presents for emergent repair of a dissecting aortic aneurysm.

HPI: Pt was an unrestrained driver in a high speed motor vehicle accident. After being
extracted from his demolished automobile, he was found to be awake and complaining of
severe chest pain. Evaluation in the emergency room revealed a widened mediastinum.
Contrast CT scan revealed an aortic dissection beginning at the left subclavian artery and
descending 15 cm down the aorta. The rest of his exam was normal.

PMI: Pt reports occasional episodes of lightheadedness, but is otherwise healthy. He does not
smoke or drink. Coffee makes him nervous.

PE: P 140, BP 160/85, RR 24, T 37°C. There is an impression of the steering wheel on his
chest. His sternum seems to have a concave inwards deformity. His lungs are clear to
auscultation. A mid-systolic ejection click is heard over the precordium. His fingers are
unusually long and slim.

CXR: Widened mediastinum

ECG: Sinus tachycardia with early ventricular depolarization (short PR interval).

Labs: Hct 35. His electrolytes, UA, and liver function studies are normal.
77 The Essential Oral Board Review

Cardiovascular VII (Session 2)

An 18 year old 60 kg, Ht 6'6", male presents for emergent repair of a dissecting aortic aneurysm
following a high speed motor vehicle accident in which he was the driver. The patient is awake
and complaining of severe chest pain. You notice that he has pectus excavatum and long
fingers. A mid-systolic ejection click is heard on exam. Wt 60 kg, Ht 77 inches, P 140, BP
160/85, RR 24, T 37°C, Hct 35, CXR - wide mediastinum.
7 b The Essential Oral Board Review

Cardiovascular VII (Questions)

An 18 year old 60 kg, Ht 6'6", male presents for emergent repair of a dissecting aortic aneurysm
following a high speed motor vehicle accident in which he was the driver. The patient is awake
and complaining of severe chest pain. You notice that he has pectus excavatum and long
fingers. A mid-systolic ejection click is heard on exam. Wt 60 kg, Ht 77 inches, P 140, BP
160/85, RR 24, T 37°C, Hct 35, CXR - wide mediastinum.

A. Preoperative Evaluation
1. Cardiac: What are your priorities in managing this case? Do you think it would be wise
to lower the blood pressure? How - esmolol or nitroprusside? What is the likely location
of the dissection? Why? What are the principles of management for aortic dissection?
2. Marfan's Syndrome: There in an impression of a steering wheel on his chest. Do you
think the accident caused the chest wall deformity? Could he have a syndrome? What
anomalies are associated with Marphan's syndrome? Is an EKG necessary? Why? Is an
echocardiogram?
B. Intraoperative Management
1. Monitors: Where would you place the A-line? Why? Does he need 2? Is there a
difference between radial and femoral AL tracings?
2. Preinduction: If he had mitral valve prolapse, how should you handle his fluids? Would
you give him a bolus preop? Is fluid administration somehow therapeutic for MVP? Which
is more important-administering fluids to minimize the chances of MR or restricting them
to minimize the chances or further dissection?
3. Anesthetic Choices: What drugs would you choose for induction? After intubation, the
BP is 220/120, HR 160. What are you going to do? What are your concerns? The
surgeon requests trimethephan. Is that OK? Ventricular tachycardia is seen on the EKG.
Now what? What are some possible causes? The medical student says the new ACLS
protocol says to give adenosine. How much would you give? Could this be SVT? How
could you tell?
4. Aortic Crossclamp: How are you going to prepare for aortic X-clamp? Describe the
cardiovascular physiology of aortic X-clamping. What happens to BP, HR, CO, SVR?
What organs are at risk for ischemia? How can your protect them? How would you
prepare for coming off? After X-clamp release, the ETC02 rises. Why - increased
production or decreased dead space? If it increased, should bicarb be given? Would you
use pH as a guide for administration? At what pH would your give it?
C. Postoperative Care
1. Hypertension: The patient's BP 180/90. DDx? What would you do? Would verapamil
be a good choice to give?
2. Delayed Emergence: After the operation, the patient does not awaken. DDx? What
would you do? What would your PE include? The left pupil is fixed and dilated.
Significance?

^
80 The Essential Oral Board Review

Cardiovascular VIII (Session 1)

A 66 year old 50 kg, 5'4" female presents for an emergency exploratory laparotomy.

HPI: Pt has a history of Crohn's disease for several years and has been receiving prednisone
and azathioprine. Associated problems include rectocutaneous fistulas and perirectal
abscesses. 2 days prior to admission, she presented with acute abdominal pain.
Abdominal XR revealed free air in her abdomen.

PMI: Mitral stenosis and COPD. She has a 100 pack-year smoking history and chronic
productive cough. She has been seen by her cardiologist, but the records are
unavailable at this point. She is taking digoxin, furosemide, and albuterol PRN.

PE: P 88, BP 90/60, R 30, T 35.8°C. Her airway exam is normal. Bilateral wheezing is heard
and prominent second heart sound is heard on auscultation. Her abdomen is very
distended.

CXR: Cardiomegaly, hyperinflation, bibasilar atalectasis, small right-sided pleural effusion.

ECG: Atrial fibrillation, RVH.

Labs: Hgb 8.2 gm/dl, K+ 2.4, ABG on room air pH 7.45, pC02 48, p02 58.
The Essential Oral Board Review

M\
Cardiovascular VIII (Session 2)

A 66 year old 50 kg woman presents with free air in her abdomen for an emergency exploratory
laparotomy. She has mitral stenosis and COPD. She is taking digoxin, furosemide, and albuterol
PRN. P 88, BP 90/60, R 30, T 35.8°C, K+ 2.4, EKG shows atria! fibrillation.
i»ft\

v. -
0&i

^ S

fe.-

0 \

(9\

fi h
82 The Essential Oral Board Review

Cardiovascular VIII (Questions)

A 66 year old 50 kg woman presents with free air in her abdomen for an emergency exploratory
laparotomy. She has mitral stenosis and COPD. She is taking digoxin, furosemide, and albuterol
PRN. P 88, BP 90/60, R 30, T 35.8°C, K+ 2.4, EKG shows atrial fibrillation.

A. Preoperative Evaluation
1. Fluid Status: Do you think this patient is dehydrated? How would you tell? What are
your goals for preoperative fluid management?
2. Cardiac Status: Is this patient adequately digitalized? Would you obtain a serum digoxin
level? Would you delay the case to treat the potassium? What level would you like to
achieve?
3. Respiratory Status: Why is she tachypneic? Would you treat her tachypnea with a dose
of albuterol? After the albuterol, she complains of chest pain. Your response?
B. Intraoperative Management
1. Monitors: How would you monitor this patient? Is a PAC better than a CVP? What
specials risks exist for PA catheter placement? Which leads would you use to maximize
your detection of ischemia?
2. Anesthetic Choices: Would you prefer, GA or RA? Why? What effect does each have
on the heart?
3. Induction: Which drugs would you use for induction? After intubation, the HR increases
to 180 and ST segment depression occurs. What would you do? Rationale for therapy?
What do you think is going on?
4. Pulmonary Edema: The peak airway pressure is 50 and the pulse oximeter reads 85%.
What would you do? What are your physiologic goals of management? How would you
achieve them? Would amrinone be a good choice?
5. PAC: How could the PAC be used to optimize her care? How would you calculate the
SVR? If the CO is limited by a stenotic mitral valve, could the high SVR reflect artifact
rather than a real state of arteriolar tone? What else effects SVR?
C. Postoperative Care
1. Pain: How would you control her pain? Would Demerol PCA be a good choice? Would
IM morphine?
2. Sepsis: What is the significance of an MV02 saturation of 85%? When you go to the
bedside, she is febrile and hypotensive. Interpretation? DDx?
o4 The Essential Oral Board Review

Cardiovascular IX (Session 1)

A 70 year 72 kg, 6'9", old male is scheduled for an emergency triple vessel coronary artery
bypass graft.

HPI: Pt developed sudden excruciating chest pain 2 hrs ago. In the emergency room, an
echocardiogram revealed LVEF 14% with diffuse LV hypokinesis. In the catheterization V.
lab, the cardiologist inserted right radial intra-arterial and right internal jugular pulmonary
artery catheters. Emergency cardiac catheterization revealed 100% occlusion of his left
anterior descending, 100% occlusion of his right, and 75% occlusion of his circumflex
coronary arteries. He is currently receiving epinephrine 4 mc/min and nitroglycerine 2
mc/kg/min through an 18 gauge IV for cardiogenic shock.

PMI: Known coronary artery disease and insulin-dependent diabetes mellitus. His usual
medications include propranolol, lisinopril, dipyridamole, insulin, and thiazide. He has
had a prior femoral-femoral bypass and right carotid endarterectomy, both without event.
He golfs 2 hrs every day and mild weakness of his right hand.

PE: P 108, BP 80/50, R 32, Temp 36.6°C. His airway exam reveals a large tongue and short
neck. Auscultation of both lungs reveals bilateral crackles. His peripheral pulses are
thready. He is alert and oriented. CVP 20, PA 38/28, PCWP 25, CO 2.5.

CXR: Bilateral pulmonary edema, pulmonary artery catheter in good position, no


pneumothorax.

ECG: Sinus tachycardia, 4-5 mm ST segment depression in the inferior and lateral leads.

Labs: Hgb 12 mg/dl. The platelet count, PT/PTT, and electrolytes are within normal limits.

He is coming to your operating room stat.

i
(
o5 The Essential Oral Board Review

Cardiovascular IX (Session 2)

A 70 year 72 kg, 6'9", old male is scheduled for an emergency triple vessel coronary artery
bypass graft. He has a history of insulin-dependent diabetes mellitus and carotid vascular
disease. He is currently receiving epinephrine 4 mc/min and nitroglycerine 2 mc/kg/min. CXR
shows bilateral pulmonary edema. Echocardiogram reveals LVEF 14% with diffuse LV
hypokinesis. P 108, BP 80/50, R 32, Temp 36.6°C, CVP 20, PA 38/28, PCWP 25, CO 2.5.
86 The Essential Oral Board Review

Cardiovascular IX (Questions)

A 70 year 72 kg, 6'9", old male is scheduled for an emergency triple vessel coronary artery
bypass graft. He has a history of insulin-dependent diabetes mellitus, carotid vascular disease,
and angina associated with an acute evolving Ml. He is currently in cardiogenic shock and is
receiving epinephrine 4 mc/min and nitroglycerine 2 mc/kg/min. CXR shows bilateral pulmonary
edema. Echocardiogram reveals LVEF 14% with diffuse LV hypokinesis. P 108, BP 80/50, R 32,
Temp 36.6°C, CVP 20, PA 38/28, PCWP 25, CO 2.5.

A. Preoperative Assessment
1. IABP: What are the indications, contraindications? Is one indicated preoperatively for this
patient? How does it work? How is proper timing achieved? What pressure do you
follow, SAP, DBP, or MAP? What if the patient also had a VOO pacemaker? Other
details regarding its use?
2. Preop evaluation: How would describe his anesthetic risk? Does his drug regimen make
sense?
B. Intraoperative Management
1. Monitoring: How will the PAC help you with his management?
2. Alpha-stat/pH-stat: How does C02 solubility change with temperature? (Why?) What is
the significance of the neutrality of water? What is meant by temperature corrected blood
gases? The temperature used during CPB is 27°C. Would you use alpha-stat or pH-stat?
What if deep hypothermic arrest were intended?
3. Anesthetic: What anesthetic technique is best for this patient? Should you give any at
all?
4. Anticoagulation: What is the ACT? What does it have to be before going on bypass?
How does heparin work? After the usual dose, the ACT is still 120. Why? Would you
give additional heparin or FFP?
5. Hypotension: Immediately after CBP is initiated, the MAP becomes 50, then climbs to 70.
Why? Would you treat it? DDx?
6. Perfusion Pressure: What perfusion pressure is necessary for CPB, 50, 70,100? Would
pulsatile flow help? Which organs?
C. Postoperative Care
1. Failure to wean off CPB: The BP is 70/40 as the flows are reduced. What would you
do? What if V-waves were seen on the PAC?
2. Postop Neurologic Dysfunction: Postop Day 1 - After a 3 hour bypass time using pH-stat
management, during which the MAP was >50 mm Hg, the patient still does not wake up.
How would you approach the problem? DDx? Could it have been prevented? Was the
BP high enough? Did rapid rewarming have anything to do with it? Could deep
hypothermia have helped? What is the Q10? Other possible contributing factors?
88 The Essential Oral Board Review

Cardiovascular X (Session 1)

A 62 year old, 65 kg, 5'7" male presents for a left total knee replacement.

HPI: Pt has severe degenerative joint disease from years of jogging. Until the past 6 months,
he has been walking 2 miles every day, but has now stopped due to knee pain.

PMI: Hypertension, coronary artery disease with exercise-induced angina, and insulin-
dependent diabetes mellitis. He complains of tingling in both lower extremities. His
medications include aspirin, insulin, diltiazem, nitroglycerin, and nonsteroidal
antiinflammatory agents.

PE: P 72, BP 160/80, R 20, Temp 36.9°C. His airway exam is unremarkable. His lungs are
clear to auscultation. Heart tones are normal. His extremities are well perfused. His
pulses are strong and symmetric.

CXR: Normal.

ECG: Normal sinus rhythm, nonspecific ST and T wave changes in the lateral leads.

Labs: Hct 32, BUN 28, Cr 2.4. Other labs and coagulation studies are normal.

A note from his internist states that the pt is cleared for surgery, but invasive intraoperative
monitoring and regional anesthesia are recommended.
89 The Essential Oral Board Review

Cardiovascular X (Session 2)

A 62 year old, 65 kg, 57" male with a history of degenerative joint disease, hypertension, angina,
and insulin-dependent diabetes mellitis presents for a left total knee replacement. He complains
of numbness and tingling in both lower extremities. His medications include aspirin, insulin,
diltiazem, nitroglycerin, and nonsteroidal antiinflammatory agents. P 108, BP 80/50, R 32, Temp
36.6°C, Hct 32, BUN 28, Cr 2.4.
90 The Essential Oral Board Review

Cardiovascular X (Questions)

A 62 year old, 65 kg, 5'7" male with a history of degenerative joint disease, hypertension, angina,
and insulin-dependent diabetes mellitis presents for a left total knee replacement. He complains
of numbness and tingling in both lower extremities. His medications include aspirin, insulin,
diltiazem, nitroglycerin, and nonsteroidal antiinflammatory agents. P 108, BP 80/50, R 32, Temp
36.6°C, Hct 32, BUN 28, Cr 2.4.

A. Preoperative Assessment
1. Renal: What do you think about the BUN/Cr? Contributing factors?
2. Cardiac Physiology: Draw the normal Starling curve and that for severe CAD. How would
enalipril affect it?
3. Cardiac Assessment: What questions would you ask the cardiology consult? Would you
delay the case to have these questions answered?
B. Intraoperative Management
1. Spinal: Is RA a good idea in this patient? What if the operation will last 5 hours?
2. High spinal: Is spinal analgesia the best for her cardiac condition? Is it therapeutic for
myocardial ischemia? After the spinal is administered, bradycardia is seen on the EKG.
Is this myocardial ischemia or a high spinal?
3. Positioning: What concern do you have about positioning?
4. Methylmethacrylate toxicity: During insertion of the artificial femoral shaft, hypotension,
tachycardia are noted. Your response? Could the byproduct of methylmethacrylate
metabolism cause the hypotension? (pyruvate) What is the pathogenesis of
methylmethacrylate toxicity? (CO increase is c/w a decrease in SVR. Alternatively, the
hypoxia and increase in CVP are c/w fat and bone pulmonary emboli - released from
increased intramedullary pressure.) Other problems with methylmethacrylate? (It is a
thromboplastin that causes pit activation)
5. Pulmonary Hypertension: If the CO decreases, the pulmonary artery pressure increases,
and the PCWP decreases, what is the diagnosis? What is normal PVR? (100-300 dyne-
sec/cm5)
C. Postoperative Care
1. Fat Embolism: Your colleague informs you that the patient is confused on postop day 3
and a note in the chart by medicine attributes it to "residual anesthetic effect." Your
response? He has dyspnea. Petechiae on his chest. Other signs of fat embolism?
(thombocytopenia; prolonged clotting times; fat globules in the retina, urine, and sputum;
RV strain; dead space ventilation during anesthesia)
2. Pressure-volume loop: Draw the normal PV loop. Does it describe work? Contractility?
What happens with pulmonary hypertension?
92 The Essential Oral Board Review

Cardiovascular XI (Session 1)

A 45 year old, 79 kg, 5'11" male is scheduled for a heart transplantation.

HPI: Pt has a history of dilated cardiomyopathy for 2 years. He has had increasing shortness
of breath over the past several months. He is currently short of breath at rest. He was
admitted to the coronary care unit one week ago, where he has been given dobutamine
and amrinone infusions.

PMI: HTN, tobacco use for 20 pack-years, allergy to penicillin.

PE: P 120, BP 60/40, R 34, Temp 38.7°C. His airway exam is unremarkable. Auscultation of
his lungs reveals bilateral crackles. His heart tones are muffled. His extremities are
vasoconstricted and his pulses thready.

CXR: Cardiomegaly with bilateral pulmonary edema.

ECG: Normal sinus rhythm, LBBB, Q waves in the lateral limb leads, frequent multifocal PVCS.

Labs: Hct 25, BUN 28, Cr 2.4. Other labs and coagulation studies are normal.
93 The Essential Oral Board Review

Cardiovascular XI (Session 2)

A 45 year old, 79 kg, 5*11" male is scheduled for a heart transplantation for dilated
cardiomyopathy. He is currently receiving furosemide, dobutamine, and amrinone. He has a
history of HTN. P 120, BP 60/40, R 34, Temp 38.9°C. CXR - cardiomegaly, pulmonary edema.
ECG - LBBB, Q waves in the V4-6, multifocal PVCS. Hct 25, BUN 51, Cr 2.4.
94 The Essential Oral Board Review

Cardiovascular XI (Questions)
A 45 year old, 79 kg, 5'11" male is scheduled for a heart transplantation for dilated
cardiomyopathy. He is currently receiving furosemide, dobutamine, and amrinone. He has a
history of HTN. P 120, BP 60/40, R 34, Temp 38.9°C. CXR - cardiomegaly, pulmonary edema.
ECG - LBBB, Q waves in the V4-6, multifocal PVCS. Hct 25, BUN 51, Cr 2.4.

A. Preoperative Assessment
1. Cardiomyopathy: What is the pathophysiology of dilated cardiomyopathy? Cause?
(viral, rheumatic, congenital, valvular) What do you think his EF is? Is he in shock? What
type?
2. Pharmacologic support: Why current drug regimen? Should he be on epinephrine?
Norepinephrine? (increase in afterload may decrease SV) How to lower HR? (fever)
3. ischemia: Determinants of myocardial ischemia most important in this pt? How is this
pt's cardiac physiology different from that of CAD?
4. HTN: How does his history of HTN affect your management?
5. LV assist devices: What would you do if harvesting delayed for another 8hrs?
B. Intraoperative Management
1. Monitors: How would you monitor the pt? Is a PAC necessary? Site? What are you
going to do with it during the transplantation? What if after inserting the PAC, you found
the PVR to be 640 dyne-sec/cm5? Other pros? (CO, LVP, RVP, PAP, drugs, MV02) /cons
(usually no PHTN, withdrawn, ID, difficult with CM, PAC passage across fresh suture)?
2. Induction: What anesthetic technique are you going to use? Full stomach?
(cyclosporin) RSI? Vecuronium instead of pancuronium given pt's tachycardia?
Ketamine?
3. CPB: What is CPB? What makes your anesthetic management during CPB different
from, say, a vascular case?
4. Coagulation: Why is heparin added? How does it work? How do you reverse its
effects? Any concerns with reversal?
5. Temperature: Why is hypothermia use? Anesthetic implications of hypothermia? What
are alpha-stat/pH-stat?
6. Myocardial protection: How are you going to protect the heart during CPB? Is it even
necessary?
7. Neurologic protection: Are you more or less worried about stroke than with a CABG?
You see air entering the aortic cannula. What would you do? (Discontinue CPB, T-burg,
remove Ao cannula, vent air, retrograde SVC perfusion by connect arterial pump SVC
cannula at 20C x 1-2 min, intermittent carotid compression, consider retrograde IVC
perfusion for splancnic/renal, anterograde hypothermic CPB s 45 min).
C. Postoperative Care
1. Hypotension: Postop BP 80/40. Why? The most common cardiac cause is RV failure).
Therapy of PHTN? (PGE1, NO)
2. Denervation: Summarize the physiology of the donor heart in one word. (Denervation.
Also, preload dependence, delayed HR, contractility catecholamine response) The donor
heart HR is 80. What are you going to do about it? (Pacing or isoproterenol, not atropine
or pancuronium. Isoprel better for CO. PDEIs work)
3. Stress response: How does the donor heart respond to postop fever? (Circulating
catecholamines increases CO via preload augmentation, max response low).
4. Reinervation: The pt has been on a vecuronium infusion. Any concerns with reversal?
Is glycopyrolate necessary with neostigmine? The pt complains of postop chest pain.
Could it be angina? Would you treat tachycardia with digoxin?
5. Rejection: Significance of new dysrhythmia or diastolic dysfunction?
96 The Essential Oral Board Review

Cardiovascular XII (Session 1)

A 25 year old, 75 kg, 5'11" male presents for emergency thoracotomy for an aortic dissection.

HPI: The pt fell from a 3 story house while repairing the roof. There was no loss of
consciousness, but the pt complains of chest pain, especially with inspiration. CT scan
revealed an aortic dissection involving the ascending aorta extending from the left
subclavian artery to the renal arteries.

PMI: Palpitations with occasional syncope and anxiety attacks. He takes propranolol and
alprazolam (Xanax).

PE: P 64, BP 170/90, R 26, Temp 36.0°C. The pt is in a cervical collar. There is a mid
systolic click over the left sternal border. His airway exam is unremarkable. His lungs
are clear to auscultation. His fingers are long and slender. He has pectus excavatum.

CXR: Widened mediastinum and multiple bilateral rib fractures, but no pneumothorax.

ECG: Short PR interval with a delta wave, prolonged QT interval corrected for HR.

Labs: Hct 29. Other labs are pending.


0S
97 The Essential Oral Board Review
0>

Cardiovascular XII (Session 2)


0\
0s A 25 year old, 75 kg, 5'11" male who fell 3 stories presents for thoracotomy for an aortic
dissection from the left subclavian artery to the renal arteries. He has a history of palpitations
^ with occasional syncope and anxiety attacks. He takes propranolol and alprazolam (Xanax). P
0^ 64, BP 170/90, R 26, Temp 36.0°C. The pt is in a cervical collar. There is a mid-systolic click
over the left sternal border. He has pectus excavatum. Hct 29, ECG shows a short PR interval
(^ with a delta wave, prolonged QT interval corrected for HR.
0\

0\
0S

f^\

m\

0\
t
0?\

»'

0&\
98 The Essential Oral Board Review

Cardiovascular XII (Questions)

A 25 year old, 75 kg, 5'11" male who fell 3 stories presents for thoracotomy for an aortic
dissection from the left subclavian artery to the renal arteries. He has a history of palpitations
with occasional syncope and anxiety attacks. He takes propranolol and alprazolam (Xanax). P
64, BP 170/90, R 26, Temp 36.0°C. The pt is in a cervical collar. There is a mid-systolic click
over the left sternal border. He has pectus excavatum. Hct 29, ECG shows a short PR interval
with a delta wave, prolonged QT interval corrected for HR.

A. Preoperative Assessment
1. Cervical spine: Do you need to see neck films before proceeding? What if the neck
were nontender, there were no focal findings, and the pt were sober? Can you clinically
clear a C-spine? What if C1-6 were clear?
2. BP: How would you control the BP? (Normal is 110/60) Nitroprusside? p-blockers?
What about HR? Why not morphine? What is best for dissection? How do your goals
differ from an AAA?
3. Marphan's Syndrome: Did the chest wall defect result from the fall? Is the mid-systolic
click related to the dizziness? What is Marphan's syndrome? (CT disorder of collagen
synthesis - fibrillin, decorin - with decreased tensile strength; Ao dissection, MR, AR, tall,
pectus, scoliosis, lenticular subluxation/dislocation, cataracts).
4. MVP: What are your goals with MVP? (avoid HTN, which could cause rupture).
Associations? (Endocarditis, emboli, SVT, MR with cordae rupture, SNS/PSNS
alterations, Marphan's, Ehlers-Danlos, pseudoxanthoma elasticum.)
B. Intraoperative Management
1. Monitoring: Where would you place the AL? Would you use a PAC? Will it improve
outcome? How?
2. Induction: How? What are your goals? Difficult airway, no fiberoptic, now what?
3. SVT: BP 60/40 after induction. Is this dangerous? To what organs? Doesn't
autoregulation compensate? How treat?
4. Maintenance: What? Why? Which inhalational agent?
5. Aortic cross-clamping: Suddenly, BP 200/100. Why? ST depression. Rx? Physiology
ofAoX?
6. Ischemia: What is it? Which organs are at risk? How protect? Spinal cord? When
would you consider circulatory arrest? ST depression (dissection to coronaries). Pupils
dilated.
7. AoX release: Preparation? PCWP 25. What is your target value?
C. Postoperative Care
1. Postop pain: How manage? Epidural advantages? Place asleep?
2. Oliguria: 20 cc/2 hrs. UA helpful? Urinary Na+? BUN/Cr? Why do they increase with
ARF? What else?
3. Tamponade. BP 70/50. Rx? P150. CVP 32. Related to dissection? PCWP 31.
Related to dissection? PAP 40/32. Related to dissection?
4. Seizures: What is the best drug to treat seizures?
100 The Essential Oral Board Review

Cardiovascular XIII (Session 1)

A 75 year old, 80 kg, 5*4" female is scheduled for bilateral cataract phacoemulsification and
intraocular lens implantation.

HPI: Pt has had progress loss of vision due to cataracts for the past 10 years.

PMI: Chronic bronchitis with a productive cough, rheumatoid arthritis involving the hands and
cervical spine, degenerative joint disease involving the low back, and symptomatic reflux.
Over the past several months, she has had several episodes of syncope. Carotid
Doppler studies have been normal. Holter monitor shows episodes of complete heart
block, requiring placement of a temporary pacemaker. Her medications include aspirin,
prednisone, and thiazide.

PE: P 52, BP 110/55, R 24, Temp 35.9°C. Her airway exam reveals a short chin, large
tongue, visualization of only the hard palate. Auscultation of his lungs reveals diffuse
rhonchi. Her heart tones are distant. She has moderate kyphosis. Peripheral pulses are
adequate. Her voice is hoarse. She has a resting tremor in both hands.

CXR: Diffuse patches of infiltrates and fibrosis, unchanged compared with previous films.

ECG: 2nd degree AV block (type 2), RVH, RBBB, and LVH.

Labs: Hct 51, electrolytes normal except the K+ is 3.4 mEq/L and the Na+ is 131 mEq/L.
1 0 1 The Essentia) Oral Board Review

Cardiovascular XIII (Session 2)

A 75 year old, 80 kg, 5'4" female is scheduled for cataract surgery. She has bronchitis,
rheumatoid arthritis of the cervical spine, syncope, and GE reflux. She is taking aspirin,
prednisone, and thiazide. P 52, BP 110/55, R 24, Temp 35.9°C. Only the hard palate can be
visualized on PE. Her voice is hoarse. Diffuse infiltrates are seen by CXR. ECG shows 2nd
degree AV block (type 2), RVH, RBBB. Hct 51, K+ 3.4 mEq/L, Na+131 mEq/L.
102 The Essential Oral Board Review

Cardiovascular XIII (Questions)

A 75 year old, 80 kg, 5'4" female is scheduled for cataract surgery. She has bronchitis,
rheumatoid arthritis of the cervical spine, syncope, and GE reflux. She is taking aspirin,
prednisone, and thiazide. P 52, BP 110/55, R 24, Temp 35.9°C. Only the hard palate can be
visualized on PE. Her voice is hoarse. Diffuse infiltrates are seen by CXR. ECG shows 2nd
degree AV block (type 2), RVH, RBBB. Hct 51, K+ 3.4 mEq/L, Na+ 131 mEq/L.

A. Preoperative Assessment
1. Cataract: What is it? Associations? (DM, myotonia, galactosemia, aging)
2. Geriatrics: Changes (SBP, LVH, diastolic dysfunction, need for atrial kick, more intraop
hypotension, lower max HR, CO, and antiAch response, CAD, more AL overshoot/NIBP
discrepancy, B-receptor ineffectiveness/decrease, AW reflexes, elastic recoil, sign VC and
RV, VQ MM, dead space, mild FRC, vent response to hypoxia, chest wall rigidity with
narcotics, vertebrobasilar insufficiency, MAC, delirium, loss of liver and kidney, GFR, high
spinals/epidurals).
3. Pacemaker: Is this pt ready for surgery? How would you workup the syncope? Should
she receive a temporary pacemaker? Indications? Are Stokes Adams attacks one of
them? Do you need a cardiology consult? What specific questions would you ask? Why
do you think he has a heart block?
4. Preoperative labs: Are other labs necessary? CXR? ECG? What if the pt had no
symptoms? Would you cancel the case because of the Na and K?
5. COPD: What characterizes COPD? Are there different types? How is this pt different
from a pt with emphysema?
B. Intraoperative Management
1. Eye block: Usual type of eye block? Anesthetic? Concerns? What is the OCR?
Glycopyrolate in the elderly? Is the pt at increased risk for the OCR? Is this true with a
pacemaker? Should you place a magnet before the block? Would you perform the block
without having one ready? Would a peribulbar block be OK?
2. Pacemaker: What information about her pacemaker would you like to know? What type
of pacemaker is likely to have been placed? Would you delay the case to talk to the
cardiologist? Prophylactic magnet placement? How would you prepare the room?
Interaction of anesthesia/surgery with pacemaker?
3. Anesthetic technique: If the pt has severe cor pulmonale, what would be the simplest
technique? Go over your thinking aloud. What sedation would you give for the eye block?
For RA, would you administer sedation?
4. Rheumatoid arthritis: Anesthetic implications? (C1-2 subluxation, cricoarytenoid,
pericardial thickening/effusion, myocarditis, coronary arteritis, conduction, vasculitis, valve
fibrosis, AR, pleural effusions, nodules, fibrosis, anemia, eos, platelet, thrombocytopenia,
steroids, ASA) Problem for GA?
5. Steroids: Would you give stress dose steroids? Why/why not?
6. Cough: During the case, the pt begins coughing. Would you give lidocaine? If GA were
required, would you give propofol and insert an LMA under the drapes? Combitube?
7. Heart block: During MAC, the pt stops breathing. What would you do? What if the
rhythm were HR 15? Does atropine work well in the elderly?
8. Difficult intubation: With GA, a 7.0 ETT becomes stuck at the cords. What would you
do?
C. Postoperative Care
1. Delirium: The pt is disoriented postoperatively. Would physostigmine help? Naloxone?
2. Myocardial ischemia: Pt complains of indigestion in the PACU. Which is better,
odansetron or droperidol? What about her tremor? How would you rule out ischemia?
ST segment depression is seen. BBs or NTB? How does morphine help?
104 The Essential Oral Board Review

Endocrine I (Session 1)

A 33 year old 40 kg, 5'0" woman requires surgery for a ruptured appendix.

HPI- The pt developed sharp abdominal pain in her right lower quadrant yesterday. She had
been vomiting all morning and was seen by her surgeon this afternoon, who discovered
positive peritoneal signs. Abdominal XR and ultrasound were normal.

PMI" Suspicion of hyperthyroidism raised over past week in Mexico. Her endocrinologic
evaluation is in progress, but communication with her physicians in Mexico is impossible
at this time. She also relates a history of frequent fevers with a productive cough and
severe indigestion after meals. She speaks no English.

She has a 20 pack-year smoking history.

PE- P 160, BP 135/66, R 26, Temp 39.0°C. Her airway is normal. She has a large goiterous
mass over her anterior neck. Breath sounds are diminished on the left. The rest of her
exam is unremarkable.

CXR: Large left-sided pulmonary bleb.

ECG: None.

Labs: Hct 44, electrolytes within normal limits.

With the help of a family member who speaks English, consent for anesthesia and surgery is
obtained.
105 The Essential Oral Board Review

Endocrine I (Session 2)

A 33 year old 45 kg, 4'9" woman with hyperthyroidism requires surgery for a ruptured appendix.
She is extremely thin and has pulmonary blebs, a large goiterous mass, hiatal hernia, and is
coughing up sputum. P 160, BP 135/66, R 26, Temp 39.0°C. Hct 44.
106 The Essential Oral Board Review

Endocrine I (Questions)

A 33 year old 45 kg, 4'9" woman with hyperthyroidism requires surgery for a ruptured appendix.
She is extremely thin and has pulmonary blebs, a large goiterous mass, hiatal hernia, and is
coughing up sputum. P 160, BP 135/66, R 26, Temp 39°C. Hct 44.

A. Preoperative Evaluation
1. Thyroid function: Is this patient hyperthyroid? How can you tell? Do you need TFTs?
(if T3RU and T3 vary concordantly = dz) Is she having a thyroid storm? How can you
tell? Is making her euthyroid a reasonable preoperative goal? What premedications
would you give? How do iodine, steroids, PTU, and B-blockers work? (I - organify,
release; PTU - synthesis)
2. Pulmonary status: What is the significance of her pulmonary blebs? Of her sputum?
How do they it affect your management? Should you delay the case for antibiotic
therapy?
3. Tachycardia: What is your DDx for her tachycardia? Should she have her HR slowed
before induction? Which calcium channel blocker is best for treating tachycardia-
diltiazem, verapamil, or nifedipine?
4. Premedication: How would you premedicate this patient? Are H2 blockers or oral
antacids necessary?
B. Intraoperative Course
1. Monitors: Is a lead V5 better than lead 2? Is a CVP needed? Where would you place it?
2. Induction: Is there any advantage to thiopental? The cords are visible, but you are
unable to insert a 7.5 ETT. What would you do? What is a retrograde wire technique?
Would be appropriate? Could you administer anesthesia with an LMA?
3. Maintenance: Is MAC increased in this patient? Is it increased with hyperthyroidism?
With fever? What is MAC? Is induction time increased in this patient?
4. Tachycardia: The HR has never decreased from its preoperative value. Would you
administer propranolol? DDx? Which inhalational agent would you choose?
5. Fluids: What type of fluids would you select? How would you estimate her needs?
Hypotension and tachycardia persist despite administration of 2 L of LR. Does she need
colloid?
6. Pneumothorax: 15 minutes after propranolol, the peak airway pressures rise. The
tachycardia and hypotension persist. The surgeon asks if you have listened to breath
sounds recently?
C. Postoperative Care
1. Fever: The temperature is 39.5°C. How would you treat it? Does it matter whether you
give aspirin or acetominophen? How can you distinguish thyroid storm from MH? Is
thyroid storm likely in the postoperative period?
2. Stridor: Stridor develops following extubation. How does the timing of stridor in
relationship to the operation affect your DDx? Rx? Pulmonary edema develops. Why?
3. Airway: 2 days later, the patient is scheduled for repeat exploratory laparotomy because
of the finding of pus from the wound and persistent fever. How would you intubate her this
time?
108 The Essential Oral Board Review

Endocrine II (Session 1)

A 16 year old 70 kg, 5'8" male presents for resection of a pheochromocytoma.

HPI: Pt has had a history of paroxysmal palpitations, headaches, and diaphoresis all his life. A
recent evaluation revealed significantly elevated urinary catecholamines. A CT scan
showed a right-sided supra-adrenal mass, though to be the pheochromocytoma. Since
the diagnosis has been made, the pt has been started on labetolol and diltiazem, and is
reportedly feeling better.

PMI: Over the past year, he has become increasingly tired, and recently gave up his job
delivering pizzas. He admits to smoking marijuana on occasion but denies other drug
use.

His brother died during an appendectomy at the age of 22, the details of which are
obscure.

PE: P 88, BP 200/100, R 20, T 38.0°C. Oral exam shows several teeth either loose or
missing. Lungs clear to auscultation. Cardiac exam revealed an occasional extrasystole.

CXR: Cardiomegaly.

ECG: Left axis deviation, large S waves in V2-3 and R waves in V5-6, T wave inversion in V4-6,
and multifocal PVCs.

Labs: Hct 35, electrolytes within normal limits. His urine is positive for cocaine and
amphetamines.

The lives family 100 miles away and the pt has been reluctant to undergo surgery because of
repeated delays. Due to these concerns, the cardiologist, internist, surgeon, and family are all in
agreement that the surgery should proceed as scheduled today.
jL ' v9 The Essentia! Oral Board Review

r Endocrine II (Session 2)

, A 16 year old 70 kg, 5'8" male presents for resection of a pheochromocytoma. He has a history
!* of hypertension, palpitations, and easy fatiguability. His urine is positive for cocaine abuse He is
^■ taking labetolol EKG
cardiomegaly. and shows
diltiazem.
leftP ventricular
88, BP 200/100, R 20,T Twave
hypertrophy, 38.0°C, Hct 35,
inversion CXR shows
in V4-6, and multifocal
p> PVCs.

^N

jlp*

JpN

Ms
10 The Essential Oral Board Review

Endocrine II (Questions)

A 16 year old 70 kg, 5'8" male presents for resection of a pheochromocytoma. He has a history
of hypertension, palpitations, and easy fatiguability. His urine is positive for cocaine abuse. He is
taking labetolol and diltiazem. P 88, BP 200/100, R 20, T 38.0°C, Hct 35, CXR shows
cardiomegaly. EKG shows left ventricular hypertrophy, T wave inversion in V4-6, and multifocal
PVCs.

A. Preoperative Evaluation
1. Pheochromocytoma: What is it? What is the symptom triad? (diaphoresis, HA,
tachycardia, not flushing) What hormone is usually released? What difference does it
make? How is it diagnosed? Is a clonidine suppression test valid? Is this patient
adequately p-blocked? Which should you give first, a or p-blockers? When and why are
p-blockers used? Is labetolol a good first-line drug? Which a-blocker is best? (prazocin -
oc1 only so neg feedback present, phenoxyb - a-1 and 2) How would you treat his BP
before surgery? What are your goals of management?
2. Cardiomyopathy: What is the significance of his fatiguability? Of his cardiomegaly? Of
his EKG findings? Would Mg2+ be appropriate to give? Would you order a cardiology
consult? What specific questions would you want addressed? Is the cardiomyopathy
reversible?
3. Anemia: Why do you think he is anemic? Could it be somehow related to his rhinorrhea?
To his volume status? Because of his heart condition, do you think he should be
transfused?
4. Cocaine: What is the significance of its use? Is it important to distinguish between
cocaine- and catecholamine induced-cardiomyopathy?
5. Premedication: Would you premedicate this patient? Why? With what? Would you
administer droperidol for nausea? Would you discontinue her phenoxybenzamine?
B. Intraoperative Management
1. Monitors: How would you monitor this patient? Why an intra-arterial catheter? Why or
why not a CVP? A PAC? Why a PAC vs. TEE?
2. Anesthetic choices: Would a regional technique be appropriate? Does it matter which
drugs would you use for induction? What if he had a difficult airway? What if he also had
reflux esophagitis? Problems with succinylcholine?
3. Hypertension: The BP is 220/110 after intubation. Is this critically dangerous? Why?
How would you treat it, NTG or SNP? Would you add halothane to increase the depth of
anesthesia? Which relaxant is better, mivacurium or pancuronium?
4. Dysrhythmias: 30 minutes after the case, the HR suddenly increases to 140. DDx? Rx?
Could this be due to tumor manipulation?
5. Delayed emergence: 5 minutes after discontinuing the isoflurane and N20, the patient is
spontaneously breathing, but still not awake. What would you do?
C. Postoperative Care
1. Hypotension: What postoperative complications do you expect? The nurse calls you
with the patient's BP of 100/60. Is this critically low? What would you do? Is
postoperative hyper- or hypotension more likely?
2. Hypoglycemia: Why might postoperative seizures occur? Should you give glucose
postoperatively?
12 The Essential Oral Board Review

Endocrine III (Session 1)

A 33 year old 66 kg, 5'5" female presents for a dilatation and curettage for a missed abortion.

HPI: Pt has had crampy lower quadrant pain and menometorrhagia. She is showing active
vaginal bleeding.

PMI: Juvenile onset diabetes mellitis. She is taking birth control pills, NPH/regular insulin
40/20 q AM and 30/15 q PM. She has good exercise tolerance. Her urine output has
been good.

The last she ate or drank anything was over 24 hrs ago.

PE: P 96, BP 130/70, R 36, T 38.8°C. Only the base of the tongue can be seen with full
mouth opening due to limited jaw mobility. The lungs are clear to ausculation. Heart
tones are normal. Her extremities are cool with 3-4 second capillary refill. Her mucous
membranes are dry and pink. She is complaining of an intense headache.

CXR: None.

ECG: Q-waves in leads 2, 3, and aVF.

Labs: Hct 48, K+ 5.7 mEq/L, WBC 16.2, glucose 302 mg/dl. ABG reveals pH 7.22, pC02 46,
p02 80.

In the emergency room, an 18 gauge IV has been started. 10 units of regular insulin have been
given IV. She is now receiving D5 lactated Ringer's solution at a rate of 250 cc/hr. No type and
cross has been ordered by her gynecologist.
13 The Essential Oral Board Review

Endocrine III (Session 2)

A 33 year old 66 kg. 5'5" female presents for a dilatation and curettage for a missed abortion.
She has juvenile onset diabetes mellitis. She is taking NPH/regular insulin 40/20 q AM and 30/15
q PM. P 96, BP 130/70, R 36, T 38.8°C. Hct 48, K+ 5.5 mEq/L, WBC 16.2, glucose 302 mg/dl.
ABG reveals pH 7.22, pC02 46, p02 80.
11 4 T h e E s s e n t i a l O r a l B o a r d R e v i e w

Endocrine III (Questions)

A 33 year old 66 kg. 5'5" female presents for a dilatation and curettage for a missed abortion.
She has juvenile onset diabetes mellitis. She is taking NPH/regular insulin 40/20 q AM and 30/15
q PM. P 96, BP 130/70, R 36, T 38.8°C. Hct 48, K+ 5.5 mEq/L, WBC 16.2, glucose 302 mg/dl.
ABG reveals pH 7.22, pC02 46, p02 80.

A. Preoperative Evaluation
1. Porphyria: How do you know this is an acute abdomen and not porphyria? (acute
intermittent: defect in glycine to heme, ALAS stimulated, uroporphyrinogin synthetase
inhibited, increase in porphobilinogen; Sx: weak, BP lability; Rx: IVF, hematin to make
cytochromes and suppress enzymes, glucose to suppress enzymes) How do you know
this is not DKA?
2. Diabetes: What is the difference between type I and type 2 diabetes? What does insulin
do? (anabolic: glycogen, protein, TG, chol; inhibit: glycogenosis, lipolysis, ketogenesis,
gluconeogenesis) How would you handle her insulin regimen? What is your rationale for
doing so? Why couldn't you merely divide her glucose level by 150 and give that amount
as units of insulin/hr? What is DKA? What is the perioperative significance of her
hyperglycemia?
3. Laboratory: Do you require a UA? Why? What is the significance of proteinuria?
4. Premedication: What would you administer for anxiety? What are the effects of
benzodiazepines on porphyria?
B. Intraoperative Management
1. Monitors: How would you monitor the patient? Could you do with just a large bore IV?
2. Opiates: Would you give narcotics before induction? Which would you choose? Is the
respiratory depression (mu-2) mediated by the same receptor effect as supraspinal
analgesia (mu-1)? How about dysphoria (sigma) and epileptogenicity (delta)? She
complains of itching. Can you Rx before induction?
3. Anesthetic choice: Would regional be a good idea? Is a rapid sequence induction
necessary? Is she at risk for having a difficult airway? Would you induce with ketamine?
Aren't you worried about porphyria?
4. Maintenance: What is the best drug for maintenance with DKA? Does the acidity affect
anesthetic action? (acids like thiopental, opiates)
5. Hypotension: BP 90/40. What would you do? DDx?
6. Electrolytes: At what point would you add K+ in her IVFs? Why? P04? Why?
C. Postoperative Care
1. Nonketonic hyperosmolar coma (NKHC): The patient seems lethargic and the Na is
155. Is this NKHC? Why or why not? What is NKHC?
2. Autonomic neuropathy: The glucose is 45. Why isn't the patient tachycardic?
11 6 The Essential Oral Board Review

Endocrine IV (Session 1)

A 300 kg, 57" female is scheduled for an lumbar laminectomy and stabilization.

HPI: Pt has had a history of severe back pain for years, with recent pain and weakness of her
right leg. X-ray and CT show herniation of the L4-5 disc with impingement of the right L4
nerve root and compression fractures of L3-5.

PMI: Hypertension, a symptomatic hiatal hernia, and mild asthma. The last episode of
bronchospasm was 6 months ago. She was exposed to TB and had a subsequent
positive PPD. A motor vehicle accident from 2 years ago (she fell asleep while driving)
left her with chronic upper back and neck pain. She is taking isoniazid, rifampin, and
prednisone.

She had a gastric stapling procedure 5 years ago.

The pt sleeps sitting in a chair.

PE: P 80, BP 180/80, R 26, Temp 37.1°C. The pt is extremely anxious and tearful. Airway
exam normal. Breath sounds and heart tones inaudible. Abdomen markedly obese.
Mild clubbing noted.

CXR: Obesity.

ECG: Biventricular hypertrophy.

Labs: Hct 48, electrolytes normal, slightly elevated GGT. Pulmonary function studies reveal a
reduction in both the FEV1 and FVC, but a FEV1/FVC ration of 66%. ABG pH 7.35,
pC02 49, p02 56 on room air.
117 The Essential Oral Board Review

Endocrine IV (Session 2)

A 300 kg, 57" female with a herniated L4-5 disk is scheduled for a lumbar laminectomy. She has
hypertension, a hiatal hernia, mild asthma, and a history of tuberculosis. She is taking isoniazid,
rifampin, and prednisone. P 80, BP 180/80, R 26, Temp 37.1°C, Hct 46.
1 18 The Essential Oral Board Review

Endocrine IV (Questions)

A 300 kg, 57" female with a herniated L4-5 disk is scheduled for a lumbar laminectomy. She has
hypertension, a hiatal hernia, mild asthma, and a history of tuberculosis. She is taking isoniazid,
rifampin, and prednisone. P 80, BP 180/80, R 26, Temp 37.1°C, Hct 46.

A. Preoperative Evaluation
1. Morbid obesity: Definition (BMI >27.5, morbid >40). How does MO increase her
anesthetic risk? Can you reduce that risk? ASA classification?
2. Preoperative evaluation: Do you require PFTs? An ABG? CXR? EKG?
3. Premedication: Premedications? (insulin, Gl prophylaxis, hydrocortisone) She is
extremely anxious. After 50 mc of fentanyl, she is apneic. Would you administer
naloxone? Effect on subsequent narcotics? Would you have to cancel the case?
B. Intraoperative Management
1. Monitors: How does obesity affect your selection and placement of monitors? Is a
noninvasive blood pressure reliable? Would you place an arterial catheter?
2. Anesthetic technique: GAorRA? Greater local anesthetic requirements for epidural?
Significance of isoniazid? (increases metabolism in rapid acetylators, but not phenytoin,
phenobarbitol, or alcohol). Using the concept of volume of distribution, explain why you
think she will need more or less muscle relaxants?
3. Induction: Is a difficult airway expected? (limited TMJ, atlanto-occipital joint) Risk for
aspiration? Awake intubation, then have her move herself to prone position?
4. Positioning: How would you position the patient? On 2 beds? How would you turn her
to the prone position? What is the effect of the prone position on ventilation? The patient
is placed prone on 2 hydraulic beds. During the case, you notice a puddle of pink oily
liquid rapidly forming on the floor. One of the bed rapidly begins to sink, and the patient
rolls onto the floor. She is in the prone position. What would you do? Would you cancel
the case?
5. Maintenance: What drugs would you use for maintenance and why? How does obesity
affect your selection of inhalational agents? Of their duration? What is the significance of
her medications?
6. Hypotension: The blood pressure is 70/50. What would you do? The Hct is 22. Is your
transfusion trigger any different for the obese patient? She has 2 units of predonated
autologous blood. Would you give them?
7. Positional changes: Effects of the supine position and GA? Illustrate these changes
using the compliance curve. Illustrate the changes in lung volume. What happens to
closing capacity with obesity? The PIP is 30. What would you do?
8. Extubation: How would you extubate the patient? What are the advantages to sitting this
patient up prior to extubation? Of a deep extubation?
C. Postoperative Care
1. Pain control: As you are walking down the hall of your hospital 3 days later, you hear a
pulse oximeter alarm coming from a room with an open door. You enter the room and find
your patient with pulse oximeter reading of 85%. Does obesity predispose to hypoxia by
altering the closing capacity? Illustrate how. What would you do? The nurses just gave 6
mg of IM morphine. Would you give naloxone? The patient wakes up in pain, and the
saturation decreases to 75%. Is she predisposed to flash pulmonary edema?
Recommendations for postop pain control? Would you place an epidural if her insurance
company refused to pay for it?
2. Pickwickian syndrome: The EKG shows RVH. Is this diagnostic for the Pickwickian
syndrome? (hypercapnia, cyanosis, polycythemia, cor pulmonale, somnulence) What is
the pathophysiology of the RVH?
3. Halothane hepatitis: The patient seems jaundiced. Is she predisposed to halothane
hepatitis? (increased metabolism)
120 The Essential Oral Board Review

Endocrine V (Session 1)

A 68 year old, 72 kg, 5'2" male is scheduled for a single vessel coronary artery bypass graft.

HPI: Pt had a posterior wall Ml 2 weeks ago. Catheterization at that time revealed 100%
occlusion of the right coronary artery, 60% occlusion of his left main coronary artery, and
50% occlusion of the circumflex coronary artery. The LV ejection fraction was 65%.
Echocardiogram revealed RV dissension with inferoposterior hypokinesis. The pt was
managed with heparin and a dopamine infusion. He is now receiving propranolol,
diltiazem, and spironolactone for his cardiac condition.

PMI: During his hospitalization for the Ml, the pt was found to have hypothyroidism due to
autoimmune thryoiditis. Thyroxine was begun 1 week ago.

PE: P 54, BP 90/40, R 10, T 35.1°C. The pt is lethargic and sleepy. The oral airway exam is
difficult due to lack of pt cooperation. Jugular venous dissension is present, and
increases with palpation of liver and inspiration. The lungs are clear to auscultation.
Heart tones are diminished. Hepatomegaly is noted on abdominal exam.

CXR: Cardiomegaly with increased pulmonary vascularity.

ECG: NSR, RBBB, low voltage QRS complexes, ST segment elevation in leads V1-3.

Labs: Na* 128 mEq/L, Hct 24, other electrolytes normal, PT 26, PTT 49.
121 The Essential Oral Board Review

Endocrine V (Session 2)

A 68 year old, 72 kg, 5'2" male with hypothyroidism is scheduled for an elective coronary artery
bypass surgery. He is very lethargic. His medications include propranolol, diltiazem,
spironolactone, and thyroxine. Jugular venous distention is noted on exam. P 54, BP 90/40, R
10, T 35.1°C, Na+128 mEq/L, Hct 24, EKG reads low voltage QRS with a RBBB.
r
122 The Essential Oral Board Review

Endocrine V (Questions)

A 68 year old, 72 kg, 5'2" male with hypothyroidism is scheduled for an elective coronary artery ^
bypass surgery. He is very lethargic. His medications include propranolol, diltiazem,
spironolactone, and thyroxine. Jugular venous distention is noted on exam. P 54, BP 90/40, R C*"
10, T 35.1 °C, Na+ .128 mEq/L, Hct 24, EKG reads low voltage QRS with a RBBB. ^

A. Preoperative Evaluation ("**


1. Myxedema coma: What likely condition does this patient have? What complications are ^
associated with myxedema coma? (C02 retention, Gl bleeding, hyponatremia, refractory *
hypotension). Why are hypothyroid pts hyponatremic? How would you replace the thyroid (^
hormone? (T4 400-500 mc then 50-200 q day, T3 25-50 mc is faster, more dangerous) ^
Which is more dangerous to give, T3 or T4? '
2. Cardiac: Is it possible to have hypothyroidism and CAD? Should this patients severe /«*
hypothyroidism be treated prior to surgery? How would you decide? (80% mortality in _
untreated myxedema coma). Would treatment improve outcome? Does he need a v
preoperative echocardiogram? Should you discontinue his propranolol? /%
3. Emergency surgery: The patient develops a significant upper Gl bleed and requires
e m e r g e n c y s u r g e r y. W h a t w o u l d b e y o u r g o a l s f o r m a n a g e m e n t ? D o e s h e n e e d ( /
replacement steroid therapy? Why? Side effects of therapy? /%
B. Intraoperative Management
1. Monitors: How would you monitor this patient? The BP is 70/30. Which is more O
important at this point, the AL or CVP? ^
2. Induction: How would you induce anesthesia? Which is better, ketamine or alfentanil?
Are you concerned about adrenal function with etomidate? (^
3. Maintenance: What drugs would you use for maintenance? Are muscle relaxants /*%!
necessary?
4. Hypotension: After initiation of CPB, the MAP remains at 30 mm Hg. What would you r**.
do? Would you administer a Ca2+ infusion? Why/why not? *®&\
Hypothermia: The temperature is 33°C. How low would you the temperature drop?
What mechanism of heat loss usually occurs during anesthesia? What deleterious ^
consequences of hypothermia could occur?
Postoperative Care ^
1. Hyperthyroidism: The patient develops pulmonary edema on postop day 3. DDx? Could ^
this be thyroid storm? Would you hydrate, give PTU, Kl, steroids, or provide a cooling
blanket? What are the contraindications to BB? How long would you expect this effect of ^
T4 to last? _^L
<"%

^%

"*%

"SJk
124 The Essential Oral Board Review

Endocrine VI (Session 1)

A 25 year old G1P0 female with a 29 week intrauterine pregnancy is scheduled for a
parathyroidectomy. She has hyperparathyroidism, thyroid cell carcinoma, and
pheochromocytoma. She complains of nausea, vomiting, heat intolerance, and shortness of
breath. Her EKG shows a normal sinus rhythm with left axis deviation.

HPI: Pt initially presented with kidney stones. Subsequent evaluation revealed hypercalcemia
associated with an elevated parathyroid hormone level due to hyperparathyroidism. An
asymptomatic pheochromocytoma and thyroid cell carcinoma were also discovered.

PMI: Pt has been depressed since becoming pregnant. She has frequent bouts of nausea and
vomiting. Her medications include mithramycin and ranitadine.

PE: P 100, BP 160/90, R 16, T 36.6°C. Her airway is normal. Her lungs are clear to
auscultation. Her cardiac exam is normal. She is having difficulty standing due to leg
weakness.

CXR: Normal.

ECG: NSR, LVH, short QT interval.

Labs: Ca2+16.5 mg/dl, P04 0.8 mEq/L, Glu 190, Hct 30.
125 The Essential Oral Board Review

Endocrine VI (Session 2)

A 25 year old, 58 kg, 5'4" G1P0 female with a 29 week intrauterine pregnancy is scheduled for a
parathyroidectomy. She has hyperparathyroidism, thyroid cell carcinoma, and a
pheochromocytoma. She complains of nausea, vomiting, heat intolerance, and shortness of
breath. P 100, BP 160/90, R 16, T 36.6°C. Ca2+16.5 mg/dl, Glu 190, Hct 30. EKG shows a
normal sinus rhythm with left axis deviation.
126 The Essential Oral Board Review

Endocrine VI (Questions)

A 25 year old, 58 kg, 5'4" G1P0 female with a 29 week intrauterine pregnancy is scheduled for a
parathyroidectomy. She has hyperparathyroidism, thyroid cell carcinoma, and a
pheochromocytoma. She complains of nausea, vomiting, heat intolerance, and shortness of
breath. P 100, BP 160/90, R 16, T 36.6°C. Ca2+ 16.5 mg/dl, Glu 190, Hct 30. EKG shows a
normal sinus rhythm with left axis deviation.

A. Preoperative Evaluation
1. Multiple endocrine neoplasia (MEN): What is it? Are there different types?
2. Hyperparathyroidism: What are the symptoms of hyperparathyroidism? (polyuria, short
QT, HTN, PVCs, NA/, weak, MS) What is their cause? Would you expect the calcitonin
level to be high or jow? What if it were high (c/w parathyroid tumor, but PTH effect
supersedes)
3. Normal pregnancy: What is the significance of her heat intolerance, N/V, and SOB?
4. Pheochromocytoma: What symptoms are C/W a pheo? (HA, palpitations, persperation)
If the pheo were symptomatic, which vasodilator would you give preop? (phentolamine
and phenoxybenzamine - <x1 and 2; prazocin - a2 only so less tachycardia; only phenoxy
is covalent) What are your preoperative goals for a pheo?
B. Intraoperative Management
1. Hypercalcemia: How would you treat the electrolyte abnormality? (NS, lasix, then
glucocorticoids, calcitonin, mithramycin)
2. Monitors: How would you monitor the patient? The CVP is -7 mm Hg. Should you re-
zero the transducer and check its height?
3. Induction: After the hypercalcemia is treated, does it really matter how would you induce
anesthesia? Why, if her pheochromocytoma is not directly innervated?
4. Maintenance: Are there any drugs you would avoid? (N2Ot droperidol, atracurium,
curare, MS, halothane, pancuronium, ketamine)
5. Ventilation: What are your goals of ventilator management? What is the problem with
hyperventilation? (UBF, 02 sat curve, ionized Ca2+)
6. Hypertension: The BP is 190/110. Based on their pharmacokinetic properties, which
would you rather give, phentolamine or nitroprusside? Why?
C. Postoperative Care
1. Hypocalcemia: Your are called for respiratory difficulties. What are you thinking? The
patient has labored respirations. How can you confirm hypocalcemia as a cause (stridor,
prolonged QT, contractures, seizures, Chvostek's and Trousseau's). She is hypotensive
and a 500 cc bag of 5% albumin is hanging.
2. Infant of a diabetic mother (IDM): 10 weeks later a 12 pound baby boy is born. The
neonate has a seizure shortly after birth. You are called to assist. Your
recommendations? If glucose and insulin cross the placenta, why are Dims at risk for
seizures?
3. MEN: Would you recommend anything for the family? (pheo, thyroid Ca2+, hyperPTH)
128 The Essential Oral Board Review

Endocrine VII (Session 1)

A 41 year old, 85 kg, 50" female presents for resection of a tumor in the ileum obstructing bowel.

HPI: The pt had 48 hour history of abdominal cramping, distention, pain, and vomiting.
She has not had anything to eat or drink for the past 16 hours.

PMI: Pt has a history of type II diabetes mellitus, asthma, paroxysmal facial flushing, and
chronic diarrhea. Her medications include NPH insulin, ventolin aerosols PRN, and
diphenhydramine.

PE: P 86, BP 100/56, R 24, Temp 36.2°C. Her heart rate is irregular. The lungs are clear to
auscultation. A blowing systolic murmur is heard along the left sternal border. The
abdomen is distended. She has hepatomegaly. Jugular venous distention is noted. An
NG tube in inserted.

CXR: Normal with proper placement of the NG tube.

ECG: Atrial fibrillation, RBBB.

Labs: Na+150 mEq/L, K+ 4.8 mEq/L, CI" 101 mEq/L, C02 20 mEq/L, glucose 160 mg/dl,
Hgb 15 g/dl, PT/PTT and LFTs (all obtained yesterday).

The pt took half her usual NHP insulin dose, none of her regular insulin dose, and has an IV with
D5LR running at 125 cc/hr.
29 The Essential Oral Board Review

^ Endocrine VII (Session 2)

~ A 41 year old, 85 kg, 50" female presents for resection of a tumor in the ileum obstructing bowel.
^ She has a history of type II diabetes mellitus, asthma, paroxysmal facial flushing, and chronic
^n diarrhea. Her medications include insulin, ventolin aerosols PRN, and diphenhydramine. P 86,
^ BP 100/56, R 24, T 36.2°C. Her heart rate is irregular. A blowing systolic murmur is heard along
C' tne left sternal border. She has hepatomegaly. Hct 36, Na+150.

{9\

^^

/$$\

fi$\

^\

Ms
130 The Essential Oral Board Review

Endocrine VII (Questions)

A 41 year old, 85 kg, 50" female presents for resection of a tumor in the ileum obstructing bowel.
She has a history of type II diabetes mellitus, asthma, paroxysmal facial flushing, and chronic
diarrhea. Her medications include insulin, ventolin aerosols PRN, and diphenhydramine. P 86,
BP 100/56, R 24, T 36.2°C. Her heart rate is irregular. A blowing systolic murmur is heard along
the left sternal border. She has hepatomegaly. Hct 36.

A. Preoperative Assessment
1. Diabetes mellitus: Should you use dextrose-containing solutions? What does the lack or
respiratory variation in HR tell you?
2. Asthma: How would you assess the severity of her asthma?
3. Obesity: What are the physiologic consequence of her obesity?
4. Carcinoid syndrome: Why does the pt have flushing? (serotonin) What is carcinoid
syndrome? (enterochromaffin tumor) What is the malignant carcinoid syndrome? (flush,
diarrhea, RAD, TR, maybe PS, usually with liver mets) What are the mediators?
(serotonin, bradykinin, PG, histamine, substance P) Diagnosis? (urine 5-
hydroxyindolacetic acid) What are your anesthetic goals for managing a pt with carcinoid
syndrome? (assess, prevent mediator release)
5. Tricuspid regurgitation/A Fib: Why the murmur? (TR from serotonin) What do you
think about the rhythm. Would you recommend cardioversion?
6. Liver mets: Why the hepatomegaly? Why found in pts with carcinoid syndrome?
Related to finding of JVD?
7. Preop: Goals? Additional labs? Premeds? (octreotide, diphenhydramine, ranitidine,
aprotinin - bradykinin agonist, parachlorophenyalanine - 5HT synthesis, cyproheptadine,
ketanserin, methylsergide - 5HT blockers, bronchodilators, steroids for asthma.)
B. Intraoperative Management
1. Monitoring: How would you monitor? Will a CVP tell you everything you need to know?
How can it, with TR? What is
2. Induction: Best technique? (avoid SNS stim, tumor manipulation, histamine releasers,
hypotension, hypercarbia, hypothermia). Are you good enough to do regional? Are you
concerned about her airway? Her asthma? Her obesity?
3. Maintenance: Best technique? Which drugs would you avoid?
4. Hypotension: BP 60/30. What would you do? (avoid epi, norepi, dopa, phenylephrine,
ephedrine; use methoxamine)
5. Carcinoid crisis: BP 340/280. What would you do? How much desflurane? After giving
a bolus SNP and turning up isoflurane to 5, you find transducer on the floor.
6. Bronchospasm: The peak airway pressures are elevated. How do you whether it's a
problem with lung compliance or airway resistance? How can determination of the
dynamic (airway resistance and thoracic compliance) or static (thoracic compliance only)
compliances help? What is difference?
C. Postoperative Care
1. DKA: The BS is 400 mg/dl and the pt is confused. Is this nonketotic hyperosmolar coma?
Why not? How is the treatment for the 2 different? What is DKA? Caveats in treatment?
After hours of treatment, the urinary ketones are still positive. Should you give more
insulin? (p-OH butyrate made into aceto-acetoacetate)
2. Hypoglycemia: The next day, you find the pt had to be intubated. No one knows why.
Your suggestions.
3. Obesity: How would postop spirometry be expected to differ compared to preop
spirometry? Why? Would supplemental 02 be expected to help with atalectasis? Why?
Aren't you worried about absorption atalectasis? Why don't you see absorption
atalectasis with N20?
132 The Essential Oral Board Review

Renal I (Session 1)

A 44 year old, 69 kg, 57" male is scheduled for revision of his left arm arteriovenous fistula.

HPI: Pt has had chronic renal failure due to hypertension for 7 years, and has had several AV
fistulae revisions. His current catheter failed one week ago. Since then, he has been
dialyzed through a subclavian catheter. His last dialysis was performed one hr ago. He
has had several episodes of CHF related to fluid overload and previous catheter failure.
His medications include captopril, lasix, calcium gluconate, aluminum hydroxide.

PMI: Non-insulin dependent diabetes mellitus, 40 pack-years of smoking.

PE: P 68, BP 104/66, R 24, T 37.1°C. His airway exam is unremarkable. Cardiac,
pulmonary, and other system exams are unremarkable.

CXR: None.

ECG: (Obtained yesterday) NSR, LVH, peaked T-waves.

Labs: (Obtained immediately after dialysis) Hct 22, K+ 2.8 mEq/L. Other electrolytes are
normal.

He is presents for surgery one hour after dialysis.


1 33 The Essential Oral Board Review

Renal I (Session 2)

A 44 year old, 69 kg, 57" male with chronic renal failure is scheduled for revision of his
arteriovenous fistula. He has hypertension, non-insulin dependent diabetes mellitus, and 40
pack-years of smoking. Dialysis though a subclavian catheter was just performed one hour ago.
His medications include captopril, lasix, calcium gluconate, aluminum hydroxide. P 68, BP
104/66, R 24, Temp 37.1°C, Hct 22, K+ 2.8 mEq/L.
1 34 The Essential Oral Board Review

Renal I (Questions)

A 44 year old, 69 kg, 57" male with chronic renal failure is scheduled for revision of his
arteriovenous fistula. He has hypertension, non-insulin dependent diabetes mellitus, and 40
pack-years of smoking. Dialysis though a subclavian catheter was just performed one hour ago.
His medications include captopril, lasix, calcium gluconate, aluminum hydroxide. P 68, BP
104/66, R 24, Temp 37.1°C, Hct 22, K+ 2.8 mEq/L.

A. Preoperative Evaluation
1. Chronic renal failure: Does CRF have any systemic effects? Which is this patient likely
to have? Do you need to know his serum Ca? What if it is low? Why is his K low?
2. Hematologic: Why is his Hct low? Should he receive erythropoetin? Should he be
transfused? Should you obtain a PT/PTT to rule out coagulopathy? How can DDAVP
help?
3. Blood pressure: Why is CRF associated with HTN? Are you satisfied with this patient's
BP? Why is it low? (volume, pericardial effusion) Do you think it is significant? If it was
significant, wouldn't there be tachycardia? Is hypotension more or less deleterious for the
patient with CRF? What compensation normally exists for anemia?
4. Dysequilibrium syndrome: The patient seems confused. DDx? Do you need to see his
post-dialysis labs? Can dialysis contribute to this finding?
B. Intraoperative Management
1. Anesthetic technique: What are you goals for management? Would regional
anesthesia be an appropriate choice? What are the problems with an axillary block? Is
the duration shortened by CRF?
2. Choice of anesthetics: What induction drugs would you use? Is a rapid sequence
induction indicated? What are the pros and cons? Would you give more or less thiopental
to this patient? Which is the best inhalational agent? Will renal autoregulation be
affected?
3. Choice of relaxants: Would you use succinylcholine? What if the K+ were 5.0? Can
pancuronium be safely used? Why is atracurium your relaxant of choice? Is the duration
of mivacurium prolonged? After dialysis?
4. Maintenance: Which drugs would you choose for maintenance? Why? Which drugs can
have prolonged effects due to delayed excretion of active metabolites? Is MAC altered in
CRF? Why? What factors increase the sensitivity to intravenous anesthetics? How might
his intravascular volume affect your administration of drugs?
5. Ventilation: What are your goals for ventilation? What side effects can result from
hyperventilation?
6. Reversal: Is it safe to give reversal to patients with CRF? What special caveats apply?
C. Postoperative Care
1. Oliguria: There has been no urine output over the past 2 hours. What would you do?
How would you distinguish between prerenal, renal, and postrenal causes? Would a
FENa or UA useful? Is it better to give lasix or mannitol? Which is better for renal
protection? Could this be ATN? What part of the kidney is most vulnerable? Is there a
role for calcium entry blockers? Dopamine? What is the most effective (simplest)
measure to minimize the chances of acute tubular necrosis?
2. Acidosis: An ABG shows pH 7.25, pC02 30, p02 85. Interpretation? DDx?
136 The Essential Oral Board Review

Renal II (Session 1)

A 57 year old 90kg, 5*6" female presents for a laparoscopic cholecystectomy.

HPI: Pt has a history cholelithiasis and now acute cholecystitis as confirmed by ultrasound.

PMI: She has a history of chronic renal failure from hypertension, and dementia. She is
receiving Ca supplements, thiazide, ampicillin, gentamycin, and clindamycin.

PE: P 110, BP 160/90, R 22, T 39°C. The pt does not respond when spoken to. Auscultation
reveals bibasilar crackles and a 2/6 systolic ejection murmur over the precordium. Her
abdomen is diffusely tender.

CXR: Lower lobe atalectasis.

ECG: Left bundle-branch block.

Labs: Hct 23, K+ 5.5, C0218 mEq/L, bilirubin 8.8 mg/dl.


I 3 / The Essential Oral Board Review

Renal II (Session 2)

A 57 year old 90kg, 5'6" female presents for a laparoscopic cholecystectomy for acute
cholecystitis. She has a history of chronic renal failure, hypertension, and dementia. She is
receiving Ca supplements, thiazide, ampicillin, gentamycin, and clindamycin. P 110, BP 160/90,
R 22, T 39°C. Hct 23, K+ 5.5, bilirubin 8.8 mg/dl. EKG reveals a left bundle-branch block.

'•/■

138 The Essential Oral Board Review

Renal II (Questions)

A 57 year old 90kg, 5'6" female presents for a laparoscopic cholecystectomy for acute
cholecystitis. She has a history of chronic renal failure, hypertension, and dementia. She is
receiving Ca supplements, thiazide, ampicillin, gentamycin, and clindamycin. P 110, BP 160/90, "*
R 22, T 39°C. Hct 23, K+ 5.5, bilirubin 8.8 mg/dl. EKG reveals a left bundle-branch block. ^

A. Preoperative Evaluation ^
1. Chronic renal failure (CRF): What are the systemic manifestations of CRF? ^
2. Hematologic: Why the anemia? Is she likely to have a coagulopathy? What kind? ^
3. Cardiovascular: Why the hypertension? Do you think the CO would be high or low? <**
What is the significance of the LBBB? /^
4. Respiratory: Would it surprise you if the room air sat was 89%
5. Metabolic: Is her Ca2+likely to be high or low? Would you cancel the case due to the **
hyperkalemia? How can K+ be lowered? Would you administer Ca2+ preoperatively?
B. Intraoperative Management
1. Monitoring: Is an AL required? How would you monitor for myocardial ischemia? >«*
2. Induction: Should a rapid sequence be performed? Is succinylcholine contraindicated? k*-
3. Maintenance: Is N20 contraindicated in laparoscopies? Are opiates contraindicated ^
here? If biliary spasm were suspected, how could it be treated? Are you concerned ^
a b o u t a c c u m u l a t i o n o f a c t i v e m e t a b o l i t e s ? I s e n fl u r a n e c o n t r a i n d i c a t e d ? ^
4. Hypotension: C02 is used to create a pneumoperitoneum ad the patient is placed in the
Trendelenberg's position. After 30 minutes of surgery, the ETC02 is 50. What would you
do? Gradual hypotension develops. What would you do?
5. Reversal: At the end of the case, the patient has one twitch on TOF stimulation. Do you m
have any special concerns about reversing her neuromuscular blockade? Twenty minutes ^
after administering pyridostigmine and glycopyrolate, the patient is bucking on the ETT but ^
still does not offer a 5 second head lift. DDx? Would you administer edrophonium? ?**
C. Postoperative Care ^
1. Acute renal failure (ARF): Urine output declines. Is this patient at risk for ARF? How ^
does bilirubin act as a nephrotoxin? Would ketamine be a good way to restore renal /■»
b l o o d fl o w ? E p i d u r a l a n e s t h e s i a ? Wo u l d y o u b e g i n a d o p a m i n e i n f u s i o n ? W h i c h v
receptor is of interest, D1 or D2? Is there a role for calcium entry blockers? (D1 - renal,
splancnic, causes vasodilation; D2 - presynaptic sympathetic nerve terminals, decreases ^
NE release.) w
/*|

\w
*/%
V-

\
140 The Essential Oral Board Review

Renal III (Session 1)

A 90 year old 66 kg male is scheduled for a transurethral resection of the prostate.

HPI: Pt has prostatic hypertrophy and was recently treated for urosepsis.

PMI: Hypertension, COPD, and IDDM. He had a lateral wall Ml 5 months ago, and since then
has had decreased exercise intolerance. An echocardiogram revealed a 35% ejection
fraction with moderate hypokinesis. He is taking timolol, verapamil, ventolin, and NPH
60U/Regular 20U q AM and NPH 30U/Regular 10U q PM.

PE: P 50, BP 120/80, R 18, T 37.0°C. His airway exam is normal. Auscultation of his chest
reveals bilateral crackles and regular heart tones. His abdomen is benign.

CXR: Hyperinflated lungs consistent with emphysematous changes.

ECG: LVH with Q waves in V5-6.

Labs: Hgb 14 gm/dl, Na + 136 mEq/L, other electrolytes normal.

He did not take his morning insulin. His AM blood sugar is 190 mg/dl.
14 I The Essential Oral Board Review

Renal III (Session 2)

A 90 year old 66 kg male is scheduled for a transurethral resection of the prostate. He has
hypertension, COPD, and IDDM. He also had a Ml 5 months ago. He is taking timolol,
verapamil, ventolin, and NPH 60U/Regular 20U q AM and NPH 30U/Regular 10U q PM. HR 50
BP 120/80, R 18, T 37, Hgb 14, Na+136. EKG shows LVH and Q waves in V5-6. CXR shows '
hyperinflated lungs.

/9\

0S\

/9\

/UN

f9\
142 The Essential Oral Board Review

Renal III (Questions)

A 90 year old 66 kg male is scheduled for a transurethral resection of the prostate. He has
hypertension, COPD, and IDDM. He also had a Ml 5 months ago. He is taking timolol,
verapamil, ventolin, and NPH 60U/Regular 20U q AM and NPH 30U/Regular 10U q PM. HR 50,
BP 120/80, R 18, T 37, Hgb 14, Na+ 136. EKG shows LVH and Q waves in V5-6. CXR shows
hyperinflated lungs.

A. Preoperative Evaluation
1. Cardiac: Should delay surgery until at least 6 months have passed since his Ml?
Why/why not? Are his medications optimized?
2. IDDM: What are your perioperative goals in managing his diabetes? How would you
manage his insulin therapy?
3. Age: What special considerations are warranted because of the patient's advanced age?
Do changes occur with both pharmacokinetics and pharmacodynamics? Distinguish the
two.
4. Naj What other essential information do you require before proceeding?
B. Intraoperative Management
1. Monitors: How would you monitor this patient? If you had difficulty obtaining arterial
access, would you proceed without it? Is this procedure with significant volume shifts?
(Quantitate.) Is a CVP necessary? Is it a valid indicator of preload?
2. Anesthetic choices: The patient wishes to go to sleep. Is that acceptable to you? Is
there any advantage to spinal? The patient agrees to a spinal. What level of blockade do
you require? Why? What would you use for sedation? You decide on a propofol infusion.
How often do you plan to check serum Na+ values?
3. Bladder perforation: 30 minutes into the case, the patient becomes agitated and the HR
is 100. What would you do? He complains of left shoulder pain. Your response? Could
this result from represent mannitol absorption? Can you distinguish between extra- and
intraperitoneal perforation? Are there any early signs of rupture?
4. TURP Syndrome: He later becomes cyanotic. Your response? What is the TURP
syndrome? What properties of the irrigating solutions cause it? What is the soonest and
latest it can occur? What are the EKG findings (nodal, ST changes, U waves, widened
QRS)? Would you give the patient lasix or hypertonic saline? What are the dangers
associated with rapid correction of hypernatremia?
5. Positioning: At the end of the case, the BP suddenly decreases to 80/50 and the HR
increases to 110. DDx? Could this be due to lowering the legs?
C. Postoperative Care
1. Hypothermia: The nurses call you with a mixed venous gas showing a saturation of 60%.
Interpretation? The patient is shivering. Could this be sepsis?
2. Glycine toxicity: The patient complains of blurred vision. Do you think he has ammonia
toxicity (500 mcM)? What is the role of arginine in glycine toxicity?
3. Coagulopathy: Oozing is reported from IV sites. Do you think he has DIC? Why is he
predisposed? How would you evaluate? The platelet count comes back 100K.
Interpretation? Would you treat with DDAVP? EACA?
4. Hyperglycemia: Can sorbitol lead to hyperglycemia?
144 The Essential Oral Board Review

Liver I (Session 1)

A 39 year old, 120 kg, 6'0" male is schedule for porta-caval shunt placement.

HPI: Pt has a history of alcohol abuse and bleeding esophageal varices, for which he has
received multiple attempts at sclerotherapy. Claims to drink several six-packs of beer a
day and more than that on weekends.

PMH: IV drug abuse, multiple admissions for alcohol withdrawal.

PE: P 80, BP 90/50, R 20, Temp 36.6°C. The pt is combative and does not cooperate with
the airway exam. Breath sounds are diminished on the left. There is a 3/6 systolic
ejection murmur heard over the precordium. His abdomen is obese and distended with
ascites. His skin is jaundiced with palmar erythema and spider angiomas. He is vomiting
blood currently smells of alcohol.

CXR: Cardiomegaly, left-sided pleural effusion.

ECG: Sinus tachycardia with frequent unifocal PVCs.

Labs: Hgb 7.5 g/dl, platelets 89K, PT/PTT 35/66, Na+ 128 mEq/L, K+ 3.3 mEq/L, BUN 64, Cr
3.2, glucose 60 mg/dl, bilirubin 4.3, albumin 1.2.

He has a 20 gauge IV in his left hand. He has been typed and cross-matched for 2 units of blood.
_ 145 The Essential Oral Board Review

^ Liver I (Session 2)

^ A 39 year old, 120 kg, 6'0" male is schedule for porta-caval shunt placement for bleeding
esophageal varices. He has a history of IV drug abuse and alcoholism. There is tense ascites
(to and he smells of alcohol. P 80, BP 90/50, R 20, Temp 36.6°C, Hgb 7.5, BUN 64, Cr 3.2, bilirubin
4.3.

0\

f$&s
146 The Essential Oral Board Review

Liver I (Questions)

A 39 year old, 120 kg, 6'0" male is schedule for porta-caval shunt placement for bleeding
esophageal varices. He has a history of IV drug abuse and alcoholism. There is tense ascites
and he smells of alcohol. P 80, BP 90/50, R 20, Temp 36.6°C, Hgb 7.5, BUN 64, Cr 3.2, bilirubin
4.3.

A. Preoperative Evaluation
1. Renal function: What are your perioperative goals? What is the significance of the BUN
and Cr? Does he have renal failure? How would you evaluate his renal function?
2. Anemia: What do you think about his BP? About the Hb? Why is it low? How could you
distinguish between the possibilities? Could the BP and Hb be related? Would you
transfuse with blood preoperatively? Which is better, PRBC or whole blood? Should you
transfuse platelets too?
3. Cirrhosis: Is he ready for surgery? Do you this patient has liver failure? Is it important to
understand its pathophysiology? How does cirrhosis affect your anesthetic management?
Would you tap his ascites? What if he were confused? What are you minimum required
preoperative labs? Why are the PT/PTT elevated?
B. Intraoperative Management
1. IV access: Would you induce anesthesia with a peripheral 20G IV he already has in
place as the only access. Why not? He is agitated with central line placement. How
would you sedate him?
2. Monitors: How would you monitor? Is it important to monitor his temperature? What is
core temperature? How would you monitor it?
3. Induction: How would you induce anesthesia? Should he receive a rapid sequence
intubation? Define. What are the pros and cons in his case? Should the dose of
thiopental be reduced? What if you can't ventilate? Can't intubate?
4. Maintenance: Which narcotic would you use? Does it matter which inhalational agent
you use? Which muscle relaxant? How is the pharmacokinetics of relaxants vs.
narcotics altered with cirrhosis?
C. Postoperative Care
1. Hemolytic transfusion reaction: The urine is pink. DDx? Management? Would you
obtain a direct or indirect Coombs?
2. Delirium: The patient seems confused, tachycardic, and diaphoretic on postop day 2.
The nurses call you up for the most appropriate sedation. Which would you give, Valium
or midazolam? What is your plan?
148 The Essential Oral Board Review

Liver II (Session 1)

A 52 year old 85 kg, 5'9" male is scheduled for an orthotopic liver transplantation.

HPI: Pt has alcoholic cirrhosis. He has a prior history of cirrhosis, treated with lactulose and
neomycin. Since his hospitalization, he has been transfused with blood, FFP, and
platelets, and given furosemide for diuresis. His medications include vitamin K,
neomycin, and furosemide.
PMI: Multiple admissions for aspiration pneumonia, alcohol detoxification, and delirium
tremens. At one point, he was diagnosed with Wernicke-Korsakoff's syndrome.

He also has a 100 pack-year of tobacco use.

PE: P 100, BP 150/90, R 28, T 36.5°C. His airway is normal. His breath sounds have
scattered rhonchi and atalectasis. His heart tones are regular without rubs, clicks, or
murmurs. He is jaundiced and has ascites.

CXR: Cardiomegaly, prominent perihilar pulmonary vessels, plate-like atalectasis.

ECG: NSR, LVH.

Labs: Hct 26, K+ 3.2 mEq/L, Na+ 145 mEq/L, C02 33 mEq/L, BUN 40, Cr 1.9, albumin 2.2,
bilirubin 3.0, PT/PTT 16/32. ABG shows pH 7.33, pC02 35, p02 59.

He seems confused on the morning of surgery.


14y The Essential Oral Board Review

Liver II (Session 2)

A 52 year old 85 kg, 5'9" male with alcoholic cirrhosis is scheduled for an orthotopic liver
transplantation. He is jaundiced and has ascites. His medications include vitamin K, neomycin,
lasix. He seems confused on the morning of surgery. P 100, BP 150/90, R 28, T 36.5°C, Hct 26
K+ 3.2. ABG shows pH 7.33, pC02 35, p02 59.
1 50 The Essential Oral Board Review

Liver II (Questions)

A 52 year old 85 kg, 5'9" male with alcoholic cirrhosis is scheduled for an orthotopic liver
transplantation. He is jaundiced and has ascites. His medications include vitamin K, neomycin,
lasix. He seems confused on the morning of surgery. P 100, BP 150/90, R 28, T 36.5°C, Hct 26,
K+ 3.2. ABG shows pH 7.33, pC02 35, p02 59.

A. Preoperative Evaluation
1. Cirrhosis: Describe the pathophysiology of cirrhosis.
2. Respiratory: Describe his acid-base balance. Why is he hypoxemic? Should he receive
bicarb?
3. Hematologic: Why is he anemic? Why do you need to evaluate his coagulation? Which
factors are deficient? Why?
4. Cardiac: What is his CO? Why might it be elevated?
5. Electrolytes: Why is he hypokalemic?
6. Fluid status: Is his plasma volume high or low? His total body water? Why? Should he
receive preoperative diuresis?
7. Neurologic: Why do you think he might be confused? Could it be because his neomycin
was replaced by lactulose over the past week?
8. Child's classification: What is it? (Bili 2-3, Alb 3-3.5, ascites, enceph, nutrition) Does it
apply?
B. Intraoperative Management
1. Monitoring and access: Does it matter where the central line is placed? (not in arm for
Veno-veno bypass or in LE) Do all patients require PACs? How about children? How
would you monitor coagulation? Does a thromboelastogram offer any advantages over
the PT/PTT and CBC? How would you monitor temperature?
2. Induction: Is this patient at risk for aspiration? How would you induce anesthesia? A
colleague suggests ketamine. Do you agree? During induction, he coughs up blood.
What would you do?
3. Maintenance: What would you use for maintenance? Which inhalational agent? What is
their effect on hepatic blood flow? Would you say that inhalational agents are primarily
responsible for the reductions in hepatic blood flow? What other factors affect HBF?
Which neuromuscular relaxant?
4. Ventilation: Which ventilation is better, spontaneous or controlled with PEEP?
5. Bypass: Is Veno-veno bypass necessary? How does it help?
6. Hypotension: During the revascularization phase, a decrease in the ETC02 is detected.
Your response? Hypotension persists despite several units of blood? DDx? Would you
give bicarb? CaCI2?
C. Postoperative Care
1. Metabolic alkalosis: The patient's serum C02 is 35. DDx? (citrate, NaCI, NG tube,
lasix, volume contraction, PCN nonabsorbable anions carrying H+, aldosterone xs) What
is the harm in metabolic alkalosis? Does it reduce cerebral blood flow? (hypoK, hypoCa,
02-Hgb, NM blockade) If this were due to citrate intoxication, what would you expect the
K+ and CI" to be? (low in any case) How would you Rx? (NaCI, respiratory acidosis,
spironolactone).
2. Hepatorenal syndrome (HRS): After receiving a dose of ibuprofen, the urine is found to
contain elevated Na levels and casts. Do you think this is an early sign of the hepatorenal
syndrome? Is the cause of HRS toxic or ischemic in nature?
152 The Essential Oral Board Review

Fluids and Hematology I (Session 1)

A 14 year old boy presents with for amputation of both legs following a train accident.

HPI: Pt was playing near railroad tracks, when he fell and had his legs run over by a moving
train. He was rescued by paramedics, who started 14 gauge IVs in both arms. He was
given 02 by face mask and a total of 150 cc of normal saline.

PMI: Pt claims he is a Jehovah's Witness with hemophilia B. He has no allergies, does not
smoke or drink alcohol, and denies drug use. He has never had surgery before. The
family history is negative for anesthetic complications.

PE: P 130, BP 70/40, R 32, T 35.0°C. The pt is alert and oriented. His airway appears
normal. His lungs are clear to auscultation. His heart has a regular rate and rhythm.
Both legs are amputated below the knees. Tourniquets have been applied to both thighs.

CXR: Small right pneumothorax.

ECG: None obtained.

Labs: Hct 29, electrolytes and coagulation studies pending.

The parents have given their written permission to allow the child to die rather than receive blood
products.
53 The Essential Oral Board Review

Fluids and Hematology I (Session 2)

A 14 year old, 45 kg, 4'10" male presents with a traumatic amputation of both legs from a train
accident. He is awake, oriented, and claims that he is a Jehovah's Witness with hemophilia B
and would rather die than receive blood. P 130, BP 70/40, R 32, T 35.0°C, Hct 29.
154 The Essential Oral Board Review

Fluids and Hematology I

A14 year old, 45 kg, 4'10" male presents with a traumatic amputation of both legs from a train
accident. He is awake, oriented, and claims that he is a Jehovah's Witness with hemophilia B
and would rather die than receive blood. P 130, BP 70/40, R 32, T 35.0°C. Hct 29.

A. Preoperative Evaluation
1. Jehovah's Witness: How does the patient's religious status affect your initial
management? Fluid administration? Which fluid would you give? Why? Decision to give
blood? Would you proceed with surgery? Is it appropriate to call another colleague from
home to do the case?
2. Trauma: What are your priorities in caring for this patient in the ER? Does he need to be
intubated right away?
3. Fluid resuscitation: Which is better to give, crystalloids or colloids? How much would
you give? Is there a limit on hydroxyethylstarch? The surgeon says not to give any fluids
at all? Why do you think he said that? Is he correct? What are the theoretical problems
with crystalloid resuscitation?
4. Hemophilia B: What is it? The PT or the PTT or the TT expected to be elevated? How
can the missing factor be replaced?
B. Intraoperative Management
1. Monitors: Would you want a central line to measure mixed venous blood gases? You
are unable to place arterial catheters in both radial and both brachial arteries. Where
would you try next?
2. Induction: How would you induce anesthesia?
3. Intraoperative salvaging: Would you use intraoperative cell salvage? What are the
contraindications to its use? Would you use blood contaminated by fecal material if it has
been washed?
4. Massive blood transfusion: The blood pressure drops over 3 minutes to 60/40. Your
response? 1 L of blood is lost, but the BP is 80/30 after 2 L of crystalloid is given. Would
you transfuse? What are the indications? After how many units of PRBCs would you
administer platelets? FFP? Cryoprecipitate?
5. Hypocalcemia: The ETC02 decreases during transfusion. Your differential diagnosis
and treatment? Could this be an anaphylactic transfusion reaction? The surgeon says he
notices shortening of the QT interval and suggests administration of Ca2+. Do you agree?
6. Type and crossmatch: If no type specific blood had been available, what type would you
have ordered? After 10 units of type O blood, would you administer type specific blood if
it became available? What is a type and cross? A type and screen? Does the maximal
blood ordering schedule apply here?
7. Complications: What is the most common acid-base abnormality from a massive blood
transfusion? What is the most common complication in general? What would you tell the
parents about the risks of HIV infection? (1:30,000 to 1:300,000). Of hepatitis C and B?
(1:200,000 and 1:300-1:3,300). Could he contract malaria?
C. Postoperative Care
1. Disseminated intravascular coagulation (PIC): The patient is bleeding from IV sites
and his abdominal wound. What do you think is going on? What tests would you order to
confirm your suspicions? Define DIC? What is the pathophysiology. What is the
treatment? Is heparin indicated.
2. Delayed transfusion reaction: 3 weeks after the surgery, the patient experiences
anemia, jaundice, and fever. What could be going on? (Keif, Duffy, Kidd) Could this be a
hemolytic transfusion reaction? Would you perform a Coombs test?
156 The Essential Oral Board Review

Fluids and Hematology II (Session 1)

A 75 year old 59 kg, 57" male is scheduled for an open prostatectomy.

HPI: Pt has been diagnosed with prostate cancer from a routine prostate specific antigen
screen. His bone scan is negative.

PMI: Coronary artery disease with stable angina. He is able to climb 1 flight of stairs without
becoming short of breath. He also has left-sided upper extremity weakness from a stroke
suffered 10 year ago and a mild case of glaucoma. He is taking echothiophate, aspirin,
and propranolol.

PE: P 58, BP 160/80, RR 20, T 37.6°C. He is edentulous and has decreased range of motion
in his neck. His lungs are clear to auscultation. His heart has a regular rhythm, without
rubs, clicks, or murmurs. His abdomen is benign. There is atrophy and contractures of
his left upper extremity.

CXR: Mass in the left upper lobe.

ECG: NSR, Q waves in leads V4-6.

Labs: Hct 37, platelets adequate, Na+ 126, K 3.7, PT/PTT within normal limits.

The pt has heard about the possibility of HIV transmission through blood and wants you to do
whatever you can to avoid homologous blood transfusions.
57 The Essential Oral Board Review

Fluids and Hematology II (Session 2)

A 75 year old 59 kg, 57" male with prostate cancer is scheduled for an open prostatectomy. He
has coronary artery disease, partial left-sided hemiplegia from a stroke 10 year ago, and
glaucoma. He is taking echothiophate, propranolol. P 58, BP 160/80, RR 20, T 37.6°C, Hct 37,
Na+126. His CXR shows a mass in the left upper lobe.
158 The Essential Oral Board Review

Fluids and Hematology II (Questions)

A 75 year old 59 kg, 57" male with prostate cancer is scheduled for an open prostatectomy. He
has coronary artery disease, partial left-sided hemiplegia from a stroke 10 year ago, and
glaucoma. He is taking echothiophate, propranolol. P 58, BP 160/80, RR 20, T 37.6°C, Hct 37,
Na+126. His CXR shows a mass in the left upper lobe.

A. Preoperative Evaluation
1. Hyponatremia: What could be the cause of his low Na? (Hi, low, or normal volume)
Could it be related to the CXR finding? Is MAC affected? Other symptoms? What tests
would you order to help diagnose the problem? What is his total body Na deficit?
2. LUL mass: What is the significance of the lesion on CXR? What questions would you
ask? What if it were TB? Would you delay surgery to have it evaluated?
3. Glaucoma: What is glaucoma? (>25 mm) What factors affect intraocular pressure?
(aqueous humor, blood volume, CVP, EOM tone) How does echothiophate work?
(parasympathomimetic miosis opens iridocorneal angle, allowing aqueous flow to occur).
Should you avoid atropine? (IV anticholinergics do not affect IOP) Ketamine?
B. Intraoperative Management
1. Monitoring: How does the noninvasive BP device work? How does the temperature
device work? (thermistor = semiconductor, thermocouple = 2 metals with potential
difference) What happens to temperature regulation and gradients under GA? What are
the ASA standards for intraoperative monitoring?
2. Induction: Can induced hypotension be employed with an epidural catheter?
3. Avoidance of transfusion: How can the use of homologous blood be avoided? Which is
better-induced hypotension or isovolemic hemodilution? Any contraindications to either?
4. Positioning: Before proceeding, the surgeons want to place the patient in the dorsal
lithotomy and Trendelenberg's position for cystoscopy. Any deleterious effects? The BP
decreases. What would you do?
5. Primary fibrinolysis: Diffuse oozing is noted from his IV puncture sites. What would you
do? Is this DIC?
6. Pseudocholinesterase deficiency: The patient has no twitch on TOF at the end of the
case. DDx? What is the dibucaine #?
C. Postoperative Care
1. Femoral nerve injury: The patient is unable to move his left leg and his deep tendon
reflexes are diminished. Could this have been due to positioning or the retractors? How
would you investigate?
2. Acid-base balance: With the patient breathing 30% 02, the ABG reads pH 7.25, pC02
44, P02 155. Is this an anion gap acidosis? How can you tell? What is the anion gap? If
the anion gap were 25 mEq/L, how would that affect your therapy? (nl 9-15)

v
160 The Essential Oral Board Review

Fluids and Hematology III (Session 1)

A 68 year old, 70 kg, 5'4" female is scheduled for a uretosigmoidectomy.

HPI: Pt has bladder cancer, discovered with an episode of obstructive uropathy. Her
metastatic workup has been negative.

PMI: She has insulin dependent diabetes mellitis, hypercholesterolemia, and mild coronary
artery disease. She walks several blocks when she goes shopping without difficulty.
During a recent colonoscopy for excision of polyps, she bled profusely, requiring a blood
transfusion. During that event, she had a hemolytic transfusion reaction and was found
to have an extremely rare blood type. Her medications include insulin, clofibrate,
thiazide, and nitroglycerine.

PE: P 90, BP 150/70, R 20, Temp 37.0°C. Her uvula and tonsillar pillars can be completely
visualized, her neck is supple, and her thyromental distance is 6.5 cm. Her lungs are
clear to auscultation. Her heart exam reveals a 2/6 systolic ejection murmur over the left
upper sternal border. Her abdomen is obese.
CXR: No apparent disease.

ECG: Normal sinus rhythm, RBBB.

Labs: Na+127 mEq/L, glucose 500 mg/dl, triglyceride 310, creatinine 3.2.

10 units of her rare blood type are available for the procedure.
1 6 1 The Essential Oral Board Review

Fluids and Hematology III (Session 2)

A 68 year old, 70 kg, 5*4" female is scheduled for a uretosigmoidectomy for bladder cancer. She
has insulin dependent diabetes mellitis and a rare blood type. Her medications include insulin,
clofibrate, thiazide, and nitroglycerine. P 90, BP 150/70, R 20, Temp 37.0°C, Na+ 127 mEq/L,
glucose 500 mg/dl, triglyceride 310, creatinine 3.2.
1 62 The Essential Oral Board Review

Fluids and Hematology III (Questions)

A 68 year old, 70 kg, 5'4" female is scheduled for a uretosigmoidectomy for bladder cancer. She
has insulin dependent diabetes mellitis and a rare blood type. Her medications include insulin,
clofibrate, thiazide, and nitroglycerine. P 90, BP 150/70, R 20, Temp 37.0°C, Na 127 mEq/L,
glucose 500 mg/dl, triglyceride 310, creatinine 3.2.

A. Preoperative Evaluation
1. Hyponatremia: What is your DDx? (RF, thiazide, hyperglycemia 1.6 Mm/glu g/L,
hyperlipidemia) What is her likely total body H20? Symptoms of hypoNa*? Would you
Rx? (chronic well tolerated; cells lose Na+/K/AA) Deficit? [(0.5 x target Na+) - (current
Na)] Why"xTBW"?
2. Diabetes: Would you cancel the case? Wouldn't you want to know her baseline glucose
first? She claims that this is her usual glucose.
3. Acid-base balance: What sort of acid-base disturbance is likely to occur with obstructive
uropathy? (accumulation of P04"2, S04"2, organic acids with GFR <20 ml/min).
4. Transfusion: How would you handle her problem with the rare blood type? How often
can he donate his own blood? (q 3 days with Fe and ? Eo) If you found a compatible
donor in the family, is it safer to collect whole blood or PRBCs from this person?
B. Intraoperative Management
1. Monitors: How?
2. Anesthetic choices: Which?
3. Fluid choices: What fluid would you choose? Why? Explain your answer in terms of
oncotic and osmotic forces. What is the difference between osmolality, osmolarity, and
tonicity? How do you calculate osmolarity?
4. Colloids vs. crystalloids: Bleeding occurs. At what point would you administer
colloids? Pros/cons? (D40 antiplt, ARF, rare anaphylaxis, cost; good-microcirculation)
5. Cell-Saver: Would you use? What if contaminated by fecal contents? Evident that
cancerous cells are bad? Does her rare blood type alter your cell-saver transfusion
threshold? Would you use LR to wash blood?
6. Hypocalcemia: The BP drops to 80/40. Rx? Would you use the QT interval to guide
your therapy? What does Ca2+ do? (SVR)
7. Autologous products: At the end of the case, they finally locate his autologous units.
The surgeon tells you to give them. Would you?
C. Postoperative Care
1. Metabolic acidosis: To what electrolyte abnormality is this patient predisposed? (Bowel
absorbs NaCI in exchange for KHC03, causing a hyperchloremic, hypokalemic metabolic
acidosis)
2. Oliguria: From an endocrinologic perspective, is postop oliguria expected? Which is the
more potent stimulator of ADH release, volume (baroreceptors) or concentration
(osmoreceptors)? Where are they located? (carotid body for ADH secretion and afferent
renal arterioles = JGA for renin secretion)
I 63 The Essential Oral Board Review

Hyponatremia
1. High body H?Q - CHF, cirrhosis, CRF, nephrotic syndrome.
2. Normal body H?Q - SIADH, hyperglycemia, hypothyroidism, chlopropamide,
cyclophosphamide, vincristine, carbamezapine.
3. Low body H?Q - diuretic (mannitol, glucose), Addison's (mineraldo def), salt-losing
nephropathy, RTA. Symptoms: anorexia, N/V, weak, lethargy, confusion, SZ, coma, death.
Rx: Na+ rate < 0.5-1.5 mEq/L/hr, depending on symptoms, o/w central pontine myelinolysis.
Hypernatremia
1. Low body Na* - renal loss (diuretic), Gl, sweat.
2. Normal body Na* - Dl, bums, respiratory.
3. High body Na* - intake, hypertonic NS, NaHC03, hyperaldo, Cushing's. Symptoms: <125,
restlessness, lethargy, hyperreflexia, seizures, coma, death. Rx: using normal TBW x 140 =
present TBW x present [Na+], calculate deficit Normal TBW - Present TBW and give as D5W.
Metabolic acidosis
1. Anion gap - methanol, uremia, DKA, paraldehyde, ischemia, ethylene glycol, salicylate, sulfur.
2. Non-anion gap - Gl (diarrhea, resins, fistulae, ureterosigmoidostomy or obstructed ileal loop),
renal (RTA, CA inhibitor, hypoaldo), HC03\ TPN, massive Txf.
RTA (nonanion gap hyperchloremic)
I Distal - back leak or failure to resorb H+ (cannot lower urine pH), asso with low K+ and
demineralized bone (from hypercalcuria), may Rx with HC03.
H Prox- rare, defect in HC03' resorption (urine not acidotic because distal tubule resorbs bicarb),
other substances lost in urine (P042', glu, AA), often no need to Rx.
IV Hyporeninemic hypoaldosteronism (in most) - high K, can lower urine pH (unlike type 1) and
do secrete some HC03" (unlike type 2).
164 The Essential Oral Board Review

Fluids and Hematology IV (Session 1)

A 12 year old, 45 kg, 4'6" male is scheduled for a cadaveric renal transplantation.

HPI: Chronic renal failure from posterior post-streptococcal glomerulosclerosis. He has


received a prior renal transplant 5 years ago, which is now undergoing rejection. He has
been on chronic peritoneal dialysis. His last dialysis was 3 days ago.

PMI: He has a history of hypertension, diarrhea, and CHF. Despite extensive evaluation, the
cause of his diarrhea is unknown. He is taking prednisone, cyclophosphamide, calcium,
vitamin D, and phosphate binders.

PE: P 80/50, HR 100, K 6.0, Temp 36.4°C. The pt has steroid facies and a fat pad at the
base of his neck. His airway, lung, and cardiac exam are unremarkable.

CXR: No apparent disease.

ECG: NSR, peaked T waves.

Labs: Hct 22, Na+ 137 mEq/L, K+ 5.9 mEq/L, C0219 mEq/L, CI" 105 mEq/L, Ca2+ 7.2. P04"2 4.0.
PT/PTT normal.

Prior to surgery, the pt received 1 unit of packed RBCs from the renal donor.
165 The Essential Oral Board Review

Fluids and Hematology IV (Session 2)

A 12 year old, 45 kg, 4'6" male is scheduled for a cadaveric renal transplantation.
He has a history of hypertension, diarrhea, and CHF. He is taking prednisone, calcium, vitamin
D, phosphate binders. P 80/50, HR 100, K+ 6.0, Temp 36.4°C. Hct 22.
166 The Essential Oral Board Review

Fluids and Hematology IV (Questions)

A 12 year old, 45 kg, 4'6" male is scheduled for a cadaveric renal transplantation.
He has a history of hypertension, diarrhea, and CHF. He is taking prednisone, calcium, vitamin
D, phosphate binders. P 80/50, HR 100, K+6.0, Temp 36.4°C. Hct 22.

A. Preoperative Evaluation
1. Immunosuppression: Does he need a transfusion prior to surgery? The surgeons order
2 units of PRBCs. Should they be WBC depleted? Effect on graft survival? On
malignancy? On ID? Why? Is he at risk for GVH disease? For hepatitis?
2. Hyperkalemia: Why is he hyperkalemic? Symptoms?
3. Diarrhea: What electrolyte abnormality is likely to result?
B. Intraoperative Management
1. Monitors: What are the routine ASA monitors? Which are continual vs. continuous?
2. Anesthesia: Is he a good candidate for isovolemic hemodilution? For induced
hypotension?
3. Ventilation: What are your goals for intraoperative ventilation?
4. Anaphylactic transfusion reaction: Profuse bleeding occurs at the surgical site. What
would you do? After 10 cc of transfused cells, the ETC02 drops to 15. Your response?
Could his response to blood be related to his chronic diarrhea? Is this a common form of
transfusion reaction?
5. Transfusion reactions: What is the most common one? Could both have been
prevented with use of appropriate filters? Are white cell or platelet sensitization the same
cause of urticarial reactions? After 10 units of PRBCs, is there some other blood product
you should think of administering? How much would you order?
C. Postoperative Care
1. Hyponatremia: You are called to the ICU to treat a seizure. You notice that all the IVFs
are D5W. Would you administer 3% NS? Risks vs. benefits?
2. Die: The PT/PTT are 20/65 and the pit count is 50. Likely syndrome? Define DIC.
Differential from primary fibrinolysis. Rx with aprotinin, DDAVP, EACA, or heparin?

v.

^v
167 The Essential Oral Board Review

Hyperkalemia
1. Pseudo - hemolysis, WBC, pit
2. Shift - acidosis, hypertonicity, rhabdomyolysis, injury, BB, periodic paralysis
3. Decreased excretion - RF, mineraldo def (Addison's or primary = RTA 4), spironolactone,
ACEI, NSAIDs, decreased distal nephron excretion (pseudoaldo, amiloride, triamterene,
sickle cell
4. Increased uptake - blood, PCN, KCI; Symptoms: weak > 8 mEq/L due to sustained depol,
cardiac > 7: tT, wide QRS, tPR, iP, |R, 1ST, sine

Hypokalemia
1. Shifts - alkalosis, I, B, periodic paralysis
2. Losses - renal (diuretic, mineraldo xs, licorice, CAH, RTA, Bartter's-hyper-renin/hyperaldo due
to TAL defect in NaCI resorption leading to hyperaldo, which causes hypoK), salt-wasting
nephropathy, amphoB, hypoMg, uretoileostomy; Gl (diarrhea, laxatives, vomiting, fistula),
sweat; dialysis
3. Decreased uptake
Symptoms: weak, ileus, cramps, tetany, ADH resistance, alkalosis, cardiac T flat, U appears, 4T,
tP, tPR

Metabolic alkalosis
1. Chloride-sensitive (ECF depletion, with Na resorption, H excretion because insufficient CI to
accompany Na, leading to reclamation of HC03) - Gl (V, Gl drain, diarrhea, villous
adenoma), renal (diuretic, posthypercapneic), CF
2. Chloride-resistant (mineraldo xs) - increased mineraldo effect (primary hyperaldo, secondary
hyperaldo, Cushing's, licorice, Barrier's), severe hypoK*
168 The Essential Oral Board Review

Fluids and Hematology V (Session 1)

A 39 year old, 80 kg, 6'1" male is scheduled for resection of a renal mass.

HPI: Pt was found to have a renal mass after workup for anemia and hematuria. Abdominal
CT scan revealed a 12 x 8 cm renal mass wrapped around the inferior
vena cava. Metastatic workup was negative. The presumed diagnosis is a renal cell
carcinoma.

PMI: Hepatitis B 10 years ago, asymptotic and well-controlled HTN, migraine headaches
occurring every month. The pt is a Jehovah's witness and refuses all blood products,
except that from a cell-saver. He is the single parent or 2 children, ages 5 and 8. His
medications include furosemide and propranolol.

PE: P 110, BP 130/60, R 22, Temp 36.5°C. His airway, lung, and cardiac exams are normal,
except for a 2/6 systolic ejection murmur over the precordium.

CXR: Normal.

ECG: NSR, LVH.

Labs: K* 2.9 mEq/L, C02 30 mEq/L, BUN 20, Cr 1.9, Hct 26. Other labs are unremarkable.
rim

/ fl y 169 The Essential Oral Board Review

/Pfflr

Fluids and Hematology V (Session 2)

A 39 year old, 80 kg, 6T male is scheduled for resection of a large renal mass wrapped around
the inferior vena cava. The pt is a Jehovah's witness and refuses all blood products, except that
from a cell-saver. His medications include furosemide for HTN and propranolol for migraine
headaches. P 110, BP 130/60, R 22, Temp 36.5°C. He has a 2/6 systolic ejection murmur over
the precordium. ECG: NSR, LVH, K+ 2.9 mEq/L, C02 30 mEq/L, BUN 20, Cr 1.9, Hct 26.

^kT

dpf

zip?
170 The Essential Oral Board Review

Fluids and Hematology V (Questions)

A 39 year old, 80 kg, 6*1" male is scheduled for resection of a large renal mass wrapped around
the inferior vena cava. The pt is a Jehovah's witness and refuses all blood products, except that
from a cell-saver. His medications include furosemide for HTN and propranolol for migraine
headaches. P 110, BP 130/60, R 22, Temp 36.5°C. He has a 2/6 systolic ejection murmur over
the precordium. ECG: NSR, LVH, K+ 2.9 mEq/L, C02 30 mEq/L, BUN 20, Cr 1.9, Hct 26.

A. Preoperative Assessment
1. Anemia: Why is the pt anemic? What is the compensation? Do you think the 02 deliveiy
is adequate? Equation for D02? Do you plan to use 100% 02 to improve 02 delivery?
2. Renal failure: Is he in renal failure? How is it defined?
3. Jehovah's Witness: Are you going to tell him that without blood, he may die? Are you
going to try to talk him out of it? Is it appropriate for you to refuse to do the case?
4. HR: What is propranolol? For what disease process is the pt receiving propranolol? Is
he adequately p-blocked? What would be a more reasonable HR? Would you delay the
case to achieve that HR? Did you overlook the anemia? Would you like to retract that HR
goal? Why? How do you know his D02 wouldn't be adequate with a lower HR despite his
anemia?
5. Lasix: Would you delay surgery to correct his K+? What would be a more reasonable K+?
How do you know?
B. Intraoperative Management
1. Monitor: How? Why? PAC? How will it help you? Do you need an oximetric Swan?
How does it work?
2. Intubation: Special technique, given pt's problems (anemia)?
3. Maintenance: Special technique, given pt's problems (bleeding)? Would an epidural be
useful? Would it decrease blood loss? Would intrathecal narcotics help?
4. Bleeding: How prevent? Which technique is most effective?
5. Crystalloid/colloid: Severe blood loss occurs. The BP is 50/30. Which would you give
crystalloid or colloid? Albumin or Hespan? Will JW pts accept albumin?
6. Transfusion: Would you give cell saver blood if it were contaminated by tumor cells and
fecal contents? After several units of cell saver, the BP is still low. Should you give Ca2+?
BP 40/15. O negative units become available. How many units would you give?
7. Oximetric Swan: Would you look at it? MV sat is 94%. Do you think the pt is dying?
C. Postoperative Care
1. Ischemia: What postop complication do you fear the most? Which organs are at greatest
risk for ischemia? Doesn't autoregulation help reduce the chances of ischemia?
2. Anemia: At what Hct would you discharge the pt? 2.9? 3.5? 4.5? Why?
172 The Essential Oral Board Review

Respiratory I (Session 1)

A 7 year old, 20 kg boy is scheduled for laser excision of laryngeal polyps.

HPI: Juvenile laryngeal polyposis with several prior laser excisions. He was scheduled for
surgery 1 week from today, but has become audibly stridorous with labored respirations.

PMI: He is moderately mentally retarded from Down's syndrome. He also has a generalized
tonic-clonic seizure disorder, which is well controlled by phenobarbitol and tegrelul.
Besides these problems, his pediatrician claims he is otherwise healthy.

PE: P 100, R crying, Hb 14, Temp 37.0°C. The pt is combative, making further examination
difficult. He is audibly stridorous and has intercostal retractions. It takes both his parents
to hold him down.

CXR: None.

ECG: None.

Labs: Hgb 14 gm/dl.

The parents want to accompany the child to the operating room for induction. The pediatric
otolaryngologist does not mind.
/P\

173 The Essential Oral Board Review

Respiratory I (Session 2)
/?fSr

A 7 year old, 20 kg boy is scheduled for laser excision of laryngeal polyps. He is mentally
retarded, has a seizure disorder, and is audibly stridorous. His medications include phenobarbitol
and tegretol. He is combative and has no IV. BP unobtainable due to patient movement, P 100,
R crying, Temp 37.0°C, Hgb 14 gm/dl.

/0S
174 The Essential Oral Board Review

Respiratory I (Questions)

A 7 year old, 20 kg boy is scheduled for laser excision of laryngeal polyps. He is mentally
retarded, has a seizure disorder, and is audibly stridorous. His medications include phenobarbitol
and tegretol. He is combative and has no IV. BP unobtainable due to patient movement, P 100,
R crying, Temp 37.0°C, Hgb 14 gm/dl.

A. Preoperative Evaluation
1. Lasers: What is a laser? What type will they likely use? Does the laser wavelength
matter to you?
2. Seizure disorder: What is the significance of his seizure disorder? How do his
medications affect your choice anesthetics? Will they prolong the duration of action of
thiopental? Would you like to see levels? The labs were lost. Would you proceed without
them? Aren't you concerned about intraoperative seizures?
3. Mental retardation: Does mental retardation affect your history and physical? What
organ systems are likely affected? If he had Down's syndrome, does he need any special
premedications? SBP prophylaxis?
4. Premedications: What would use to premedicate him? Is he a full stomach? Would you
administer bicitra? The student suggests you pass an NG to administer it. What would
you use to sedate him.
5. Airway obstruction: The student asks about is the significance of his stridor. What is
the likely source? Explain to the student why. He asks whether you would order a flow-
volume loop on this patient? Draw what you would expect to see.
B. Intraoperative Management
1. Airway control: The surgeons state the ETT will be in their way in the beginning of the
case. How will you oxygenate and ventilate the patient? What is apneic oxygenation?
What are some of the problems with jet ventilation?
2. Induction: How would you induce anesthesia? Why is an inhalation anesthetic a bad
idea?
3. Maintenance: After 1 hour of multiple attempts, you are able to intubate the patient. How
will you maintain anesthesia? Would you prefer a thiopental, propofol, ketamine, or
etomidate infusion? Is it better to relax the patient or allow him to spontaneously
ventilate? Are there any drugs you would avoid?
4. Hypoxemia: The patient develops desaturation during jet ventilation. What would you
do? DDx? What else? What else? Would you check the neuromuscular stimulator?
You notice the patient breathing with severe retractions? Would you relax the patient?
5. Airway fire: After intubation, the surgery continues. How would you maintain
anesthesia? How would you handle an airway fire? How would you extubate the patient?
Is you decision affected by the difficult intubation?
6. Hypoxemia: The sat drops to 85%. Management? Peak airway pressure rise to 30.
DDx? Would you use the fiberoptic bronchoscope? What is the significance of crackles?
C. Postoperative Care
1. ARDS: The lungs develop bilateral opacification, the peak airway pressures continue to
rise, and on a Fi02 of 80%, rate 30, pressure limited setting of 29/3 the Pa02 is 96 mm
Hg. What is going on? Would you read just his ventilatory setting? Does PEEP help or
hurt in this setting? What is the best PEEP?
2. Starling's Law: The patient develops a prolonged ICU stay. How does nutrition affect
the development of pulmonary edema? What is Starling's law? How does cardiac
function affect the formation of pulmonary edema? Does CVP have an effect? Would the
administration of albumin decrease his edema?
176 The Essential Oral Board Review

Respiratory II (Session 1)

A 59 year old, 89 kg. 6'1" is scheduled for resection of a left upper lobe pulmonary bleb.

HPI: Pt is a coal miner. He has a chronic cough productive of thick, dark sputum. He also has
a 120 pack-year smoking history, but has stopped 5 days ago. He has been hospitalized
3 times over the past 5 years for pneumonia.

PMI: He has received bleomycin several years ago for testicular cancer. He drinks 2-3 six
packs of beer every weekend. His family history is negative for adverse intraoperative
events. He is allergic to penicillin.

PE: P 80, BP 150/80, R 26, Temp 38.7°C. Only his soft palate can be seen during his airway
exam. Auscultation of his lungs reveals decreased breaths sounds bilaterally. His heart
tones are also distant. He has clubbing in both hands.

CXR: Multiple areas of fibrosis and bilateral blebs, the largest in the left upper lobe.

ECG: Atrial fibrillation, RVH, right bundle branch block.

Labs: Hgb 12, electrolytes normal except for a C02 of 33 mEq/L. ABG pH 7.36, pC02, 52, p02
69. Spirometry reveals an FEV1 and FVC of 60% predicted values and an FEV1/FVC
ratio of 55%.
ill The Essential Oral Board Review

Respiratory II (Session 2)

A 59 year old, 89 kg. 6T coal miner is scheduled for resection of a left upper lobe pulmonary
bleb. He has cough productive of thick, dark sputum. He drinks 2-3 six packs of beer every
weekend and has an 80 pack-year smoking history. He has received bleomycin several years
ago for testicular cancer. On airway exam, you can only see his soft palate. P 80, BP 150/80, R
28, Temp 38.7°C, Hgb 12, EKG shows atrial fibrillation, right bundle branch block and RVH.
178 The Essential Oral Board Review

Respiratory II (Questions)

A 59 year old, 89 kg. 6'1" coal miner is scheduled for resection of a left upper lobe pulmonary
bleb. He has cough productive of thick, dark sputum. He drinks 2-3 six packs of beer every
weekend and has an 80 pack-year smoking history. He has received bleomycin several years
ago for testicular cancer. On airway exam, you can only see his soft palate. P 80, BP 150/80, R
28, Temp 38.7°C, Hgb 12, EKG shows atrial fibrillation, right bundle branch block and RVH.

A. Preoperative Evaluation
1. Pulmonary: What preoperative information would you like to know before proceeding
with surgery? Would you expect his PFTs to reveal a restrictive or obstructive lung
disease? Anesthetic significance of either? Would you ask the patient to stop smoking?
ABG pH 7.36, pC02, 50, p02 68. Is he ready for surgery?
2. Cardiac: What is the significance of his EKG findings? Should be on digoxin?
Prophylactically? Is he at increased cardiac risk? His echocardiogram shows right atrial
enlargement, right ventricular hypertrophy, tricuspid insufficiency, and preserved left
ventricular function. Significance? Does this mean the right ventricular function is also
preserved?
3. Alcohol: What is the significance of its use? Are there any respiratory effects of chronic
abuse? Does it cause shunt?
4. Fever: Would you cancel surgery because of the fever? Why/why not?
B. Intraoperative Management
1. Monitoring: How would you monitor this patient? Does it matter where you place the
AL? Is a PAC necessary? Would a CVP be sufficient? Does the positioning of the
patient affect you use of these monitors?
2. Induction: How do you plan to intubate this patient? Is a right or left-sided double lumen
ETT indicated?
3. Difficult airway: The patient becomes difficult to ventilate. What would you do? At what
point would you give succinylcholine? What choices do are available for the patient that
you can't ventilate/can't intubate, bearing in mind he will later need one-lung ventilation?
4. Hypoxemia: With a single lumen ETT, the patient's ABG shows pH 7.50, pC02 40, p02
51. Adjustments on the ventilator? DDx? Would you administer 100% 02 during the
case? Does the history of previous bleomycin exposure affect your decision? BS are
louder on the left than right. Would you expect to detect a right mainstem intubation with
the capnogram? How do you know by how much to pull the ETT? Could this be a
pneumothorax? How could you tell?
5. One-lung ventilation: How would you provide OLV? What are your options? How do
you confirm its position?
6. Cor pulmonale: Following left pneumonectomy with the chest still open, the CVP
increases to 15. DDx? Could this represent a recurrence of the pneumothorax? How
could you tell? What would you do? How would you treat suspected right-sided failure?
C. Postoperative Care
1. Thoracic epidural: How would you control postop pain? Thoracic vs. lumbar epidural?
Would you insert an epidural if the patient were still intubated?
2. Pneumonia: Copious amount of thick secretions are suctioned from the ETT by the ICU
nurses. Could you decrease them with the help of any agent? What is the effect of
glycopyrolate on the ETC02? A right upper lobe infiltrate seen by CXR is called
aspiration pneumonia by the surgeons. Your response?
180 The Essential Oral Board Review

Respiratory III (Session 1)

A 24 year old, 89 kg, 5'11" male presents for mediastinoscopy.

HPI: Pt complains of increasing shortness of breath when he lays down. CXR revealed a
large anterior mediastinal mass. CT scan shows that the mass compresses the trachea.
Ultrasound shows that the mass impinges upon the right atria and ventricle. Flow-volume
loops shows a FEV1 of 50% predicted with a normal FVC.

PMI: He has a 10 pack year smoking history and severe asthma.

PE: P 90, BP 120/80, R 26, Temp 36.7°C. He has a normal airway exam, but his face looks
plethoric. He pt is unable to lay back beyond 45 degrees due to severe dyspnea. Breath
sounds are clear bilaterally. Heart tones are normal.

CXR: Large anterior mediastinal mass.

ECG: Normal sinus rhythm, right axis deviation.

Labs: Hct 35, electrolytes normal.

His oncologist does not wish to perform radiation therapy prior to mediastinoscopy in order to
obtain tissue samples for an accurate diagnosis.
S®\

181 The Essential Oral Board Review

Respiratory III (Session 2)

A 24 year old, 89 kg, 5'11" male with a large anterior mediastinal mass presents for
mediastinoscopy. He has a 10 pack year smoking history and asthma. Every time he lays back
beyond 45 degrees, he becomes dyspneic. Face looks plethoric. P 90, BP 120/80, R 26, Temp
36.7°C, Hct 35.

wpi
182 The Essential Oral Board Review ^^

Respiratory III (Questions) X

A 24 year old, 89 kg, 5'11" male with a large anterior mediastinal mass presents for
mediastinoscopy. He has a 10 pack year smoking history and asthma. Every time he lays back /9s
beyond 45 degrees, he becomes dyspneic. Face looks plethoric. P 90, BP 120/80, R 26, Temp ^'
36.7°C, Hct 35. ^
ms
A. Preoperative Evaluation ^
1. Mediastinal mass: What are your priorities for this case? Should he receive ^/%
chemotherapy before surgery to reduce the tumor mass? Do you need to see the CT ^
scan? Flow-volume loops? Echocardiogram? Do you think he has a restrictive or ^
obstructive lung disease? ^
2. Asthma: If this patient were wheezing, would you cancel the case? What is the cause of J^
wheezing? What drugs are used to stop and prevent wheezing? .
3. Tobacco: Should he stop smoking? Why? Why might the complication rate increased M
after acute discontinuation? ^
/tgS\

B. Intraoperative Management 5|
1. Monitors: Do you have any special concerns for monitoring? Does it matter where you w.
place the AL? Would you place a central line? Where?
2. Intubation: How would you intubate this patient? What nerve blocks would you perform ^
and why? What precautions would you take? What position should the patient be in?
What drugs are best for preventing bronchospasm? Should you place this patient on fem-
fem bypass before induction? mm
3. Hypoxia: After fiberoptic intubation, the patient coughs and turns blue? Why? Does this
have anything to do with his closing volume? Would you administer succinylcholine?
4. Hypotension: The BP on the AL reads 50/20. What would you do? The noninvasive V^
monitor reads 80/50. Which do you believe? Would you sit the patient up, turn him prone,
or perform a stat median sternotomy?
5. Hemorrhage: Massive hemorrhage occurs in the chest. Where would you place the jt
central line for resuscitation? .

C. Postoperative Care >&


1. Extubation: How would you extubate this patient? What are your goals? To prevent ^
cough, it is better to give IV or intratracheal lidocaine? ^
2. Complications: The patient has a hoarse voice postoperatively. The nurses tell the y™
patient it is likely due to the endotracheal tube. Do you agree? What complications do ^
you expect from this procedure? If he has right sided hemiplegia, could that be due to ^
anesthesia? To innominate artery compression? His breath sounds are diminished on *
the left. Could this be due to phrenic nerve injury? How could you tell? *"*
/slsy

*n
184 The Essential Oral Board Review

Respiratory IV (Session 1)

A 60 year old, 110 kg, 5'10" male is scheduled for a left pneumonectomy.

HPI: Pt developed several episodes of pneumonia beginning 6 month ago, which were found
to be due to epidermoid carcinoma of the left lung. Since then, the tumor has almost
completely obstructed the left lobe of the lung. Metastatic workup was positive
metastases in the brain. Radiation therapy to the brain is scheduled to follow resection of
the left lung.

PMI: 60 pack-year smoking history, moderate COPD, well-controlled asthma, and


hypertension. He takes nifedipine, theophylline, and thiazide.
PE: P 75, BP 160/109, R 26, Temp 36.8°C. Limited oral aperture, allowing visualization of
the only the base of the tongue. Decreased breath sounds on the left. Heart tones are
normal.

CXR: Opacification of the left lobe. Emphysematous changes in the right lobe.

ECG: NSR, RBBB, RVH.

Labs: Hct 52, platelets normal; ABG pH 7.38, pC02 50, p02 59; electrolytes normal except for a
C02 of 31 mEq/L.
185 The Essential Oral Board Review

Respiratory IV (Session 2)

A 60 year old, 110 kg, 5'10" male is scheduled for a left pneumonectomy for epidermoid
carcinoma of the lung. He has a 60 pack-year smoking history, COPD, asthma, and
hypertension. He has clubbing and complains of exertional dyspnea. He takes nifedipine,
theophylline, and thiazide. P 75, BP 160/109, R 26, Temp 36.8°C, Hct 52.

(Sf

/IPs

m£y
186 The Essential Oral Board Review

Respiratory IV (Questions)

A 60 year old, 110 kg, 5'10" male is scheduled for a left pneumonectomy for epidermoid
carcinoma of the lung. He has a 60 pack-year smoking history, COPD, asthma, and
hypertension. He has clubbing and complains of exertional dyspnea. He takes nifedipine,
theophylline, and thiazide. P 75, BP 160/109, R 26, Temp 36.8°C, Hct 52.
A. Preoperative Evaluation
1. COPD: What are your goals for managing this case? What is the significance of his
DOE? Of his clubbing? What is COPD? What type do you think he has? What does a
Hct of 52 mean to you?
2. Preoperative Laboratory Assessment: What PFTs would you like to see? His FEV1 is
1.6L. Significance? Is this a restrictive or obstructive disease? (forced) Should he
receive a xenon scan? His ABG on 2L NC is pH 7.33, PaC02 55, Pa02 60.
Interpretation? Do you expect his serum bicarb to be high or low? Would you want to
know his serum theophylline level? Why?
3. Pneumonia: He is actively coughing up sputum and L sided breath sounds are
diminished. Dont you think you should delay the case? What exactly would you say to
the surgeon?
4. Hypertension: What is the significance of his BP? Would you cancel the case? What if
the DBP was 112? 120?
B. Intraoperative Management
1. Monitors: Would you use ETC02 monitoring? Why/why not? If you have pulse oximetry,
capnography, and a NIBP, is an AL really necessary? What are some arguments against
placement of a PAC?
2. PLT: Would you use one-lung anesthesia? What type of ETT would you use? Why
might a R-sided DLT superior to a L? Isn't a L-sided DLT contraindicated for a L
pneumonectomy? What is the standard of care for confirming placement of a L-sided
DLT? If intubation was difficult, are there any alternatives to the DLT?
3. Choice of Anesthesia: What is the best IV induction drug? Is propofol better than
ketamine? Is halothane the best inhalational agent? Every time you turn on the
inhalational agent, the SBP drops to below 100. Would N20 be useful? What are the
hemodynamic effects of N20? Are neuromuscular relaxants necessary?
4. Management of OLV: How would you set the ventilator? What are the effects of OLV on
lung function? Do muscle relaxants and thoracotomy add to these problems? On Fi02 of
100%, The pulse oximeter reads 99% and ETC02 is 25. The CRNA wants to send off an
ABG. Do you agree? ABG pH 7.25, PaC02 45, Pa02120. The CRNA wants to add
PEEP. Do you agree? How can oxygenation be improved during OLV? Why? The pulse
oximeter reads 66%. Mgmt?
5. Tachycardia: During maintenance with fentanyl, midazolam, vecuronium, and halothane,
the HR suddenly increases to 140 with PVCs. Would labetolol be a good drug to use?
Esmolol? What else do you want to know? Whatever the cause, isn't immediate
treatment indicated? What if ST segment depression occurs? 5 minutes after esmolol is
given, the peak airway pressure alarm goes off. Your response?
C. Postoperative Care
1. Changing the DLT: Are you going to extubate this patient? If the intubation was difficult,
how would you change the DLT? Following removal of the DLT, you are unable to open
the jaw and the patient develops paradoxical breathing. What would you do? Would you
perform transtracheal jet ventilation?
2. Negative pressure pulmonary edema: Following intubation, the saturation is low. What
would you do? Crackles are heard t^laterajjy^ DDx? Mgmt?
187 The Essential Oral Board Review

(m\
188 The Essential Oral Board Review

Respiratory V (Session 1)

A19 year old 66 kg, 5'5" woman is scheduled for a dilatation and curettage for heavy vaginal
bleeding.

HPI: Pt has an incomplete abortion several hours ago.

PMI: She has a history of asthma with multiple intubations and admissions to the ICU. Her last
admission was 2 months ago, during which she had a prolonged period of intubation will i
high dose steroids and isoproterenol infusion. She takes aminopnylline, cromolyn, and
an albuterol inhaler.

She last ate 2 hours ago.

PE: BP 70/40, HR 116, R 22, T 98.0°C. Airway exam normal. Bilateral wheezing is heard.
Her hands and feet are cold. No murmurs are heard. Her abdomen is soft.

CXR: NAD.

ECG: None obtained.

Labs: Hct 26, theophylline level and electrolytes are pending.

The surgeon wants to operate as soon as possible to control the vaginal bleeding.
/FN

189 The Essential Oral Board Review

Respiratory V (Session 2)

A 19 year old 66 kg, 5'5" woman is scheduled for a dilatation and curettage for heavy vaginal
bleeding. She has a history of asthma with multiple intubations and admissions to the ICU. She
chronically takes aminophylline, cromolyn, and an albuterol inhaler. She is currently wheezing
and last ate 2 hours ago. BP 70/40, HR 116, R 22, T 98.0°C. Hct 26.
190 The Essential Oral Board Review

Respiratory V (Questions)

A 19 year old 66 kg, 5'5" woman is scheduled for a dilatation and curettage for heavy vaginal
bleeding. She has a history of asthma with multiple intubations and admissions to the ICU. She
chronically takes aminophylline, cromolyn, and an albuterol inhaler. She is currently wheezing
and last ate 2 hours ago. BP 70/40, HR 116, R 22, T 98.0°C. Hct 26.

A. Preoperative Evaluation
1. Pulmonary: How would you induce this patient? What, in general, are your reasons for
ordering PFTs? Do you need them for her? Would a diffusion of CO test be helpful?
How? How would you optimize her preop status?
2. Anemia: What is the significance of her anemia? Does her asthma affect how you
answer this question? How would you determine whether or not she required a
transfusion? Are you more inclined to transfuse her because of her asthma? What are
the determinants of 02 delivery? What is the Fick equation? Ready for surgery?
3. Medications: Would you continue her preoperative medications? How does cromolyn
work? Would you deliver a few puffs of albuterol before induction?
B. Intraoperative Management
1. Monitoring: How would you monitor her respiratory function? Is an AL necessary with a
pulse oximeter and capnogram? Why? What is dead space? What is shunt? How would
you monitor both?
2. Induction: How would you induce this patient? Does it matter what drugs you use? If
your goal is to prevent bronchospasm, aren't all induction agents the same? Which is
more important, to prevent bronchospasm or to prevent aspiration? Preventing
bronchospasm or hypotension?
3. Bronchospasm: Immediately following intubation, the peak airway pressure is 60, no
breath sounds are heard, and there is no ETC02. What would you do? What do you think
is going on? How could you tell? Are you sure? Would you remove the ETT? Would you
administer more ventolin? Would you increase the depth of anesthesia with isoflurane?
Isn't halothane better for treating bronchospasm? Would you cancel the case?
4. Choice of anesthetics: What anesthetics are best for her condition? Which would you
avoid? On a combination of halothane, fentanyl, and pancuronium for maintenance, PVCs
develop. What would you do?
5. Mucous plug: The pulse oximeter reads 98%. What would you do? Would you
reposition the probe? It now reads 95%. Now what you do? 90%. 85%. The peak
airway pressure is 25. 30. 35. 40.
6. Extubation: What are your criteria for extubation? Would you perform a deep extubation
on this patient? Pros and cons?
C. Postoperative Care
1. Delayed emergence: Why could her emergence be delayed? The ABG shows pH 7.15,
pC02 66, p02 89. How would you describe her acid-base status? Does the pH concern
you? Would you give bicarbonate?
2. Ventilator: What ventilator settings would you order? What are your goals? How does
the ventilator used in the ICU differ from the one you use in the OR? If she were in status
asthmaticus, would high frequency ventilation help? Based on the new time constant, how
would you adjust the inspiratory/expiratory ratio? Would PEEP? Could hypotension
occurring in this patient be related to the ventilator? When would you use pressure
support?
192 The Essential Oral Board Review

Respiratory VI (Session 1)

A 16 year old, 40 kg, 5'0" female presents for removal of hardware from a tibia/fibular fracture.

HPI: Pt fell on her leg 1 month ago, when she had an external fixator placed on her tibia and
fibula. She was thought to have a difficult airway at the time, and was intubated awake
via a blind nasal technique. Later, she reported intraoperative recall.

PMI: Cystic fibrosis, diagnosed at age 7. Her medical history includes multiple admissions for
pneumonia and exacerbation of her disease. She has been coughing more lately,
producing thick yellow sputum. She is taking pancreatic enzymes, vitamins, and lasix.
PE: P 88, BP 104/72, R 26, T 36.8°C. Diffuse wheezing and rhonchi are heard on
ausculation. Heart tones are diminished. Clubbing seen. Abdomen soft.

CXR: Diffuse fibrosis and areas of cystic dilation.

ECG: RBBB, RAD, RVH.

Labs: Hct 55, WBC 20.1, K+ 3.2 mEq/L, C02 29 mEq/L. PT/PTT lost in lab.

She refuses to be intubated by the method previously employed, i.e., an awake blind nasal
technique.
193 The Essential Oral Board Review

Respiratory VI (Session 2)

A 16 year old, 40 kg, 5*0" female with cystic fibrosis presents for removal of hardware from a
tibia/fibular fracture. She is taking pancreatic enzymes, vitamins, and lasix. She refuses to be
intubated by the method previously employed, i.e., an awake blind nasal technique. P 88, BP
104/72, R 26, T 36.8°C, Hct 55, K+ 3.2 mEq/L.
194 The Essential Oral Board Review

Respiratory VI (Questions)

A 16 year old, 40 kg, 5'0" female with cystic fibrosis presents for removal of hardware from a
tibia/fibular fracture. She is taking pancreatic enzymes, vitamins, and lasix. She refuses to be
intubated by the method previously employed, i.e., an awake blind nasal technique. P 88, BP
104/72, R 26, T 36.8°C, Hct 55, K+ 3.2 mEq/L

A. Preoperative Evaluation
1. Cystic fibrosis: What additional information do you need before surgery? What
additional labs? Is cystic fibrosis a type of COPD? What type?
2. Polycythemia: What is the significance of her Hct of 55?
3. Hypokalemia: Should she receive potassium supplementation before proceeding?
Why?
B. Intraoperative Management
1. Anesthetic technique: Is regional anesthesia a good idea? What is the effect of
regional anesthesia in a patient with cystic fibrosis? Will it make bronchospasm worse?
Can you think of any reasons if may be contraindicated in a patient with cystic fibrosis?
The PT/PTT are 18/45. Why?
2. Difficult airway: You are clearly able to visualize the uvula, but the thyromental distance
is only 1.5 cm. What if no fiberoptic bronchoscope were available? Would you use an
LMA for the case?
3. Monitoring: The procedure is planned for one hour. Do you need an arterial line? Does
the duration of the case determine the type of monitoring required?
4. Mucous plug: The heart rate increases to 155 during the procedure. What would you
do? The pulse oximeter reads 60%. What would you do? You are unable to pass a
suction catheter completely down the ETT. What could be the cause? Would you pull out
the ETT?
5. Transfusion therapy: 800 cc of blood loss occurs. How much blood would you give?
What is the optimal Hct for this patient?
C. Postoperative Care
1. Polycythemia: The patient does not seem to move her left arm. What would you do?
How would a Hct of 58 affect your management?
2. Pneumothorax: You are called to address the patient's respiratory distress. How would
you evaluate the situation? If a pneumothorax were suspected, how and where would you
place a chest tube?
195 The Essential Oral Board Review
196 The Essential Oral Board Review

Respiratory VII (Session 1)

A 45 year old, 81 kg, 4"11" female with presents for repair of left shoulder.

HPI: The pt injured her shoulder playing tennis. The injury occurred 5 months ago.

PMI: Systemic lupus erythematosis with renal, pulmonary, and neurologic involvement in the
past, but not now. She claims to have had several general anesthetics in the past
without difficulty. She also states she is allergic to local anesthetics from a dental visit 10
years ago.

PE: P 66, BP 160/90, R 24, Temp 36.8°C. Lungs - decreased breath sounds on the left.
Heart - regular rate, rhythm without murmur. Oral aperture and submandibular space -
both small.

CXR: Left-sided pleural effusion, unchanged from 3 months ago.

ECG: NSR.

Labs: Hct 26, and platelets electrolytes WNL.

The surgeon wants you to perform an interscalene block for postop pain when the pt goes home
after the surgery.
197 The Essential Oral Board Review

Respiratory VII (Session 2)


A 45 year old, 81 kg, 4"11" female with systemic lupus erythematosis presents for repair of left
shoulder. She has a chronic left-sided pleural effusion. Her oral aperture and submandibular
space are small, but she claims to have had several general anesthetics without difficulty. She
also states she is allergic to local anesthetics from a dental visit 10 years ago. P 66, BP 160/90,
R 24, Temp 36.8°C, Hct 26.
198 The Essential Oral Board Review

Respiratory VII (Questions)


A 45 year old, 81 kg, 4"11" female with systemic lupus erythematosis presents for repair of left
shoulder. She has a chronic left-sided pleural effusion. Her oral aperture and submandibular
space are small, but she claims to have had several general anesthetics without difficulty. She
also states she is allergic to local anesthetics from a dental visit 10 years ago. P 66, BP 160/90,
R 24, Temp 36.8°C, Hct 26.

A. Preoperative Assessment
1. SLE: What is it? How would you tailor your preoperative assessment to a patient with
SLE?
2. Airway: What do you think about the patient's airway? What are the elements of your
routine airway assessment? What is your definition of a difficult airway? What factors
predict a difficult airway?
3. Local anesthetic allergy: What is novocaine? What is the significance of being allergic
to it? The difference between ester and amide local anesthetics?
B. Intraoperative Management
1. SLE: How does the presence of SLE affect your intraoperative care?
2. Regional anesthesia: The patient refuses GA. The dentist claims the "allergy" was a
reaction to an intravascular injection of epinephrine. Would you use an interscalene or
supraclavicular block? Do you know how they are performed? What caveats should you
bear in mind as each is performed for this patient?
2. Respiratory distress: Following the block, the patient becomes progressively more
dyspneic. Why? Is an interscalene block contraindicated in patients with COPD? A
pleural effusion? What is the incidence of phrenic nerve palsey?
3. Difficult intubation: Direct laryngoscopy reveals soft tissue, no laryngeal structures, and
a small amount of greenish liquid. What would you do now? The pulse oximeter reads
50%. Do you thinks the value is real? Accurate? Describe the function of the pulse
oximeter.
4. Allergic reaction: The patient's BP is 80/40 and HR is 140. Your response? The dentist
calls back to tell you that the intravascular injection occurred in a different patient with the
same last name. Your patient's records could not be found. Could this be an anaphylactic
reaction? How can you clinically distinguish an anaphylactoid from an anaphylactic
reaction? How do the treatments differ?
C. Postoperative Care
1. ARDS: Postoperatively, the patient is receiving 25 cm H20 of CPAP. Why? What is
Mendelson's syndrome?
2. Endotracheal rube change: According to the respiratory therapist, the return volume is
120 cc despite a delivered volume of 600 cc. Significance? How would you interrogate
the ETT for a possible leak? How would you change the ETT?
200 The Essential Oral Board Review

Respiratory VIII (Session 1)

A 20 year old, 76 kg, 6'0" male is scheduled for broncheoalveolar lavage of the right lung.

HPI: Pt has alveolar proteinosis. He has been easily fatigued and short of breath for several
years. The pt has worked installing installation in buildings since high school. He taking
digoxin and furosemide.

PMI: None.

PE: P 98 BP 130/60, R 30, Temp 37.2°C. His airway exam is unremarkable. Auscultation of
his lungs reveals bilateral crackles. He has a systolic ejection murmur over the upper
sternal border and his second heart sound shows no respiratory variation. Clubbing is
seen on all extremities.

CXR: Diffuse infiltrate bilaterally.

ECG: Biventricular hypertrophy, occasional PVCs.

Labs: Hgb 17 g/dl. ABG pH 7.35, pC02 56, p02 55. Echocardiogram reveals biventricular
enlargement. Other labs and coagulation studies are normal.
- w

201 The Essential Oral Board Review

Respiratory VIII (Session 2)

A 20 year old, 76 kg, 6'0" male is scheduled for broncheoalveolar lavage of the right lung. He has
a history of alveolar proteinosis. He has been easily fatigued and short of breath for several
years. The pt has worked installing installation in buildings since high school. He taking digoxin
and furosemide. P 98 BP 130/60, R 30, Temp 37.2°C. Clubbing is seen on all extremities. ECG
shows biventricular hypertrophy, occasional PVCs. CXR shows diffuse bilateral infiltrates. Hgb
17 g/dl. ABG pH 7.35, pC02 56, p02 55.

<&
202 The Essential Oral Board Review

Respiratory VIII (Questions)

A 20 year old, 76 kg, 6*0" male is scheduled for broncheoalveolar lavage of the right lung. He has
a history of alveolar proteinosis. He has been easily fatigued and short of breath for several
years. The pt has worked installing installation in buildings since high school. He taking digoxin
and furosemide. P 98 BP 130/60, R 30, Temp 37.2°C. Clubbing is seen on all extremities. ECG
shows biventricular hypertrophy, occasional PVCs. CXR shows diffuse bilateral infiltrates. Hgb
17 g/dl. ABG pH 7.35, pCOz 56, p02 55.

A. Preoperative Assessment
1. Shunt: What is the shunt equation? How is it derived? Does it apply to both
intra pulmonary and intracardiac shunts?
2. Alveolar proteinosis: What is it? (lungs filled with granular proteinaceous surfactant
phospholipids/proteins/immunoglobulins; macrophages stuffed with lamellar bodies show
decreased killing; no inflammation; may be asso with silica and aluminum dust, cancer,
and chinch infection) Is it restrictive or obstructive? What's the difference?
3. PFTs: Are additional PFTs required? What FEV1 is necessary to allow the procedure to
be performed? Is application of FEV1 valid?
4. Polycythemia: Should you perform an exchange transfusion? Anesthetic significance?
B. Intraoperative Management
1. Monitor: How? Why? Where? Are they turning the pt lateral?
2. ETT: What type of ETT is necessary? Indications for OLV? What if the pt had a difficult
airway? Could a combitube be used? A Univent tube? A bronchial blocker?
3. Differential auscultation: Insert L-sided DLT, tracheal cuff up, BSEB, bronchial cuff up,
clamp tracheal lumen, hear BS on R. OK? Would this be more likely with a L- or R-sided
tumor? What if you couldn't make it go down the left no matter how you twist the tube?
Can you still perform lavage?
4. Differential auscultation: After repositioning, L-sided BS are heard with bronchial lumen
ventilation, but no BS are heard with tracheal lumen ventilation. Draw what could have
happened.
5. Maintenance: What would you use for maintenance? Compare a propofol TIVA with a
Desflurane volatile agent technique.
6. Hypoxia: During the first of 10 lavages, the saturation drops. Which is better, PEEP or
CPAP? Why? Can you even apply PEEP during lavage?
7. HTN: Which drug is better to treat HTN, SNP or NTG? The BP is 190/110 in this pt.
Which would you use?
C. Postoperative Care
1. Postop ventilation: If the pt had to be mechanically ventilated, would you use a DLT or
change to a SLT? How would you in-service the nurses about the DLT? Aren't you
concerned about spillage with a SLT?
2. ABG: pH 7.18. What must you do immediately? PaC02 80. Now what? What if a SLT,
bilateral rhonchi were heard? Copious amounts of pink frothy secretions are suctioned
from the ETT.
3. Ventilation: Which mode is best, IMV, SIMV, PS, AC, PRV?
4. ECMO: What if he had life-threatening desaturation, no matter what mode was used?
What is ECMO?
203 The Essential Oral Board Review

M$\
204 The Essential Oral Board Review

Respiratory IX (Session 1)

A 39 year old, 90 kg, 5'4" female is scheduled for laser debulking of laryngeal polyps.

HPI: Pt has a history of subglottic polyps all her adult life. She is a famous opera singer and
has become more hoarse, forcing her to cancel her world singing tour. By direct
laryngoscopy, the polyps extend 4 cm below the vocal cords. She has had multiple laser
surgeries in the past. According to her last medical record from 1 year ago, the
anesthesiologist was only able to pass a 5.0 ETT.
PMI: History of depression, tobacco use, deep venous thromboses, and a symptomatic hiatal
hernia. She has had a failed gastric stapling procedure. Her medications include
fluoxetine (Prozac), aspirin, Fen/Phen, and metaclopramide.

PE: P 80, BP 150/90, R 29, Temp 36.0°C. Her airway exam is remarkable for large incisors
and a receding mandible. Her lungs are clear to auscultation. Her heart tones are
normal. There is a loud P2.

CXR: None.

ECG: RBBB, RVH.

Labs: Hct 48, BUN 28, Cr 2.4. Other labs and coagulation studies are normal.

She had a glass of tea 2 hours ago with her medications.


JS\

205 The Essential Oral Board Review

Respiratory IX (Session 2)

A 39 year old, 90 kg, 5*4" female opera singer is scheduled for laser debulking of laryngeal
polyps. By direct laryngoscopy, the polyps extend 4 cm below the vocal cords. She has a history
of depression, tobacco use, deep venous thromboses, and a symptomatic hiatal hernia. Her
medications include fluoxetine (Prozac), aspirin, Fen/Phen, and metaclopramide. P 80, BP
150/90, R 29, Temp 36.0°C. She has a receding mandible. A loud P2 is heard with auscultation.
Hct 48.
206 The Essential Oral Board Review

Respiratory IX (Questions)

A 39 year old, 90 kg, 5'4" female opera singer is scheduled for laser debulking of laryngeal
polyps. By direct laryngoscopy, the polyps extend 4 cm below the vocal cords. She has a history
of depression, tobacco use, deep venous thromboses, and a symptomatic hiatal hernia. Her
medications include fluoxetine (Prozac), aspirin, Fen/Phen, and metaclopramide. P 80, BP
150/90, R 29, Temp 36.0°C. She has a receding mandible. A loud P2 is heard with auscultation.
Hct = 48.

A. Preoperative Assessment
1. Overall: Tell me everything you are thinking about this case - clearly, succinctly, as a
consultant in Anesthesiology.
2. Airway: Is any further evaluation necessary? (cricothyroid lig) How would a flow-volume
loop help you?
3. Fen/Phen: What is it? Would you cancel surgery? How long? Why?
4. Pulmonary HTN: What do you think about her cardiopulmonary status? Does she have
pulmonary HTN? Pickwickian syndrome? Why? Significance?
B. Intraoperative Management
1. Aspiration: Would you delay surgery? How long?
2. Jet ventilation: The surgeons sat that the ETT will be in their way? Is that a reasonable
thing to ask? What is jet ventilation? How does it work? Problems? Device?
3. Induction: Goals? How achieve them? Why not other technique? Say intubation were
difficult.
4. Maintenance: Goals? How achieve them? Why not other technique? Advantage to N2?
Because of its density or viscosity?
5- Airway fire: On propofol infusion with morphine and a 50:50 mixture of 02 and N20 via a
6.0 ETT covered with aluminum tape, an airway fire develops. What would you do?
6. Extubation: The sat is 100% on Fi02 100% but 90% on 50% 02. Would you extubate?
7. Bucking: The sat drops and PIP rises. What would you do? Wouldn't you expect the
PIP to be high with obesity?
C. Postoperative Care
1. PE: Postop day 2: The sat is 92%. Are you happy with that? What if the pt were still
intubated on Fi02 50%? Before doing anything else, would you put her on 100%? How
would you determine the cause? Could this be related to Pickwickian syndrome? How
would you diagnose a PE?
2. Mucous plug: Later, after heparin has been started, you are called back because the sat
is 90%. What would you do? The PIP is high. How would you change the ETT?
3. Blood plug: Surgeons want a nasotracheal tube. Very difficult, but once in with Magill
forceps, very high PIP.
207 The Essential Oral Board Review
208 The Essential Oral Board Review

Neurology I (Session 1)

A 30 year old, 77 kg, 5'8" male is scheduled for resection of a supratentorial tumor.

HPI: He has a one month history of headache, unsteady gait, and visual changes. CT scan of
the brain reveals a 4 x 5 cm left temporal mass, causing a 1 cm shift of the midline
towards the right and slit-like cerebral ventricles.

PMI: 5 year history of hypertension, poorly controlled with thiazide. He is allergic to sulfa-
containing drugs. His mother states she had difficulty waking up from anesthesia.

PE: P 52, BP 180/90, RR 12, Temp 39.3°C. He is currently difficult to arouse. The pt does
not open his mouth to command, but his neck and jaw seem normal. His lungs are clear
to auscultation. His heart exam is unremarkable.

CXR: No apparent disease.

ECG: NSR, left ventricular hypertrophy.

Labs: Hct 24, platelets 255K, PT 13.6, PTT 26.7, electrolytes normal.
209 The Essential Oral Board Review

Neurology I (Session 2)

A 30 year old, 77 kg, 5'8" male is scheduled for resection of a supratentorial tumor. He has a one
month history of headache, unsteady gait, and visual changes and a 5 year history of
hypertension. He is currently difficult to arouse. P 52, BP 180/90, RR 12, Temp 39.3°C, Hct 24.
210 The Essential Oral Board Review ^*

Neurology I (Questions)
A 30 year old, 77 kg, 5'8" male is scheduled for resection of a supratentorial tumor. He has a one ^
month history of headache, unsteady gait, and visual changes and a 5 year history of )*>
hypertension. He is currently difficult to arouse. P 52, BP 180/90, RR 12, Temp 39.3°C, Hct 24. ^
A. Preoperative Evaluation J>
1. Elevated ICP: Why do you think he is difficult to arouse? Do you think this patient has fj
elevated ICP? What would be the significance? What is the CPP? How can you ^
decrease the ICP? Does intracranial blood volume affect ICP? What is the intracranial W
compliance curve? Can you draw it? Is position on the curve shifted or is the curve itself fj
shifted? S>
2. Cerebral perfusion pressure (CPP): Which is more important, CPP or CBF? How are
the 2 related? How is CBF controlled? How is it related to CMR02? To MAP? What is
normal CBF?
3. Hypertension: How does the patient's HTN affect your management?
4. Fever: What is the significance of his fever? How would you evaluate it? Control it?
5. Anemia: Should he be transfused prior to surgery? Why? Jtt
B. Intraoperative Management
1. Monitors: What monitors would you use?
2. Induction: How would you induce anesthesia? What are your goals? Is succinylcholine
safe to use? How can you attenuate its affect on ICP?
3. Maintenance: What drugs would you use for maintenance? Can enflurane cause
seizures? Does sufentanil increase ICP? Why is isoflurane best? (CBF, ischemic
threshold, CSF) What is its effect on coupling and autoregulation?
4. Ventilation: What are your goals for ventilation? ForPaC02? What are steal and
reverse steal?
5. Hypotension: The BP is 120/60. What would you do? Why? Is hypotension better
tolerated in this patient? Would it be preferable not to administer fluids?
6. Fluids: What is the best fluid for this case? WhynotD5W? How is the BBB different
than peripheral endothelial barriers (8 vs. 65 A)? The surgeons tells you to "run the
patient dry." Do you agree?
7. Cerebral edema: The surgeons complain of a swollen brain an inability to close the
cranium. Can you help them? (pressure, metab, dilators, mass) Could this be due to
N20?
C. Postoperative Care
1. Delayed emergence: The patient does not move, breathe, or respond to stimulation.
Your response?
2. Hypertension: What is the best drug to use to treat a BP or 180/98? What are the
pros/cons of trimethephan?

%k
211 The Essential Oral Board Review
212 The Essential Oral Board Review

Neurology II (Session 1)

A 22 year old, 90 kg, 60" female is scheduled for an open cholecystectomy.

HPI: Pt has a one year history of intermittent right upper quadrant abdominal pain. The
presence of cholelithiasis was confirmed by ultrasound.
PMI: Myasthenia gravis for 1 year, beginning as diploplia and gradually progressing to total
body weakness. Exacerbations have included respiratory muscles. Prior thymectomy at
an outside hospital has been ineffective in controlling the disease. Pt also has a positive
family history of malignant hyperthermia, and insulin dependent diabetes mellitis.
Medications include pyridostigmine, prednisone, cyclophosphamide, and NPH 40U q AM,
20U q PM. Without her pyridostigmine, she usually becomes weak within a few hours.

PE: P 80, BP 150/70, R 28, Temp 36.7°C. It is possible to see the base but not all of her
uvula with her airway exam. The neck is supple and the thyromental distance is 7 cm.
The lungs and heart are normal to examination. She does not seem weak.

CXR: No apparent disease.

ECG: Normal sinus rhythm.

Labs: Hgb 12 gm/dl, electrolytes and coagulation studies within normal limits.

The pt reports for surgery at 645 AM, after following the instructions of the preop nurse to not day
any of her usual medications on the day of surgery.
)

^pwr 213 The Essential Oral Board Review

Neurology II (Session 2)

A 22 year old, 90 kg, 60" female is scheduled for an open cholecystectomy. She has myasthenia
/fpr
gravis, a positive family history of malignant hyperthermia, and insulin dependent diabetes
mellitis. Medications include pyridostigmine, prednisone, cyclophosphamide, and NPH 40U q
AM, 20U q PM, P 80, BP 150/70, R 28, Temp 36.7°C.

/ffef

(m

m
f>
214 The Essential Oral Board Review

Neurology II (Questions)

A 22 year old, 90 kg, 60" female is scheduled for an open cholecystectomy. She has myasthenia
gravis, a positive family history of malignant hyperthermia, and insulin dependent diabetes
mellitis. Medications include pyridostigmine, prednisone, cyclophosphamide, and NPH 40U q
AM, 20U q PM, P 80, BP 150/70, R 28, Temp 36.7°C.

A. Preoperative Evaluation
1. Myasthenia gravis: What is it? How can you determine its severity? Are there different
types? A student asks about the difference between MG and myasthenic syndrome. Do
symptoms improve with exercise? How does pyridostigmine work? What are the pros
and cons of discontinuing it preoperatively?
2. Malignant hyperthermia: Should she receive a muscle biopsy for MH preoperatively?
What type of test is performed on the muscle. How would you provide anesthesia for the
biopsy? The surgeon wants to use lidocaine for the biopsy. Would your let him?
3. Obesity: If the pt had appeared at your preop clinic 3 weeks before surgery, would you
have recommend a weight loss program prior to surgery? An exercise program?
4. IDDM: How would you handle her insulin regimen?
B. Intraoperative Management
1. Anesthetic choice: Would and epidural or series of interscostal blocks be a good idea
for this patient? How would you induce GA? Would you make any special preparations
with the machine?
2. Muscle relaxants: Do any of her medications affect the response to succinylcholine or
nondepolarizing relaxants? Should more succinylcholine be used? More
nondepolarizers?
3. Malignant hyperthermia: The HR increases to 110. What would you do? The ETC02
increases to 50. What would you do? PVCs appear on the ECG. Now what? The
temperature does not change? Does this make the diagnosis of MH suspect? What is the
earliest sign of MH? At what point would you give dantrolene? How does it work? Any
side effects?
4. Extubation: How do your criteria for extubation differ for the patient with myasthenia
gravis?
C. Postoperative Care
1. Respiratory function: Mostly likely postop complication? Could you have predicted
preoperatively whether postoperative intubation is necessary?
2. DKA: You notice in the hospital computer that her serum sodium is 150 mEq/L and ABG
is pH 7.22, PaC02 35, Pa02120. What would your do? When you call the nurse, she
says that the urine output has been good and the patient (still intubated) has been resting
quietly.
216 The Essential Oral Board Review

Neurology III (Session 1)

A 15 year old, 65 kg, 5*3" male presents for emergency head CT scan.

HPI: Pt was an unrestrained front seat passenger of a high speed motor vehicle accident. He
was found conscious but nonverbal on the scene. A 14 gauge IV was started in his right
arm. He was brought directly to the emergency room.

PMI: Deafness and right-sided hemiplegia after a case of childhood meningitis. The pt usually
walks with a walker and is otherwise mentally normal.

PE: P 52, BP 90/40, R 10, Hb 13, Temp 34.0°C. He is unconscious but spontaneously
ventilating. His right anterior chest moves inwards with inspiration, while his left moves
outwards. Decreased breath sounds are heard on over the right chest. The lungs are
clear to auscultation. Heart tones seem diminished. His Foley catheter is draining blood.

CXR: Multiple rib fractures on the left side, some ribs multiply fractured; right-sided pleural
effusion.

ECG: Low amplitude QRS signals.

Labs: Hct 30, all other labs are pending.


217 The Essential Oral Board Review

£ Neurology III (Session 2)


mm^

v A 15 year old, 65 kg, 5'3" male presents for emergency head CT scan following a motor vehicle
lp accident. He was unrestrained front seat passenger of a car traveling at approximately 60 mph.
^v He is deaf and has right-sided hemiplegia after a case of childhood meningitis. He is
fir unconscious but spontaneously ventilating. P 52, BP 90/40, R 10, Hb 13, Temp 34.0°C.

(fj%

$^
218 The Essential Oral Board Review

Neurology III (Questions)

A 15 year old, 65 kg, 5'3" male presents for emergency head CT scan following a motor vehicle
accident. He was unrestrained front seat passenger of a car traveling at approximately 60 mph.
He is deaf and has right-sided hemiplegia after a case of childhood meningitis. He is
unconscious but spontaneously ventilating. P 52, BP 90/40, R 10, Hb 13, Temp 34.0°C.

A. Preoperative Evaluation
1. Evaluation of elevated ICP: What would you do first when you walked into the
emergency room? Is it important to know his GCS? Do you think this patient has
increased ICP? Do you think the neurosurgeons should place an epidural bolt to confirm
your impressions? What is an epidural bolt? Why is his HR so low? Do you think he has
high spinal cord transection? How can you diagnose elevated ICP with spinal shock?
2. Treatment of elevated ICP: What are your goals? Which is worse, hypotension or
hypoxia? How can you increase CPP without causing further bleeding? Would you call
hyperventilation a standard of care? How does mannitol work? Is there a role for
steroids?
3. Airway mgmt: Does he need immediate intubation? Should you lower his BP first?
What are your concerns before intubation? Are prior C-spine films necessary? When are
they unnecessary? Describe your head positioning. His pulse oximeter reading drops to
75% and you cannot ventilate by mask. Succinylcholine?
B. Intraoperative Management
1. Flail chest: What is flail chest?
2. Induction: The surgeons require the pt to be still for a head and abdominal CT scan.
What are you goals for induction? How do you plan to achieve them?
3. Fluids: Following induction, the BP drops to 60/40. Only is D5W available. Would you
administer it? What are your concerns? Which is better for neurosurgery, NS or LR?
4. Hypothermia: The patient's temperature is 34°C. What is the most likely cause? Would
you try to warm him up? How? Why?
5. Maintenance: What agents would you use for maintenance? Why is isoflurane the best
inhalational agent?
6. Spinal shock: How is your anesthetic management different for a pt with spinal shock?
7. PIC: The surgeons order you to give FFP. Would you do what they say? They complain
of oozing at the surgical sites. Is this likely DIC? Define? Is hematuria consistent with the
diagnosis? Could it be primary fibrinolysis? Would you wait for a PT/PTT? What would a
thromboelastogram show? Are there any drugs you can give to treat DIC? EACA?
DDAVP? Heparin?
C. Postoperative Care
1. Pseudocholinesterase deficiency: Postoperatively, he is still flaccid. No additional
muscle relaxant had been given. DDx? (resp acid, abx, hypoCa2+, hypoMg2*, cold) Could
this be a phase 2 block? After changing to a new stimulator, there is fade. Can you safely
reverse it? With what drug? Two days later, he is reversed with neostigmine and
extubated. If respiratory distress developed, would you reintubate using succinylcholine?
Why/why not?
2. Neurologic deficit: The medical student notices that the patient is unable to move the R
side of his face. Would you order a neurologic consult?
219 The Essential Oral Board Review

mix
220 The Essential Oral Board Review

Neurology IV (Session 1)

A 22 year old, 90 kg, 69" male is scheduled for a transphenoidal hypophysectomy.

HPI: Pt has a history of headache, diplopia, and galactorrhea over the past several months.
The family has noticed that his hands, tongue, and jaw seem to have enlarged.
Endocrinologic workup revealed elevated prolactin levels. Head CT revealed a 10 mm
anterior pituitary mass, through to represent a prolactinoma. He has been treated with
bromocriptine and L-dopa, without improvement in symptoms. He became confused with
his first dose of bromocriptine.

PMI: HTN for 1 year, treated with propranolol. He reports increasing fatiguability over the past
6 months, but his echocardiogram was normal. He also claims his hands have been
tingling over the past month.
PE: P 60, BP 160/90, R 20, T 37.6°C. The pt's voice is hoarse. His jaw and tongue are
large, but his uvula can be easily seen. The thyromental distance is 10 cm. His lungs
are clear to auscultation. His heart sounds are normal. His hands are large.

CXR: Normal.

ECG: Left ventricular hypertrophy with nonspecific ST-T wave changes in the lateral leads.

Labs: Hct 40, glucose 180 mg/dl. Other electrolytes and thyroid hormone and aldosterone
levels are normal. His serum Cortisol level is low.

The surgery will be performed in the sitting position.


221 The Essential Oral Board Review

Neurology IV (Session 2)

A 22 year old, 90 kg, 69" male with a history of headache, diplopia, and galactorrhea is scheduled
for a transphenoidal hypophysectomy. He is taking bromocriptine, L-dopa, and propranolol. P
60, BP 160/90, R 20, T 37.6°C, Hct 40.
222 The Essential Oral Board Review

Neurology IV (Questions)
A 22 year old, 90 kg, 69" male with a history of headache, diplopia, and galactorrhea is scheduled
for a transphenoidal hypophysectomy. He is taking bromocriptine, L-dopa, and propranolol. P
60, BP 160/90, R 20, T 37.6°C, Hct 40.

A. Preoperative Evaluation
1. Prolactinoma: What is the function of the anterior pituitary? What physiologic functions
are likely to be affected by a pituitary tumor? What is the most common type of pituitary
lesion? Does the size of the tumor matter to you? Are large tumors more likely in males?
(10 mm)
2. Panhypopituitarism: What preoperative labs would you require? What type of hormone
replacement would be required if the entire pituitary gland were dysfunctional? (T4,
Cortisol, ADH, fludrocortisone) What is pituitary apoplexy?
3. Acromegaly: Why is he taking bromocriptine (dopamine agonist = GHIF) and L-dopa
(dopamine precursor)? (suppress GHRH) Why is he on propranolol? What type of tumor
do you think he has? What problems are associated with acromegaly? (CAD, HTN, CM,
glucose intolerance) Can bromocriptine cause any CNS side effects? (hallucinations from
LSD) Does he need a stress echocardiogram?
B. Intraoperative Management
1. Monitoring: What sort of evoked potentials should you consider using? What is the
effect of anesthetics on VEPs? Would you avoid inhalational agents?
2. Total Spinal: During topicalization of the nose, the patient looses consciousness. Your
response? Hypotension and bradycardia are detected. Your DDx?
3. Difficult intubation: He is difficult to intubate. What anatomical features would make this
likely? (thick tongue; hypertrophy of turbinates, tonsils, epiglottis, and larynx; glottic
stenosis; vocal cord paralysis due to RLN stretching, immobility of cricoarytenoid joint)
How would you have been prepared to deal with them?
4. Choice of anesthetics: Would you have used thiopental for induction? Which
anesthetics would you use now? Are muscle relaxants absolutely necessary? Do any
agents affect your monitoring?
5. Cocaine Absorption: You detect PVCs. Would you administer lidocaine? What else?
6. Bleeding: Sudden bleeding occurs despite packing and applied pressure to the
cavernous sinus. The surgeons request the sitting position. Problems?
7. Venous air embolism (VAE): Sudden hypotension develops with hypocapnea on the
capnogram. What would you do? How can you confirm the diagnosis? Why does the
sitting position predispose to VAE?
C. Postoperative Care
1. Diabetes insipidus: The patient develops polyuria, tachycardia, and lethargy. Cause?
(rule out hyperglycemia, mannitol) What is Dl? The surgeon thinks this is nephrogenic Dl.
How can you tell? The nurse suspects diabetes insipidus because of polyuria. Rx?
(DDAVP 0.0005 ml IN) Alternatives with CAD? (ADH 5-10U SQ q 4-6 hrs, clofibrate,
carbamazepine, chlopropamide) DDx of polyuria? (DM, mannitol).
2. Water deficit: The serum Na+is 160. What is the likely H20 deficit? [(100 kg)(0.6L/kg) -
(140 mEq/L)(100 kg)(0.6L/kg) / (160 mEq/L) = 60L - 52.5L = 7.5 L]
3. Hemiplegia: Anesthesia is blamed for hemiplegia. What would you do? Facial
anesthesia is detected with ophthalmoplegia. Significance?
4. Hypotension: How is your approach to hypotension occurring 4 days postop different in
this pt compared to a normal one? (adrenal insufficiency, CAD)
223 The Essential Oral Board Review

sr*
224 The Essentia] Oral Board Review JJJjjL

Neurology V (Session 1)
V

A 55 year old, 59 kg, 5'4" male is scheduled for closure of a open skull fracture.

HPI: Pt was found in an alley after an undetermined time following a blow to the head with a *i
lead pipe. He had been bleeding profusely from a scalp laceration. Brain parenchyma is \
protruding from an open skull fracture. He is unidentified. A cervical collar was placed
on the pt, an IV was started, and he was brought to the emergency room.

PMI: Levodopa and digoxin are found in his pocket.

PE: P 74, BP 90/40, R 24, T 33.0°C. His pulse oximeter reads 92%. His extremities are tied
down, he is wearing a C-collar, and he looks at you briefly when spoken to, but is
nonverbal. His extremities are warm to touch.
™%
CXR: Right upper lobe infiltrate.

ECG: Atrial fibrillation.

Labs: Hct 48, Na+151 mEq/L. Coagulation studies are pending.


M r 225 The Essential Oral Board Review

Neurology V (Session 2)

A 55 year old, 59 kg, 5'4" male is a victim of an assault with a lead pipe. He is bleeding profusely
from a scalp laceration and has an open skull fracture with protruding brain parenchyma. His
extremities are tied down, he is wearing a C-collar, and he looks at you briefly when spoken to,
but is nonverbal. Levodopa is found in his pockets. His extremities are warm to touch. P 74, BP
Mj 90/40, R 24, T 33.0°C, Hct 48. His ECG shows atrial fibrillation.

pPr

flSmf

(Wfy

/$m)
V

226 The Essential Oral Board Review jS

Neurology V (Questions) ^

A 55 year old, 59 kg, 5'4" male is a victim of an assault with a lead pipe. He is bleeding profusely ^
from a scalp laceration and has an open skull fracture with protruding brain parenchyma. His
extremities are tied down, he is wearing a C-collar, and he looks at you briefly when spoken to,
but is nonverbal. Levodopa is found in his pockets. His extremities are warm to touch. P 74, BP
90/40, R 24, T 33.0°C, Hct 48. His ECG shows atrial fibrillation.

A. Preoperative Evaluation
1. Parkinson's disease: What disease is suggested by his use of levodopa? What is the
pathogenesis of Parkinson's disease? What is levodopa? Side effects? (orthostasis,
dysrhythmias, hypertension) Why is he nonverbal?
2. Head Trauma: What is his GCS? (verbal 5, motor 6, eye 4) On what findings is it tk
based? Does he have increased ICP? Is he at risk for a subdural hematoma? Could this *
represent the "lucid interval" of an epidural hematoma? Is cerebral autoregulation ^
preserved in head trauma? Is C02 responsiveness? fc
3. Premedications: How would you provide aspiration prophylaxis? p
B. Intraoperative Management 8^
1. Spinal shock: Why is he hypotensive? Why is he hypothermic but warm to touch? How |e
would you evaluate? How would treat? Isoproterenol? Atropine? Is spinal shock i
possible with a HR of 74? Do catecholamines work in spinal shock? Would you 5^
administer steroids? * '
2. Monitors: How would you monitor this patient? **>
3. Induction: Is a lightwand intubation a good idea? Would you use a rapid sequence J*
induction with succinylcholine and ketamine (levodopa depletes myocardial NE stores)? ^
How would you blunt the hemodynamic response to intubation? Why? ^
4. Maintenance: Is etomidate a good choice for maintenance? How much fentanyl is JJ*
required for neurosurgery? */
5. Hypotension: The patient's MAP has been 50 throughout the case. Is this acceptable? *%,
What would you like it to be? How would you raise it? What are the pros and cons of
doing so?
6. Seizures: At the end of the case, the jugular venous blood gas shows a saturation of
50%. Your concerns? How would you diagnose intraoperative seizures?
C. Postoperative Care
1. Nausea and vomiting: How would you Rx N/V? Which is better, metoclopramide or
chlorpromazine? Which is better if he is also hypertensive?
2. Tremor: The nurse calls and says he is having a seizure. What would you do? He
seems to be trembling. Rx? Would diphenhydramine be appropriate?
227 The Essential Oral Board Review

a?*
f5

228 The Essential Oral Board Review

Neurology VI (Session 1)

A 63 year old, 67 kg, 5'8" male is scheduled for a right carotid endarterectomy.

HPI: Pt has had 3 transient ischemic attacks over the last month, all consisting of transient
monocular blindness. Carotid Doppler suggests 75% internal carotid stenosis on the
right and 60% on the left.
PMI: Pt has hypertension and angina. The hypertension is treated with labetolol. His angina
manifests as epigastric and sometimes substernal pain, occurs 1-2 times per week, and
is relieved by sublingual nitroglycerine. He is sedentary. A stress test was positive for
inducible ischemia with moderate exertion and a HR of 110. Cardiac catherization shows
a 80% lesion of his left anterior descending artery. The ejection fraction was normal.
CABG is planned following his carotid endarterectomy. He is also taking aspirin.

PE: P 65, BP 180/100, Temp 37.4°C. His airway, lung, and cardiac exams are normal.

CXR: Normal.

ECG: Q waves in leads II, III, and aVF with poor R wave progression.

Labs: Hct 45, bleeding time 15 minutes, electrolytes normal.


The surgeon does not usually place a prophylactic shunt.
229 The Essential Oral Board Review

Neurology VI (Session 2)

A 63 year old, 67 kg, 5'8" male is scheduled for a right carotid endarterectomy. He has
hypertension, angina, and has had 3 transient ischemic attacks over the last month. His
medications include labetolol, aspirin, and sublingual nitroglycerine. P 65, BP 180/100, Temp
37.4°C, Hct 45. EKG shows Q waves in leads II, III, and aVF with poor R wave progression.

■f^

<WS

«m\
230 The Essential Oral Board Review

Neurology VI (Questions)
A 63 year old, 67 kg, 5'8" male is scheduled for a right carotid endarterectomy. He has
hypertension, angina, and has had 3 transient ischemic attacks over the last month. His
medications include labetolol, aspirin, and sublingual nitroglycerine. P 65, BP 180/100, Temp
37.4°C, Hct 45. EKG shows Q waves in leads II, III, and aVF with poor R wave progression.
A. Preoperative Assessment
1. Evaluation: WhatisaTIA? A RIND? What are the indications for a CEA?
2. Hypertension: Would you recommend more aggressive treatment of his hypertension?
Would you recommend that the patient see a cardiology consult first?
3. Risks: What would you tell the family about his anesthetic/operative risks?
B. Intraoperative Management
1. Monitoring: What special monitors do you require? How are you going to monitor the
BP? Would you perform an Allen's test? Do you need a PAC? Where would you insert a
PAC? What if the 8 F introducer was inserted in the carotid? Do you need a TEE?
2. Anesthetic technique: RA vs. GA? The patient insists on RA. What blocks? For GA,
would you prefer a N20-narcotic technique or isoflurane?
3. Ventilation: How would you control ventilation during carotid cross-clamping?
4. Carotid cross-clamping: How are you going to assure the adequacy of cerebral blood
during carotid cross-clamping? What collateral circulation exists? The surgeon wishes to
use stump pressures. What would you do if during cross-clamping the EEG showed
bilateral beta waves? Delta waves? Can the SSEP be used to monitor cerebral function?
If a prophylactic shunt were placed, do you need to monitor with an EEG? Why not
perform bilateral carotid endarterectomies?
5. Myocardial ischemia: During carotid cross-clamping ST segment elevation occurs. What
would you do? BP 150/100, HR 70.
C. Postoperative Care
1. Reversal: The surgeon's ask you to give naloxone so they can assess his mental status.
Hazards? His glucose is 250. Could this explain his delayed arousal?
2. Hypertension: What complications do you expect postoperatively? In the recovery room,
the patient's BP is 220/110. Why is this a problem?
3. Myocardial ischemia: Do you need to rule-out an Ml?
0 231 The Essential Oral Board Review
232 The Essential Oral Board Review

Neurology VII (Session 1)

A 15 year old, 60 kg, 5'4" male is scheduled for emergency craniotomy.

HPI: Pt has a 6 month history of worsening ataxic gait. He lives in the countryside with his
uncle, who does not believe in medical care. He was finally brought to the emergency
room after failing to wake up this morning.

PMI: Mild cerebral palsey, grand-mal seizure disorder, which developed 1 year ago. His last
seizure was 6 months ago. He does not take any medications.

PE: P 48, BP 180/120, R 12, Temp 37.9°C. The pt is unresponsive to sternal rub. His pupils
are dilated. There is a 3/6 systolic ejection murmur over the left sternal border. His
lungs are clear to auscultation. His external airway anatomy is unremarkable.

CXR: Normal

ECG: None.

Labs: Hgb 8 g/dl, electrolytes normal. ABG pH 7.24, PaC02 55, Pa02 80.

The pt is scheduled for an emergency head CT scan prior to surgery.


233 The Essential Oral Board Review

Neurology VII (Session 2)

A 15 year old, 60 kg, 5*4" male is scheduled for emergency craniotomy. He has a 6 month history
of worsening ataxic gait. His past medical history includes mild cerebral palsey and a grand-mal
fw^f seizure disorder. P 48, BP 180/120, R 12, Temp 37.9°C. The pt is unresponsive to sternal rub.
There is a 3/6 systolic ejection murmur over the left sternal border. Hgb 8 g/dl, ABG pH 7.24,
PaC02 55, PaOz 80.

^dpf

/$Pi

/fflPv

^pf

fw^

/•?
234 The Essential Oral Board Review v.

Neurology VII (Questions)

A 15 year old, 60 kg, 5'4" male is scheduled for emergency craniotomy. He has a 6 month history ^
of worsening ataxic gait. His past medical history includes mild cerebral palsey and a grand-mal ^
seizure disorder. P 48, BP 180/120, R 12, Temp 37.9°C. The pt is unresponsive to sternal rub.
There is a 3/6 systolic ejection murmur over the left sternal border. Hgb 8 g/dl, ABG pH 7.24,
PaC02 55, Pa02 80. ^.

A. Preoperative Assessment ^
1. Cerebral palsey: His Apgars were 1 and 3. Do you think his CP was related to ^
inadequate neonatal resuscitation? ^
2. Seizures: Anesthetic implications? Should you start dilantin? Concerns? Interactions %t
with anesthetics? ^
3. ICP: What do you think about his BP? Would you lower it before induction? ^
4. Intubation: Would you intubate now or after the CT? Would you use an LMA? Jfe
Etomidate or succinylcholine? Why? Etomidate or thiopental? Why?
B. Intraoperative Management
1. Monitor: AL? Allen's test?
2. Maintenance: What is the best anesthetic? Would you use N20? ^
3. Sitting position: Implications? Special monitors? Which is more important for the S^
sitting position, AL or CVP? Special complications? ^
4. Arterial line: What factors determine the fidelity of the AL? If the waveform looks
dampened, what would you do? The NIBP is 40/20. What would you do?
5. Air embolism: How would you confirm the diagnosis? Would you use the ETC02? How
would you treat? PEEP? Would you give thiopental?
6. Swollen brain: The surgeons are unable to close the cranium. Can you help them out? ^
C. Postoperative Care *.
1. Delayed arousal: DDx? What would you do? *
2. ]CP: The BP is 110/50 after craniotomy. Is this OK? How do you know? Would you 5fc
want ICP monitoring to fi n d out? Which one? ^
3. Medicolegal: Do you think the uncle should be put in jail? ^

«s»

OTqj
236 The Essential Oral Board Review

Neurology VIII (Session 1)

A 66 year old, 90 kg, 5'8" male is scheduled for a right carotid endarterectomy.

HPI: Pt was scheduled for CABG when he developed episodes of right sided monocular
blindness, each lasting several hours occurring daily. Angiography revealed a 100%
occlusion of the left internal carotid artery and 70% occlusion of the right internal carotid
artery.

PMI: Status post Ml 5 months ago and IDDM. Cardiac catheterization reveals a 80%
occlusion of the LAD and 70% occlusion of the circumflex arteries, 35% LVEF, and
moderate MR. His medications include Procardia, aspirin, propranolol, sublingual
nitroglycerine, and NPH insulin 20 units q AM and q PM. He has also had occasional
episodes of syncope over the past 5 years, but has never seen a physician.

PE: P 64, BP 160/80, R 22, Temp 36.4°C. His airway exam is unremarkable. A bruit is heard
over his right neck.

CXR: Mild cardiomegaly.

ECG: NSR, type II 2nd degree AVB.

Labs: Hgb 12 g/dl, other labs and coagulation studies are normal.
Jlf/ 237 The Essential Oral Board Review

!W Neurology VIII (Session 2)


Sfsx

JP A 66 year old, 90 kg, 5'8" male with IDDM is scheduled for a right carotid endarterectomy for 70%
r) occlusion of the right internal carotid artery. The left internal carotid artery is completely
r^ occluded. The pt also had an Ml 5 months ago. He has 80% occlusion of the LAD and 70%
Jfir) occlusion of the circumflex arteries, 35% LVEF, and moderate MR. He has occasional episodes
jr A of syncope, which have never been addressed by a physician. His takes Procardia, aspirin,
!*£ propranolol, sublingual nitroglycerine, and NPH insulin. P 64, BP 160/80, R 22, Temp 36.4°C.
j t $ E C G s h o w s N S R , t y p e I I 2 n d d e g r e e AV B .

0WT

(*Sf
238 The Essential Oral Board Review

Neurology VIII (Questions)


A 66 year old, 90 kg, 5'8" male with IDDM is scheduled for a right carotid endarterectomy for 70%
occlusion of the right internal carotid artery. The left internal carotid artery is completely
occluded. The pt also had an Ml 5 months ago. He has 80% occlusion of the LAD and 70%
occlusion of the circumflex arteries, a 35% LVEF, and moderate MR. He has occasional
episodes of syncope, which have never been addressed by a physician. His takes Procardia,
aspirin, propranolol, sublingual nitroglycerine, and NPH insulin. P 64, BP 160/80, R 22, Temp
36.4°C. ECG shows NSR, type II 2nd degree AVB.

A. Preoperative Assessment
1. Perspective: Is this an emergency case? Urgent?
2. Indications for CEA: Is the finding of monocular blindness consistent with carotid
disease (if transient, amaurosis fugax)? How else do TIAs present? What if he
complained of bilateral blurring of vision? What if he had a visual deficit of his left visual
field in both eyes (homonymous hemianopsia). Other indications for CEA?
3. Neurologic workup: Does he need a neurologic consult before proceeding? Can't you
perform a neurologic exam yourself? What if he had subtle neurologic findings? Wouldn't
you want those documented before surgery?
4. Stokes Adams attacks: What is the significance of the syncope? What is the Stoke-
Adams-Morgagni syndrome? Indications for pacemaker? 1st degree AVB? 2nd? 3rd?
s/p Ml? Does he need one?
B. Intraoperative Management
1. Hemodynamic monitoring: Is you choice affected by whether you will use GA or RA?
Does a difficult AW make RA contraindicated? When, if ever, would a PAC be necessary
during a CEA? Surgeon wants a LIJ. Problem? What if hematoma developed? Would
you cancel the case? What next? You pierce the artery during subclavian approach.
Problems? How could you decrease chances of bleeding?
2. Choice of anesthesia: RA or GA? Which is better, isoflurane/neosynephrine or
nitrous/narcotic? Which is better for cerebral protection?
3. Neurologic monitoring: How monitor for cerebral ischemia? Does the decision of
whether and how to monitor cerebral function depend on whether a prophylactic shunt is
to be placed? Is such monitoring a standard of care?
4. Cerebral perfusion: Is this pt at greater risk for cerebral ischemia or thrombosis? If the
left carotid were occluded, how is his brain going to be perfused during carotid cross-
clamping? What if the vertebrals were occluded too?
5. Cerebral ischemia: Several minutes after carotid clamping, the EEG begins to show
more theta and delta waves. Plan A? Plan B? What if a shunt does not improve the
situation? Could this be related to the subclavian artery puncture?
C. Postoperative Care
1. BP control: What are your goals postop? How are you going to prevent cerebral
ischemia? Steroids? What is your BP target? The BP is 140/80. Given the likelihood of
cerebral emboli, is this enough?
2. Hypothermia: Temp 34°C. Is this good or bad? Why?
3. Cardiac: Do you think the CEA/CABG should have been combined? Prior to his CBP, do
you think he should receive a persantine thallium test? Why/why not? Which is better to
obtain, a DPT or a dobutamine echo? Why dobutamine and not dopamine or epinephrine
or isoproterenol?
240 The Essential Oral Board Review

Ophthalmology, Orthopedics, Machines I (Session 1)

A 72 year old, 64 kg, 5'4" female presents for a right total hip replacement.

HPI: Pt was found at home on the floor by her daycare nurse, after falling earlier in the day.
She was crying and more disoriented than usual. Hip films reveal a fracture of the right
femoral neck. She has been admitted for 1 day in the hospital.

PMI: She has glaucoma, reflux esophagitis, and rheumatoid arthritis. Her medications include
metoclopramide, ibuprofen, prednisone, echothiophate and timolol eye drops. She had a
surgery for a detached retina last week, the details of which are unavailable.
PE: P 45, BP 110/70, R 25, T 36.0°C. The pt has limited jaw opening ability. Her neck is
fixed in a flexed position, such that her chin approximates her chest. There is marked
kyphosis of her cervical spine. Breath sounds are diminished bilaterally. Heart tones are
normal. Her hands show a "swan neck" deformity and her fingers lack sensation.

CXR: Pleural thickening and scattered bilateral fibrosis with nodules.

ECG: Sinus bradycardia, first-degree block, RVH.

Labs: Hct 28, platelets 89 K+, WBC 4.5, electrolytes within normal limits.
/TV

241 The Essential Oral Board Review

/#*r

Ophthalmology, Orthopedics, Machines I (Session 2)

JM A 72 year old, 64 kg, 5'4" female presents for a right total hip replacement. She has glaucoma,
jf) reflux esophagitis, and rheumatoid arthritis. Last week, she had surgery for a detached retina.
(*\ Her medications include metaclopramide, ibuprofen, prednisone, echothiaphate and timolol eye
Jp drops, She had a surgery for a detached retina last week. P 45, BP 110/70, R 25. T 36.0°C, Hct
28.

*3

/"ftf
242 The Essential Oral Board Review

Ophthalmology, Orthopedics, Machines I (Questions)


A 72 year old, 64 kg, 5'4" female presents for a right total hip replacement. She has glaucoma,
reflux esophagitis, and rheumatoid arthritis. Last week, she had surgery for a detached retina.
Her medications include metaclopramide, ibuprofen, prednisone, echothiaphate and timolol eye
drops, She had a surgery for a detached retina last week. P 45, BP 110/70, R 25, T 36.0°C, Hct
28.

A. Preoperative Evaluation
1. Laboratory assessment: What are your minimum preoperative labs? Why is the Hct so
low? Would red cell indices or examination of a smear help?
2. Glaucoma: What is it? Does it affect your ability to premedicate with anticholinergics?
3. Bradycardia: What is the significance of her HR?
4. Arthritis: What is the significance of her arthritis? (Pericardial thickening or effusion,
pleural effusion, pulmonary fibrosis, carpal tunnel, anemia) Do you need airway films?
5. Machine: you detect a possible hairline crack of the 02 flowmeter. Significance?
Action? Why is scavenging of gases important?
B. Intraoperative Management
1. Monitors: The surgeons say there could be significant bleeding. What are the pros/cons
of inserting an AL (small calcific radial arteries inaccessible due to flexion deformities).
Right or left?
2. Anesthetic technigue: Would a spinal anesthetic be a good choice? What if the pt was
not competent to give consent? What level of blockade do you require? (T8) What local
anesthetic would you use? Is a coaxial technique contraindicated in rheumatoid arthritis?
Would need to obtain a bleeding time first?
3. Sulfur hexafluoride (SF6): Would you use N20? What are SF6 and C3F8? What is its
solubility in blood? Precautions? For how long?
4. Intubation: The epidural is unsuccessful. Describe your approach to the airway in
terms of risks/benefits. Does her arthritis make it more difficult? Does neck positioning
increase the chances of injury? Significance of a hoarse voice?(atlanto-axial subluxation,
flexion may cause odontoid protrusion through foramen magnum into brainstem, synovitis
of the TMD, cricoarytenoid arthritis)
5. Positioning: Concerns with positioning? Purpose of an axillary roll?
6. Vaporizer misfill: During maintenance with morphine, curare, N20, and enflurane, the
BP gradually declines to 80/40. DDx? You notice the mass spectrometer reads isoflurane
instead of enflurane. Could this explain the hypotension?
C. Postoperative Care
1. Paralysis: The patient is unable to move her right lower extremity postoperatively. What
would you do? Could this be related to your intubation? What would you do?
2. Ulnar nerve injury: She complains of a weak grasp in her left hand. What would you
do? Would you perform a Doppler exam? How would you assess ulnar nerve weakness?
EMG? Nerve conduction velocities? Could it have been prevented?
3. Pulmonary embolus: On postop day 1, you notice that the patient looks dusky. Why is
she at risk for hypoxemia? How would you categorize the different causes of hypoxia?
Why would a dead space lesion like a PE cause hypoxemia?
244 The Essential Oral Board Review

Ophthalmology, Orthopedics, Machines II (Session 1)

A 17 year old, 88 kg, 70" football player is scheduled for surgery for a dislocated right shoulder
and foreign body in his right eye.

HPI: The pt sustained both injuries after being tackled during a football game. There was no
loss of consciousness. He complains of pain in the back of his neck after the injury.

PMI: Pt is healthy, except for a reading of high blood pressure from his last physical exam.

PE: P 120, BP 80/60, R 12, T 35.5°C. All oropharyngeal structures are well visualized.
Breath sounds are equal. There is a continuous flow murmur over his chest wall and a
3/6 systolic ejection murmur over his left sternal border.

CXR: Left rib fractures, a small left-sided pneumothorax, rib notching and an indentation in the
upper descending aorta.
ECG: NSR, LVH.

Labs: Hct 42, electrolytes normal.


245 The Essential Oral Board Review

0m Ophthalmology, Orthopedics, Machines II (Session 2)

A 17 year old, 88 kg, 70" football player has a dislocated right shoulder and foreign body in his
right eye after being tackled. He is scheduled for surgery for both. He complains of neck pain. P
120, BP 80/60, R 12, T 35.5°C, Hct 30, EKG shows LVH, CXR reveals left rib fractures, a small a
small left-sided pneumothorax, rib notching, and an indentation in the upper descending aorta.

/ifp;

.<*fftf
246 The Essential Oral Board Review

Ophthalmology, Orthopedics, Machines II (Questions)

A 17 year old, 88 kg, 70" football player has a dislocated right shoulder and foreign body in his
right eye after being tackled. He is scheduled for surgery for both. He complains of neck pain. P
120, BP 80/60, R 12, T 35.5°C, Hct 30, EKG shows LVH, CXR reveals left rib fractures, a small a
small left-sided pneumothorax, rib notching, and an indentation in the upper descending aorta.

A. Preoperative Evaluation
1. ACLS: In the ER, his pulse oximeter reads 77%. He is in a C-collar and his helmet is still
on. What are your priorities? How would you intubate? The fiberoptic is broken.
2. Coarctation of the aorta: He is found to have a left-sided pneumothorax and his
saturation improves following insertion of a chest tube. What is the significance of his
murmur and EKG and CXR findings? Would you order a cardiology consult? What would
you ask? What type of lesion do you think he has, pre- or post-ductal?
3. Negative pressure check valve? When you check your machine before the operation,
would you perform a negative or positive pressure leak test? Does it depend on the type
of machine? How do you check the function of your scavenging system? For leaks in the
high pressure system?
B. Intraoperative Management
1. Choice of anesthetics: What are your priorities in deciding what anesthetic to give?
What is your plan?
2. Coarctation of the aorta: What is it? Assuming the pt has coarctation of the aorta, what
is the best anesthetic to give? Would a supraclavicular block be a good idea?
3. Ventricular septal defect: What if he had a VSD? Would you remove all bubbles from
the IV? Why?
4. Open globe: What is the best anesthetic for an open globe? Would you administer
succinylcholine?
5. Cervical injury: How does the neck pain fit into your plan? How would you evaluate its
significance?
6. Monitoring/access: How would you monitor BP? Which is more important-a central line
or an AL? Site for an AL? A central line? Would you use mass spectroscopy? How does
it work? Drawbacks? Can you measure concentration of gases in other way? Does it
matter where the gas is sampled?
7. Right mainstem intubation: You notice the PIP has increased from 25 to 40. What do
you think happened? What would you do? Breath sounds are diminished on the left?
Could this be a right mainstem intubation? Shouldn't it have been detectable on the
capnogram? Where would you place a chest tube?
8. Endotracheal insertion of the NG tube: The nurse anesthetist relieves you for a break.
Five minutes later, you are called stat because of inability to ventilate. What would you
do? The bellows seems to fall but then fails to rise. The bag collapses after a few
breaths. The ambu seems to collapse too easily. The saturation is 91%.
C. Postoperative Care
1. Reversal: At the end of the case, there is no twitch via the nerve stimulator. 10 mg of
pancuronium has been given for a 2 hour case. Would reverse anyway? Should reversal
always be given after use of nondepolarizing agents? Can the stimulator be used to
determine readiness for extubation instead of a 5 second head lift? 15 minutes after 5 mg
of neostigmine, there is still only 3/4 twitches by TOF. Would you give more?
2. Corneal abrasion: The pt's left eye is painful, injected, has vascular engorgement. His
left eye vision is blurred. What would you do?
3. Pneumothorax: On postop day 2, the pt's RR is 34. Your response?
248 The Essential Oral Board Review

Orthopedics, Ophthalmology, Machines III (Session 1)

A 14 year old, 43 kg, 4*11" female is scheduled for Herrington Rod insertion.

HPI: Pt has severe scoliosis, with a 60 degree Cobb angle. Her exercise ability is limited.

PMI: Asthma and von Willebrands disease. Her last admission for asthma was 1 month ago,
during which she was intubated and given high dose steroids. An emergency
tracheostomy was once performed in the doctor's office for severe bronchospasm. She
is currently taking prednisone and ventolin aerosols.

PE: P 70, BP 130/70, RR 32, T 36.8°C. Her lungs are clear to auscultation but are decreased
bilaterally. A healed tracheostomy scar is seen. The heart tones are regular.
CXR: Severe scoliosis.

ECG: NSR, RVH.

Labs: Hgb 17 g/dl. Electrolytes and PT/PTT normal. ABG pH 7.40, pCOz 40, pOz 79. PFTs
show a decrease in the FEV1 and FVC to 55% normal and a FEV1/FVC ratio of 40%.

The pt is anxious after hearing about the wake-up test from the surgeon.
249 The Essential Oral Board Review

Orthopedics, Ophthalmology, Machines III (Session 2)

flm A 14 year old, 43 kg, 4'11" female is scheduled for Herrington Rod insertion. She has severe
scoliosis, asthma, and von Willebrands disease. Her current medications include prednisone and
ventolin aerosols. P 70, BP 130/70, RR 32, T 36.8°C, Hct 28.

r*

fsl\
250 The Essential Oral Board Review

Orthopedics, Ophthalmology, Machines III (Questions)

A 14 year old, 43 kg, 4'11" female is scheduled for Herrington Rod insertion. She has severe
scoliosis, asthma, and von Willebrands disease. Her current medications include prednisone and
ventolin aerosols. P 70, BP 130/70, RR 32, T 36.8°C, Hct 28.

A. Preoperative Assessment
1. Scoliosis: What is the Cobb angle? How is severity of scoliosis determined? Do you
need a consultant to help you decide? What difference does the severity of the scoliosis
make to you? What type of respiratory disease does it represent?
2. Preoperative evaluation: Additional history? Labs? PFTs? ABG? EKG? Why is she
anemic? Would you order a serum pregnancy test?
3. Transfusion: The parents wish to avoid a transfusion. What would you tell them about
the risk of hepatitis and AIDS? The family asks your advice about directed donation.
Would you administer DDAVP?
B. Intraoperative Management
1. Monitoring: How would you monitor this patient? How would you explain a wake-up test
to the pt? What if she refused? Would one be necessary if SSEP monitoring were used?
Pitfalls of SSEP monitoring?
2. Induction: How would you induce anesthesia? What if the patient were allergic to
thiopental and eggs? Would you use etomidate?
3. Maintenance: What anesthetic technique would you use for a wake up test vs. for
SSEP? Are inhalational agents useful for status asthmaticus?
4. Vaporizers: Would you use desflurane? What is its vapor pressure? How does a
desflurane vaporizer work? How does it differ from an isoflurane vaporizer? Why is added
heat necessary?
5. Isovolemic hemodilution: How to? {Vole to be removed = (Hct - Hct/) / [(Hctj - HcV)/2]}
Contraindications?
6. Hypertension: BPgoal? The BP is 160/80. 200/100. When would you add a
vasodilator?
7. Hypoxemia: Nitroprusside is begun. 1 hour later, you notice the HR is 120. DDx? The
pulse oximeter reads 85%. What now? More likely related to hypoxic pulmonary
vasoconstriction or hypotension?
8. Loss of central 02 supply: The 02 pressure supply alarm goes off. What would you do?
C. Postoperative Care
1. Hypotension: BP 80/40. Rx? Steroids? Which? (hydrocortisone and fludrocortisone)
Why? Cushing's syndrome or an Addisonian crisis? (N/V, abd pain, lethargy, hypoNa,
hyperK, hyperCI, met acidosis, hypoglycemia, hypotension) What is the most common
cause? Related to your use of etomidate?
2. Blindness: Is induced hypotension an acceptable approach to reduce blood loss for this
case? Isovolemic hemodilution? The patient is blind in both eyes. Are you going to
explain this as a complication of your anesthetic to the parents?
252 The Essential Oral Board Review

Orthopedics, Ophthalmology, Machines IV (Session 1)

A 33 year old, 160 kg, 5'2" female presents for gastric stapling, reduction mammoplasty, and
liposuction.

HPI: Pt has been obese all her life. She has a history of bullemia and depression related to
her obesity.

PMI: NIDDM reported controlled by diet and oral hypoglycemic medication, DVTs 2 years ago,
and once for an episode of shortness of breath. V/Q scan during that hospitalization was
equivocal. 2 months ago, she was admitted to the hospital for attempted suicide. Her
medications include buproprion, valium, and tolazamide.

PE: P82.BP 150/70, R 28, Temp 36.3°C. Her airway exam is unremarkable. Breath sounds
and heart tones are distant.

CXR: Underpenetrated due to poor inspiratory effort.

ECG: NSR, low voltage QRS.

Labs: Hct 42, Na+140 mEq/L, K+ 3.9 mEq/L, glucose 49 mg/dl.

Your case is the first one scheduled for a brand new outpatient surgicenter, which allows 23 hour
overnight observations. The anesthesiology machines have just been installed yesterday.
r .

0^* 253 The Essential Oral Board Review

mr Orthopedics, O p h t h a l m o l o g y, Machines IV (Session 2)

(JBr A 33 year old, 160 kg, 5'2" female presents for gastric stapling, reduction mammoplasty, and
jc) liposuction. She has NIDDM and a history of DVTs. Her medications include buproprion, Valium,
rK and tolazamide. P 82, BP 150/70, R 28, Temp 36.3°C. Her airway exam is unremarkable.
^ Breath sounds and heart tones are distant. ECG shows NSR with low voltage QRS. Hct 42,
»_) glucose 49 mg/dl. Yours is the first case in a brand new outpatient surgicenter.

f$s

f#\

/•Pf

0!\

fWi
254 The Essential Oral Board Review

Orthopedics, Ophthalmology, Machines IV (Questions)

A 33 year old, 160 kg, 5'2" female presents for gastric stapling, reduction mammoplasty, and
liposuction. She has NIDDM and a history of DVTs. Her medications include buproprion, Valium,
and tolazamide. P 82, BP 150/70, R 28, Temp 36.3°C. Her airway exam is unremarkable.
Breath sounds and heart tones are distant. ECG shows NSR with low voltage QRS. Hct 42,
glucose 49 mg/dl. Yours is the first case in a brand new outpatient surgicenter.

A. Preoperative Assessment
1. Obesity: How does this case differ from an ASA I hernia repair? What is the difference
between obesity and morbid obesity?
2. Machine check: How do you usually perform one? How do you check the flowmeters?
The vaporizers? 02 tank? (It's reading 2000 psi now) Disc valves of the breathing
circuit? Scavenging system? Draw the machine. Draw the circle system. Draw the
Mapleson D. The 02 analyzer reads 79%. Is this OK? Is it necessary for the case?
Why?
3. Cracked flowmeters: During your check, the IV pole is knocked into the N20 flowmeter,
which may have a hairline crack. Is it safe to continue the case? How can you tell?
Illustrate your answer with a diagram.
B. Intraoperative Management
1. Preoxygenation: Because of this pt's obesity, how long is required for preoxygenation?
Does this have anything to do with closing capacity? Does this have anything to do with
her metabolic rate?
2. Induction: The Fi02 reads 100% running at 10 LPM and her pulse oximeter reads 98%.
How would you induce anesthesia? What are the pros/cons of a RSI vs an awake
fiberoptic intubation?
3. Difficult airway: Can't intubate/can't ventilate/pt bucking. What would you do?
4. Loss of central Q2 supply: The 02 supply failure alarm goes off. What is your
response? When you turn on the tank, it reads 200 psi. How long do you have? Is
closed circuit anesthesia indicated? Does this have anything to do with her metabolic
rate?
5. Loss of cylinder O2 supply: The 02 tank runs out. Now what? The sat on air is only
85%.
6. Crack in O2 flowmeter: A new machine is brought into the room, but the 02 bobbin is
stuck. What would you do? The 02 flowmeter shatters. What would you do? Can't you
just turn on the air?
C. Postoperative Care
1. New machines: After this experience, the administrator has appointed you head of a
committee to buy new anesthesia machines. What are some of your priorities in
selecting a machine? Only 1 vaporizer will be available in these machines. Which will it
be?
2. DKA: Later that day, you receive a call from the nurse that the pt's R were 28 at noon, 32
at 4 PM, and 40 at 8 PM. Additional morphine was given at 8 PM. What is your
reaction? The saturation is 91 % and the pt is lethargic. What could be going on? Do
you think this is a PE? The nurse asks whether she should give heparin, naloxone, or
something else?
0
255 The Essential Oral Board Review

£>
256 The Essential Oral Board Review V*,

Pediatrics I (Session 1) ^

A 7 year old 32 kg boy is scheduled for emergency cauterization of bleeding tonsils. )*•

HPI: Pt had tonsillectomy for recurrent tonsillitis 1 week ago. Today, his mother found him ^
with at home with the baby sitter with an empty bag of potato chips. He was sleeping at V*
the time and was difficult to arouse. He was admitted to the hospital floor, where an IV ^
was started. He has received D51/2 NS at a rate of 75 cc/hr for 12 hours. His urine ^
output is unmeasured. Jw

PMI: Unremarkable. He has been normal until today. ( ^ T ^

PE: P 112, BP 82/54, HR 112, R 16, Temp 36.5°C. The pt is lethargic, but responds to deep
pain. The lungs are clear to ausculation. A 2/6 systolic ejection murmur is heart over the
precordium. His mucous membranes are dry. He has slightly decreased skin turgor. His
hands and feet are cool. Capillary refill time is 3 seconds.

CXR: None.

ECG: None.

Labs: Na+ 128, Hct 35, PT/PTT pending.

/^
/p^

257 The Essential Oral Board Review

Pediatrics I (Session 2)

fmk< A 7 year old 32 kg boy who had tonsillectomy 1 week ago presents for cauterization of tonsillar
) bleeding. His mother found him with an empty bag of potato chips. On the hospital floor, he has
received D51/2 NS for 12 hours, and is now lethargic. A 2/6 systolic ejection murmur is heart over
0 the precordium. P 112, BP 82/54, HR 112, R 16, Temp 36.5°C, Na+128, Hct 35.

0%

0%

/fm
258 The Essential Oral Board Review

Pediatrics I (Questions)

A 7 year old 32 kg boy who had tonsillectomy 1 week ago presents for cauterization of tonsillar
bleeding. His mother found him with an empty bag of potato chips. On the hospital floor, he has
received D51/2 NS for 12 hours, and is now lethargic. A 2/6 systolic ejection murmur is heart over
the precordium. P 112, BP 82/54, HR 112, R 16, T 36.5°C, Na+ 128, Hct 35.

A. Preoperative Evaluation
1. Mental status: You have 1 hour before the surgeon arrives. What is your plan of action,
in order of decreasing priority? He does not respond to deep pain. Why is he lethargic?
Does his serum Na explain his lethargy?
2. Volume status: Do you think he is hypovolemic? Is the Hct consistent with
hypovolemia? How would you estimate his blood loss? Would you give him a fluid
challenge? With what fluid? Does his serum Na concern you? Would you administer a
bolus of 3% NS? Is he in shock? Define shock.
3. Coagulation: Do you require a PT/PTT and bleeding time? Why?
4. Full stomach: Does he have a full stomach? Should you pass an NG to empty it?
Should you administer bicitra?
B. Intraoperative Management
1. Induction: Because of the FRC, do you need to preoxygenate longer in children? How
would you induce anesthesia? Is an inhalational induction indicated? Which agent would
you use? Would you use ketamine? Is succinylcholine contraindicated in children? Why?
Limitation in jaw mobility is noted 1 minute after succinylcholine administration. What
would you do? What could be the cause? The saturation drops to 92%. Your response?
2. Airway: After the limitation disappears, you are unable to see the larynx due to blood.
Your response? Your anesthesia technician brings in the fiberoptic bronchoscope. Would
you prefer this or a blind nasal? What sized ETT would you use?
3. Eguipment: Would you use a circle or a Bain system? What are the advantages and
disadvantages of both?
4. Extubation: At the end of the case, the patient is spontaneously breathing, and has 4
twitches on train-of-four stimulation. The surgeons request that he not cough with
extubation. Is there anyway you can achieve this? What is a deep extubation? Is he
ready for extubation? He still does not open his eyes. What would you do? You gave only
NS during the case. Is a repeat serum Na worth ordering? A serum glucose? Could this
be related to his murmur?
C. Postoperative Care
1. Seizures: The patient develops grand mal seizures, and the ventilator high pressure
alarm goes off. What would you do? DDx?
2. Hypothermia: The pts temperature is 34.5°C. Why did the temperature decrease under
GA? What is the most efficient way to provide heat? The nurse suggests using both a
radiant warmer and a forced air warmer. Is this appropriate? Is an ear temperature
accurate?
3. Nausea: Following extubation the next day, the patient develops nausea and vomiting.
Recommended treatment?
260 The Essential Oral Board Review

Pediatrics II (Session 1)

A 32 week, 1500 g premature infant is scheduled for an emergency laparotomy.

HPI: Pt was kept NPO for the first 2 weeks of life, and was recently given OG feedings of half-
strength breast milk. The feedings were intermittently held for episodes of abdominal
dissension. Today, he passed frankly bloody stool, developed apnea and bradycardia,
and abdominal dissension. Abdominal XR revealed air in the bowel wall and free air
under the diaphram. An NG tube was placed with suction. Antibiotics were begun.
Emergency surgery is scheduled.

PMI: Former 28 week gestational age infant. He was intubated at birth, given surfactant,
mechanically ventilated, then extubated on day of life 3. He has a grade II
intraventricular hemorrhage, patent ductus arteriosus, and is receiving total parental
nutrition with 20% dextrose through a long arm central line. An episode of congestive
heart failure was treated last week with lasix and fluid restriction. Medications include
indomethacin, gentamycin, clindamycin, ampicillin.

PE: P 150, BP 30/15, R 40, Temp 34.9°C. The pt is lethargic and breathing with retractions.
Breath sounds are equal bilaterally. A machinery-type murmur is heard at the apex. His
capillary refill is 4 seconds.

CXR: Mild "ground glass" appearance.

ECG: Sinus tachycardia, right axis deviation.

Labs: Na+ 154 mEq/L, K+ 6.0 mEq/L, C0218 mEq/L, CI" 119 mEq/L, Hct 35, platelets 79 K,
PT/PTT 1.5 times control.
261 The Essential Oral Board Review

Pediatrics II (Session 2)

A 32 week, 1500 g premature infant with free air under the diaphram requires an emergency
laparotomy. He has a grade II intraventricular hemorrhage, patent ductus arteriosus, and is
receiving total parental nutrition with 20% dextrose through a long arm central line. Medications
include indomethacin, gentamycin, clindamycin, ampicillin. P 150, BP 30/15, R 40, Temp 34.9°C,
K+ 6.0, Hct 48, CXR shows a "ground glass" appearance.
262 The Essential Oral Board Review

Pediatrics II (Questions)

A 32 week, 1500 g premature infant with free air under the diaphram requires an emergency
laparotomy. He has a grade II intraventricular hemorrhage, patent ductus arteriosus, and is
receiving total parental nutrition with 20% dextrose through a long arm central line. Medications
include indomethacin, gentamycin, clindamycin, ampicillin. P 150, BP 30/15, R 40, Temp 34.9°C,
K+ 6.0, Hct 48, CXR shows a "ground glass" appearance.

A. Preoperative Evaluation
1. NEC: What are your priorities in management? What is NEC? Why is it associated with
hypotension? How should the hypotension be treated?
2. Prematurity: What concerns do you have regarding the patient's prematurity (AB: RDS,
A/B; C: CHD, PDA; D: ICH; FEN: dehydrate; Gl: NEC; H: anemia, coag; ID: sepsis;
Joints, skin and temp; K: vitamin, renal; Lines, Meds)
3. Coagulation: Are coagulation studies required? Why do you think he is on
indomethacin?
4. Airway: Should the patient be intubated before transport to the OR? Should the
intubation be performed awake or with a rapid sequence induction? Why are infants
predisposed to hypoxemia? Do you want an air leak? Why/why not?
B. Intraoperative Management
1. Monitors: How would you monitor this patient? Is a NIBP accurate in patients this size?
Where would you place an AL? Which is more important? Where would you place a
central line?
2. Maintenance: What drugs would you use for maintenance? Are neonates more sensitive
to anesthetics? What is the MAC for a neonate?
3. Fluids: What rate of maintenance fluid would you give? What dextrose concentration? Is
it better to keep this patient relatively underhydrated or overhydrated?
4. Pneumothorax: After closing the abdomen, the peak airway pressures are high. Why?
After 5 minutes, the BP and saturation decrease. DDx? Mgmt?
5. PDA: After a fluid challenge, the pulse oximeter in his foot reads 77% but that in his R
arm reads 98%. Why? Does the presence of fetal hemoglobin affect the accuracy of
pulse oximeter readings? What is the harm in opening the PDA? Is this patient at
increased risk for a paradoxical air embolus? Does the presence of a PDA increase that
risk?
C. Postoperative Care
1. Acidosis/retinopathy of prematurity: The student shows you his ABG pH 7.11, PaC02
45, Pa02 210 on Fi02100%, PIP 40, PEEP 2, IMV 30. She asks you for your
recommendations based on the ABG. In this critically ill patient, is the Pa02 good or bad?
How could it affect vision?
2. Transfusion: The Hct is 35. Does this provide optimal 02 delivery? How much blood
would you administer? Does it have to be CMV negative? During the transfusion, the QT
interval increases. Significance?
3. Hypothermia: The nurses say they want to warm the child with radiant warmers, the
resident says a surface air warmer is more efficient. Who is right? What are the
deleterious effects of hypothermia, (vasoconstrict, deer C02 on vent, relaxants, delayed
arousal, dysrhythmias at 32, deer pit TxA2 at 32, not shiver) If infants do not shiver, how
do they stay warm?
264 The Essential Oral Board Review

Pediatrics III (Session 1)

A 4 year old, 15 kg male presents for elevation of a depressed skull fracture.

HPI: Pt fell out of the car onto the sidewalk, causing a severe scalp laceration which bled
profusely. A depressed parietal skull fracture was palpated in the emergency department
and detected by skull films. The pt has been crying and inconsolable since the injury,
which occurred 1 hour ago.

PMI: Tetralogy of Fallot, treated with a Blalock-Taussig shunt inserted at 1 year of life. The pt
is seen by his cardiologist every few months and has reportedly done well. He has a
simple case of TOF with moderate pulmonic stenosis. His "tet-spells" are treated by
squatting. He was scheduled for definitive repair of his TOF at a later date.

PE: P 140, BP 90/60, R crying, T 37.5°C.

CXR: "Boot-shaped" heart; no fractures, pneumothorax, or other signs of trauma.

ECG: NSR, right ventricular hypertrophy, right axis deviation.

Labs: Hct 45, electrolytes normal.


265 The Essential Oral Board Review

Pediatrics III (Session 2)

A 4 year old, 15 kg male with tetralogy of Fallot and prior Blalock-Taussig shunt presents for
elevation of a depressed skull fracture from a recent fall. P 140, BP 90/60, R crying, T 37.5°C,
Hct 45.
266 The Essential Oral Board Review

Pediatrics III (Questions)

A 4 year old, 15 kg male with tetralogy of Fallot and prior Blalock-Taussig shunt presents for
elevation of a depressed skull fracture from a recent fall. P 140, BP 90/60, R crying, T 37.5°C,
Hct 45.

A. Preoperative Evaluation
1. Tetrology of Fallot (TOF): How does TOF affect your approach with a patient with a
traumatic head injury (spell, CVA, cyanosis, Hct, shunt) Do you need to see a CXR?
Should you give sedation to stop his crying? Is crying bad or good?
2. Hematocrit: What is the significance of a Hct of 45?
B. Intraoperative Management
1. Cardiac: What are your hemodynamic goals for this patient? Which is more important,
maintaining cerebral perfusion pressure or preventing right-to-left shunting?
2. Monitors: Which would you use? AL? Where?
3. Induction: What are your goals for induction? How would you achieve them?
4. Hypoxemia: Immediately following intubation, the pulse oximeter reads 79%. Could this
be a Tet spell?
5. Hypotension: The BP is 50/25. Is this a problem with a 4 year old child?
C. Postoperative Care
1. Clotting BTS: The murmur is decreased. Concerns? What is the consequence of a
clotted BTS? Suspecting one, what would you do immediately?
2. Cerebrovascular accident: The patient does not immediately awaken. DDx? At what
point would you order a CT scan? A neurology consult?
268 The Essential Oral Board Review

Pediatrics IV (Session 1)

A 10 day old, 3.5 kg male infant is scheduled for a pylormyotomy.

HPI: He has been healthy and vigorous at home until 2 days ago, when projectile bilious
vomiting began after each feed. He has become increasingly lethargic. Today, his
diapers were dry.

PMI: Full-term infant, bom to a mother with poor prenatal care and suspected cocaine abuse
by normal vaginal delivery. He was discharged to home on day of life 3 in good
condition.

PE: HR 120, BP 70/50, RR 36, Temp 37.5°C. The pt appears normally developed. He is
lethargic and has cold extremities. His mucous membranes are dry, his anterior
fontanelle is sunken, and his skin turgor is poor. His lungs and heart sounds are normal.
He withdraws appropriately to deep pain.

CXR: Normal.

ECG: None.

Labs: Hct 48, Na+ 150 mEq/L, K+ 5.2 mEq/L, CI" 99 mEq/L, HC03" 20 mEq/L.

The pt is scheduled for emergency surgery as soon as the surgeon arrives, which should be in 15
minutes.
0S

273 The Essential Oral Board Review

Pediatrics V (Session 2)

A 35 week gestational age, 2200 gram premature infant presents for repair of a transesophageal
fistula. She was born at 32 weeks gestation but to a mother with preterm labor, but never
required intubation. She has a barking cough. P 120, BP 50/30, R 42, Temp 36.4°C, Hgb 15
(WW gm/dl.

^p>
274 The Essential Oral Board Review

Pediatrics V (Questions)

A 35 week gestational age, 2200 gram premature infant presents for repair of a transesophageal
fistula. She was born at 32 weeks gestation but to a mother with preterm labor, but never
required intubation. She has a barking cough. P 120, BP 50/30, R 42, Temp 36.4°C, Hgb 15
gm/dl.
A. Preoperative Evaluation
1. Tracheoesophageal fistula: What is a TEF? Could this have been diagnosed in utero?
Can you draw the most likely type. Where is it usually located? How does its location
affect your anesthetic management? Why do you need a preop CXR? How does
aspiration occur? Would a nasoesophageal tube prevent aspiration?
2. Tracheomalacia: Why does she have a barking cough? (TM, aspiration) If she had
tracheomalacia, would that cause inspiratory or expiratory problems?
3. VATER syndrome: Is a TEF associated with other congenital anomalies? (vertebral,
VSD, anal atresia, TEF, radial, renal)
4. Prematurity: What concerns do you have regarding the patient's prematurity (AB: RDS,
A/B; C: CHD, PDA; D: ICH; FEN: dehydrate; Gl: NEC; H: anemia, coag; ID: sepsis; Joints
and skin; Lines, Meds)
B. Intraoperative Management
1. Monitors: What monitor would you use? Where would you place a precordial
stethoscope, on the R or L axilla?
2. G-tube: Would you request a G-tube prior to surgery? Why? How? During the
procedure, the child cries and becomes cyanotic. The stomach becomes distended. Rx?
3. Induction: How would you induce anesthesia? Would you suck out the esophageal
pouch beforehand? Awake vs an inhalational induction?
4. Ventilation: How would you assure proper ETT placement? Does it matter which way
the bevel of the ETT faces? ABG shows pH 7.33, pC02 46, p02 242.
5. Transfusion: Significant hypotension occurs. Rx? There is significant bleeding. Would
you administer blood, crystalloid, or colloid? How much?
C. Postoperative Care
1. Ventricular septal defect (VSD): A large holosystolic murmur is heard postop. What
would you do? Does it matter whether it is a VSD or a PDA? Ausculation of the lung
fields reveals bilateral crackles and the BP is 40/20. Rx? Diuretic vs dopamine?
2. Reactive airway disease: The family asks that there is a family history of asthma and
asks you if their daughter has a increased risk of asthma.
269 The Essential Oral Board Review

fwc
Pediatrics IV (Session 2)

nm A 10 day old, 3.5 kg infant with a 2 day history of projectile vomiting is scheduled for a
pylormyotomy. He is lethargic and has cold extremities. HR 120, BP 70/50, RR 36, Temp
37.5°C, Hct 48, HC03" 20 mEq/L.

tWS

(w(

^
270 The Essential Oral Board Review

Pediatrics IV (Questions)

A 10 day old, 3.5 kg infant with a 2 day history of projectile vomiting is scheduled for a
pylormyotomy. He is lethargic and has cold extremities. HR 120, BP 70/50, RR 36, Temp
37.5°C, Hct 48, HC03* 20 mEq/L.

A. Preoperative Evaluation
1. Pyloric Stenosis: What is the likely diagnosis? What is it?
2. Fluids: What electrolyte abnormalities would you expect to find? Why? What is the role
of the adrenal glands in creating these abnormalities? What is the fluid of choice? Do you
need dextrose? How is the ability of the neonate to handle fluids different that the adult?
The patient becomes more active after a fluid challenge. Would you proceed with
surgery?
3. Congenital anomalies: Are associated congenital abnormalities likely? A murmur is
heard over the precordium. What is the most common congenital cardiac abnormality?
B. Intraoperative Management
1. Monitors: What routine monitors do you require? Does this patient need an arterial line?
2. Induction: Is a rapid sequence induction indicated? What exactly is it? What is a
modified RSI? Is it indicated? What if the child had been NPO x 2 days? Why not an
awake intubation? Would you insert an OG tube prior to intubation?
3. Cleft palate: With direct laryngoscopy, you discover that the patient has a cleft palate.
How does this affect your anesthetic care? Does it make intubation more difficult?
4. Aspiration: The child vomits and a large amount of vomitus enters the larynx. What
would you do? Breath sounds are louder on the left. Your response? The case
proceeds, but the pulse oximeter reads 95%. Would you order a CXR?
5. Maintenance: How would you maintain anesthesia? Are anesthetic requirements
different for pediatric patients?
C. Postoperative Care
1. Extubation: Would you extubate this patient awake or deep? How does his aspiration
affect your plans? What if the surgeon requested a deep extubation?
2. Seizure: In the PACU, the patient has a generalized tonic-clonic seizure. Your
response? Your goals?
272 The Essential Oral Board Review

Pediatrics V (Session 1)

A 35 week gestational age, 2200 gram premature infant presents for repair of a transesophageal
fistula and anoplasty.

HPI: Diagnosis of TEF was suspected in utero due to polyhydramnios. Esophagram after birth
confirmed the diagnosis of

PMI: Pt was born at 32 weeks gestation. The mother with preterm labor and was given
steroids. The only required supplemental 02, and was never intubated. A complete
genetic workup is in progress. Positive findings thus far include thoracic vertebral
defects, anal atresia, and horseshoe kidneys. The head ultrasound is normal.

PE: P 120, BP 50/30, R 42, Temp 36.4°C. The pt has an occasional barking cough. Her
lungs are clear to auscultation. No murmur is heard over the precordium. Anal atresia is
present. The rest of the exam is unremarkable.
CXR: Normal.

ECG: NSR, RVH.

Labs: Hgb 15 gm/dl, electrolytes and ABG within normal limits.


276 The Essential Oral Board Review

Pediatrics VI (Session 1)

A 12 year old, 35 kg male presents for emergency repair of an open radial fracture.

HPI: The pt fell from a swing during his 12th birthday party. He had just finished eating cake.
An empty bottle of imipramine was found in his home.

PMI: Down's syndrome with mild mental retardation. He has a history of recurrent aspiration
pneumonia and urinary incontinence.
PE: P 100, BP 140/60, R 22, T 36.9°C. The pt is difficult to arouse, old tracheostomy scar

CXR: No fractures.

ECG: NSR with a prolonged QT interval

Labs: Hct 48.


277 The Essential Oral Board Review

Pediatrics VI (Session 2)

/m\ A 12 year old, 35 kg male with Down's syndrome presents for emergency repair of an open radial
fracture after falling from a tree. An empty bottle of imipramine was found in his home. He has a
history of recurrent aspiration pneumonia, urinary incontinence, and an old tracheostomy scar.
Hct 48, his EKG shows a prolonged QT interval. P 100, BP 140/60, R 22, T 36.9°C. The pt is
difficult to arouse.

(mS

0*\

fms
278 The Essential Oral Board Review

Pediatrics VI (Questions)

A12 year old, 35 kg male with Down's syndrome presents for emergency repair of an open radial
fracture after falling from a tree. An empty bottle of imipramine was found in his home. He has a
history of recurrent aspiration pneumonia, urinary incontinence, and an old tracheostomy scar.
Hct 48, his EKG shows a prolonged QT interval. P 100, BP 140/60, R 22, T 36.9°C. The pt is
difficult to arouse.

A. Preoperative Assessment
1. Airway: Do you need airway films? Why?
2. Down's Syndrome: What is the most common associated congenital heart disease?
3. Neurologic assessment: What do you think about his neurologic status? Is there any
reason he should receive steroids?
4. Tricyclic antidepressants: What do your think about the EKG? What would you do
about the bottle of imipramine?
B. Intraoperative Management
1. Regional anesthesia: Is an axillary block appropriate for a patient with Down's
syndrome? Is it appropriate for this type of surgery? What additional nerves do you need
to block?
2. Bier block: The surgeon asks whether this case could have been done with a Bier block.
He says that he usually does them in his office all the time-without exanguinating the arm.
Your opinion? Where the hell did you learn that technique? When you ask him how he
would handle an inadvertent tourniquet deflation, he responds, "Dr., that's why God gave
us valium."
3. Induction: Is a rapid sequence induction required?
4. Elevated airway pressures: After intubation, the peak airway pressures are 60 cm H20.
What would you do? DDx?
5. Anesthetic: What type of anesthetic would you administer?
6. Bleeding: While receiving isoflurane, N20, fentanyl, and atracurium, the patient's BP is
70/30. What would you do?
7. Dysrhythmias: The EKG shows torsade de points. What is that? Rx?
C. Postoperative Care
1. Ventilator: What ventilator settings would you recommend for a Pa02 of 55 on 100% 02.
What is your approach to the problem?
2. Missing tooth: In the PACU, his right upper incisor is noted to be missing. What would
you do?
280 The Essential Oral Board Review

Pediatrics VII (Session 1)

A 4 year old, 30 kg boy is scheduled for outpatient strabismus surgery.

HPI: Pt 3 developed reflux and chronic vomiting at 3 yrs of age. He has responded fairly well to
medications, but without complete control of his reflux symptoms. He had a UR110 days
ago with coughing, a temperature of 100.0°C, and diarrhea. No antibiotics were given.
He has felt well for the past 5 days, without cough or fever. He is taking propulsid,
Prilosec, Tylenol PRN.

PMI: 3 episodes of pneumonia, none requiring hospitalization. He is allergic to penicillin, the


reaction to which is unknown.

PE: P 100, BP 89/55, R 26, T 36.6°C. Normal head and neck, airway appears normal, chest is
clear, cardiac exam is unremarkable.

Labs: None

You meet the pt and family in the holding area. He has already received his AM medications. He
offers no resistance when you reach to pick him up
i#ps 281 The Essential Oral Board Review

0 Pediatrics VII (Session 2)

A 4 year old, 30 kg boy is scheduled for outpatient strabismus surgery. The pt has
gastroesophageal reflux and a URI with fever 10 days ago. He has been asymptomatic for the
past 5 days. His past medical history is significant for 3 episodes of pneumonia, none requiring
hospitalization. He is taking propulsid, prilosec, Tylenol PRN. P 100, BP 89/55, R 26, T 36.6°C.
His airway appears normal and his chest is clear to auscultation.

r*

0m

(SPN

m0\

/P\
282 The Essential Oral Board Review

Pediatrics VII (Questions)

A 4 year old, 30 kg boy is scheduled for outpatient strabismus surgery. The pt has
gastroesophageal reflux and a URI with fever 10 days ago. He has been asymptomatic for the
past 5 days. His past medical history is significant for 3 episodes of pneumonia, none requiring
hospitalization. He is taking propulsid, Prilosec, Tylenol PRN. P 100, BP 89/55, R 26, T 36.6°C.
His airway appears normal and his chest is clear to auscultation.

A. Preoperative Assessment
1. Pediatric anatomy and physiology: How does it differ from that of an adult?
2. URI: What is the significance of a preoperative history of a URI? For how long should
you delay elective surgery? Why? Is it OK to proceed with a mask case? Does it matter
whether the secretions are yellow or green? When would you obtain a WBC or CXR?
What does the differential on the CBC mean?
3. Strabismus: What is it? Any associated anomalies?
4. GER: What is it? Prophylaxis? Cimetidine or ranitadine? Side effects?
B. Intraoperative Management
1. Induction: How will you induce anesthesia-mask or IV? What about his history of
reflux? What if you were unable to attain IV access? Eventually, you are able to place an
IV. What drugs would you use for induction? Succinylcholine? How would it affect the
operation? What is the forced duction test?
2. Trismus: After the succinylcholine, you are unable to open the mouth. How would you
proceed? After several minutes, the mouth can be opened. Would you continue with
surgery? Assuming you continued, how would this event change your anesthetic
management?
3. Hypercapnea: After 20 minutes, you notice the ETC02 reads 46, up from 32.
Differential? What if the baseline were also elevated? P 103, BP 120/77, T 37.3°C. What
tests can you do to rule out MH?
4. MH: The ABG reads pH 7.11, Pa02 599, PaC02 57. Response? Additional dantrolene?
Additional monitors?
C. Postoperative Care
1. Renal failure: The urine output is 12 cc/3 hrs. Is this normal? Additional monitors?
(CVP) Dialysis? How would you decide?
2. Aspiration: CXR reveals a RUL infiltrate. What is the treatment for aspiration
pneumonia? Steroids? Bronchoscopy? Antibiotics? Which ones?
3. Extubation: Assuming the acidosis is corrected and the pt is producing 0.5 cc/kg of tidal
volume, would you extubate? How does the finding of pneumonia affect your decision?
Don't XR findings lag behind clinical findings?
4. Hypoxia: After a week of antibiotics, the pt is still on the ventilator with PIP 45, PEEP 15,
rate 30, and Fi02 80%. The ABG shows pH 7.34, PaC02 56, PaOz 140. Interpretation?
How would adjust the ventilator? Calculate the expected PA02, assuming R = 0.8 and Pb
is 750 mm Hg. How do you explain the large A-a gradient? Dead space? Diffusion
barrier?
284 The Essential Oral Board Review

Pediatrics VIII (Session 1)

A 6 year old, 17kg male is scheduled for a reduction/internal fixation of an open tibial fracture.

HPI: Pt fell out of a tree 8 hours ago. He had no loss of consciousness, but his neck films
show a possible C5-6 subluxation. A radiologist will not be available to review the films
until the next day.

PMI: Sickle ceil anemia. He has had multiple admissions for vasoocclusive crises, first as an
infant for hand and foot dactylitis, then other times for pneumonia. He has had several
transfusions. His parents claim that he has become less active in play during the past
year. He is taking amoxicillin, pneumococcal vaccine, folate, and hydroxyurea.

PE: P 130, BP 80/45, R 29, Temp 38.8°C. His airway exam is unremarkable. He is in a
cervical collar. The lungs are clear to ausculation. A 3/6 systolic ejection murmur is
heard over the right upper sternal border.

CXR: Mild cardiomegaly.

ECG: None.

Labs: Hct 23, WBC 12.1, Na+149 mEq/L, K+ 5.9 mEq/L, CI" 113 mEq/L, HC03" 20 mEq/L.
Urine specific gravity 1.010.
/ ff r 285 The Essential Oral Board Review

Pediatrics VIII (Session 2)

^ v A 6 year old, 17kg male with sickle cell anemia is scheduled for a reduction/internal fixation of an
open tibial fracture. He fell out of a tree 8 hours ago. P 130, BP 80/45, R 29, Temp 38.8°C. He
is in a cervical collar. A 3/6 systolic ejection murmur is heard over the right upper sternal border.
Hct 23. Neck films show a possible C5-6 subluxation. CXR shows mild cardiomegaly. He is
jp taking amoxicillin, pneumococcal vaccine, folate, and hydroxyurea.
/■ft

286 The Essential Oral Board Review V^

Pediatrics VIII (Questions)


X
V
A 6 year old, 17 kg male with sickle cell anemia is scheduled for a reduction/internal fixation of an "*
open tibial fracture. He fell out of a tree 8 hours ago. P 130, BP 80/45, R 29, Temp 38.8°C. He ^
is in a cervical collar. A 3/6 systolic ejection murmur is heard over the right upper sternal border. *
Hct 23. Neck films show a possible C5-6 subluxation. CXR shows mild cardiomegaly. He is *
taking amoxicillin, pneumococcal vaccine, folate, and hydroxyurea. %

A. Preoperative Assessment "*


1. Sickle cell anemia: What is it? What is the defect? What types of crises exist? What is K.
the pathophysiology? What is the P50? Is fever good or bad? Why is he taking his .
medications? What does hydroxyurea do? **
2 . To u r n i q u e t : T h e s u r g e o n i n s i s t s o n a t o u r n i q u e t . Wo u l d y o u a l l o w t h i s ? W h y ? J ^ ,
3. Optimization: How would you prepare the pt for surgery? What are your goals? Doe
you think the pt is dehydrated? Is the urine SG useful in pts with Hgb SS? What is your %
goal for the %Hgb SS? \*
4 . H e m o s i d e r o s i s : W h y t h e D O E ? C a r d i o m e g a l y ? W h a t i s t h e d i ff e r e n c e b e t w e e n a ^
constrictive and restrictive cardiomyopathy? ^f
B. Intraoperative Management w^
1. Anesthesia: What technique? Would you consider a spinal (it's a very mature 6 year ~
old)? Is sickle cell anemia a contraindication? ^
2. Induction: How would you induce anesthesia? He last ate 12 hours ago. Does the pt ^
have a full stomach? ^
3 . M o n i t o r : W h y w o u l d y o u p l a c e a C V P ? W o u l d a PA C b e m o r e u s e f u l ? j £
4. Maintenance: What technique would you use? Is recall likely with a nitrous narcotic
technique? How about ketamine? ^>l
5. Tourniguet pain: The HR gradually increases to 150. What would you do? What is the ^.
cause of tourniquet pain? How can it be treated?
6. Hgb AS: What if he had the trait? Are crises possible? \
C. Postoperative Care w
1. Consults: Do you need a hematology consult? Why? A cardiology consult? Why? *
What s p e c i fi c questions would you ask? %t
2. CVA: What if the pt were difficult to arouse. Would you get a CT? EEG? Do you need ^
a neurology consult? What is you DDx? **
3. Asplenia: Is this pt at increased risk for infection? ^
4. Seguestration crisis: In the chart, the HR is 150. Response? BP 60/28. Response? ^
Sepsis? What if the abdominal were distended? **

Mk
288 The Essential Oral Board Review ^

r v.

Pediatrics IX (Session 1) %
M
A 5140 g, 38 weeks estimated gestational age male neonate is scheduled for closure of an %
omphalocele and repair of bladder e x t r o p h y. *

HPI: Pt was born 1 day ago to a 24 year old G4P3 female with poor prenatal care by normal Jg
spontaneous vertex vaginal delivery. Apgars were 8 at 1 minute and 9 at 5 minutes. The ^
pt developed a seizure at 2 hours of life, which was treated with a bolus of D50. The «
blood glucose at the time was 20 mg/dl. The pt also has a large VSD, which has been ^
evaluated by cardiology and found to have minimal shunt flow at this time, and a patent
PDA.

PE: P 150, BP 64/34, R 36, Temp 36.5°C. The pt is pink and vigorous. Breath sounds are
clear. There are no murmurs on auscultation. His extremities are well perfused.

CXR: Large anterior mediastinal mass.

ECG: Biventricular h y p e r t r o p h y. w

Labs: Hct 45, Na+146, K+ 4.9 mEq/L, CI* 100 mEq/L, HC03" 21 mEq/L. Coagulation studies are %
normal. ^

The pt has an IV with D10W running at 120 cc/hr. Jg

0WR
/fer

2o9 The Essential Oral Board Review

Pediatrics IX (Session 2)

A 5140 g, 38 weeks estimated gestational age neonate is scheduled for closure of an


omphalocele and repair of bladder extrophy on day of life 1. The pt developed a hypoglycemic
seizure at 2 hours of life, which was treated with a bolus of D50. The pt also has a large VSD,
which has minimal shunt flow, and a patent PDA. P 150, BP 64/34, R 36, Temp 36.5°C. Hct 45.
ECG shows biventricular hypertrophy. CXR reveals a large anterior mediastinal mass. Other
labs are normal for age.

fW
/^!

290 The Essential Oral Board Review ^

Pediatrics IX (Questions) ^

A 5140 g, 38 weeks estimated gestational age neonate is scheduled for closure of an ^


omphalocele and repair of bladder extrophy on day of life 1. The pt developed a hypoglycemic ^
seizure at 2 hours of life, which was treated with a bolus of D50. The pt also has a large VSD, .
which has minimal shunt flow, and a patent PDA. P 150, BP 64/34, R 36, Temp 36.5°C. Hct 45. ^
ECG shows biventricular hypertrophy. CXR reveals a large anterior mediastinal mass. Other ^
labs are normal for age. "'
rail
A. Preoperative Assessment w
1. Gastroschesis vs Omphalocele: What is an omphalocele? Which is easier to care for? ^
Why? Which is associated with congenital anomalies? (trisomy 13,15, 21) What is \
pentalogy of Cantrell? (ectopia cordis, sternal cleft, diaphragmatic defect, cardiac defect, fc,
omphalocele) **>
2. Beckwidth-Wiedemann syndrome: What is it? Related to seizures? Is the J^
pathogenesis of the seizures similar to those in IDMs? To pts with neisidioblastosis? Are
IDMs predisposed to any congenital cardiac anomaly?
3. Cardiovascular: What are your priorities? How would you assess his cardiac system?
Do you think he has a thoracic lymphoma? How would you assess the significance of the
VSD? If it is large, why isn't there a murmur? A shunt? What do you think about the
ECG? How does a PDA sound on auscultation?
4. Fluids: Is it appropriate to give D10W only? Why/why not? What factors affect the pt's
fl u i d needs? M,
5. Temperature: Why is heat loss in this pt greater than with an adult? How would you fcj
prevent it? **
6. Other: What other measures should be taken preoperatively? (abx, NG) Do you need to SS
see a PT/PTT? ^
B. Intraoperative Management ^
1. Monitoring: How? CL necessary? fe»
2. Induction: How? If awake, what about ICH? Ketamine contraindicated in neonates?
How big an air leak?
3. Maintenance: How? Why? Jfe
4. Circuit: Is a circle system safe to use in neonates? Does your answer depend on ^.
whether or not the pt is breathing spontaneously? ^
5. IVFs: What type during case? When to use colloids? Surgeon says just give FFP. Jj
6. ABG: Would you send off ABGs? Why if you have ETC02 and pulse oximetry? How ^
many? Cost per? 7.32/46/120.
7. Bleeding: When to give blood? Hct is still 38. How much? Type of blood? CMV
negative? Irradiated?
8. Elevation of PIP: With closure of abdomen, desaturation occurs. Response? When to SBj
use staged closure? (intraabdominal pressure >20 mm Hg or CVP <4 cm H20) Would J^
the CVP help you decide? Doesn't the CVP go up with the abdominal compartment 7j
syndrome? &
9. Extubation: At end of case what are your extubation criteria? ^j
C. Postoperative Care *5
1. Hypoglycemia: Days later, pt lethargic, hypotensive. DDx? %£
2. Sepsis: How do you diagnose sepsis in the neonate? If it were suspected, should you y
perform an LP? Would you do it yourself? What would you send in the tubes? What *?
antibiotics should the pt receive? %d
3. CHF: On DOL 2, the pt has bilateral pulmonary edema. Your approach? Would you give ^j
prostaglandins? Indomethacin? How do you balance the need to fluid restrict for ^5
pulmonary edema with the need to volume resuscitate for sepsis?
4. Prematurity: How would your management have changed if the pt were 26 week
jM
premature infant with ileal atresia?
292 The Essential Oral Board Review

Obstetrics I (Session 1)

A 16 year old G2Ab1 60 kg, 5*2" woman who is 36 weeks pregnant requires a labor epidural.

HPI: Pt complains of swelling in her ankles and headaches for the past month. She has
become increasingly short of breath during the last trimester. She has been given
magnesium infusion. The fetal heart tracing is normal.

PMI: The pi had poor prenatal care.

PE: P 90, BP 190/110, R 30, T 39.4°C. Only the tongue is visible with oral exam. The chin is
short. Her neck is supple. The lungs are clear to auscultation. There are no rubs, clicks,
or murmurs. The right upper quadrant is tender to palpation. Total body edema is noted.
The pt is sleepy, but oriented when aroused.

CXR: Slight increased pulmonary vascularity.

ECG: NSR, left-axis deviation, occasional PVCs.

Labs: Hct 35, Hgb 9 gm/dl, platelets 95K 4 hrs ago, electrolytes normal, PT/PTT pending. The
urine drug screen is positive for cocaine. The Mg2+ level is 11 mEq/L.
293 The Essential Oral Board Review

Obstetrics I (Session 2)

A 16 year old G2Ab1 60 kg, 5'2" woman who is 36 weeks pregnant requires a labor epidural.
She has had swelling in her ankles for the past month, recent headaches, and constant right
upper quadrant abdominal pain today. The urine drug screen is positive for cocaine. P 90, BP
190/110, R 30, T 39.4°C, Hct 35.
294 The Essential Oral Board Review

Obstetrics I (Questions)

A 16 year old G2Ab1 60 kg, 5'2" woman who is 36 weeks pregnant requires a labor epidural.
She has had swelling in her ankles for the past month, recent headaches, and constant right
upper quadrant abdominal pain today. The urine drug screen is positive for cocaine. P 90, BP
190/110, R 30, T 39.4°C, Hct 35.

A. Preoperative Evaluation
1. Pregnancy induced hypertension: Does she have a syndrome? How is the diagnosis
made? What systems are affected by PIH? What is its pathogenesis?
2. Cardiovascular: Are an ECG and CXR necessary? What is the significance of her
hypertension? Why does she have it? Why is it necessary to Rx? What complications
can occur? What cardiac abnormalities might she have (fibrosis, dysrhythmias from
cocaine)? What is the treatment? What is her likely intravascular volume and CO? How
does hydralazine work? Is it better than nitroprusside? Could you use labetolol or
esmolol?
3. Neurologic: What is the difference between eclampsia and preeclampsia? How does Mg
work? Interaction with other drugs?
4. Liver rupture: How would you address her abdominal pain? Would an epidural be
helpful? Would you obtain LFTs?
5. Coagulation: What is the basis of her coagulopathy? How would you detect?
6- fetal: How would you assess fetal well being? How does PIH affect the infant?
B. Intraoperative Management
1. Monitoring: What special monitors do you require? Is a CVP essential? A PA catheter?
What is the significance of early decelerations? Late decelerations are seen with good
variability. Should scalp a pH be obtained?
2. Choice of anesthetics: Is a regional or general for a C-section preferable? What is the
basis for your decision? Would a platelet count of 95K change your plan? 75K? 25K?
Would you preload the patient before an epidural? What local anesthetic would you use?
Is phenylephrine better than ephedrine? (cocaine NE depletion)
3. Failed epidural: The patient complains of pain in her left lower quadrant with placement
of a surgical clamp. Why? Would you administer more local anesthetic, ketamine,
fentanyl, or induce general anesthesia? The airway is class 3, the TMD is 5 cm, and there
is full range of motion of her neck. How would you induce general anesthesia? Fetal
bradycardia is noted and a stat C/S is necessary. How would you proceed? Wheezing is
noted after intubation. Rx?
4. Hypertension: Following intubation, the BP increases to 220/120. Is this an emergency?
Would a bolus of intravenous lidocaine be appropriate?
C. Postoperative Care
1. Neonatal resuscitation: The neonate born with thick meconium aspiration and Apgars of
2. Would you abandon the mother to care for the infant? What are your goals in
resuscitation? Is naloxone the drug of choice? Bicarbonate? Intubate or mask ventilate?
When would you stop chest compressions? How would you administer epinephrine?
What other drugs can you administer through the ETT? The baby is floppy. Would you
administer Ca2+?
2. Seizures: While transporting the mother in the hall, she begins twitching her arm. Would
you give midazolam? By the time you arrive in the recovery room, she is having a tonic-
clonic seizure. Would you administer succinylcholine?
3. Congestive heart failure: After intubation, her pulse oximeter reads 88%. What would
you do? Diffuse crackles are heard bilaterally. DDx? Why is she at risk for pulmonary
edema? Is nitroglycerine a good choice? Doesn't it inhibit hypoxic pulmonary
vasoconstriction?
296 The Essential Oral Board Review

Obstetrics II (Session 1)

You are asked to evaluate and make recommendations on a 33 year old, 80 kg, 58" G4P3
parturient who develops painless vaginal bleeding at 36 weeks gestation.
HPI: The pt developed contractions 6 hrs ago. The bleeding, which is moderate and bright
red, began 3 hrs ago.

PMH: She has a history of hypertension well controlled by diet during pregnancy. She has had
3 prior cesarean sections, all by epidural anesthesia. She required a blood transfusion
from the last one, but is unfamiliar with any details.

There have been some episodes of early fetal heart tone deceleration, but they seem to
have diminished with repositioning the pt.

PE: P 110, BP 170/100, R 26, T 39.0°C. Airway, lung, cardiac exam are normal.

CXR: None.

ECG: None.

Labs: Hct 35, BUN 15, creatinine 0.8, ABG pH 7.35, PaC02 27, Pa02 115.
297 The Essential Oral Board Review

Obstetrics II (Session 2)

You are asked to evaluate and make recommendations on a 33 year old, 80 kg, 58" G4P3
parturient who develops painless vaginal bleeding at 36 weeks gestation. She has a history of
hypertension and has had 3 prior cesarean sections. She is receiving magnesium. P 110, BP
170/100, R 26, T 39.0°C. Hct 35, BUN 15, creatinine 0.8, ABG pH 7.35, pC02 27, p02 115.
298 The Essentia! Oral Board Review

Obstetrics II (Questions)

You are asked to evaluate and make recommendations on a 33 year old, 80 kg, 58" G4P3
parturient who develops painless vaginal bleeding at 36 weeks gestation. She has a history of
hypertension and has had 3 prior cesarean sections. She is receiving magnesium. P 110, BP
170/100, R 26, T 39.0°C. Hct 35, BUN 15, creatinine 0.8, ABG pH 7.35, pCOz 27, p02 115.

A. Preoperative Evaluation
1. Changes of pregnancy: Are you concerned about any of her labs? Is her anemia
consistent with bleeding? Does the hypocoaguable state of pregnancy predispose to
hemorrhage? The blood gas lab calls with the concern that the reported ABG values may
belong to another patient? Based on their values, is that likely? Do you think this patient
has renal failure? What happens to renal function during pregnancy?
2. Antepartum bleeding: What are the most likely causes of antepartum bleeding? (previa,
accreta/increta/percreta, abruptio, rupture)
3. Placenta previa: What is it? Risk factors? (age, parity, scar, D/C, myomectomy, tumor,
smoking, daughter of DES mom, C/S) How does it present? (painless) Types? (total,
partial, marginal) What the difference between acreta (attaches to myometrium), increata
(invades myometrium), and percreta (goes through myometrium)? Does her C/S history
increase the likelihood of either? (with previa: =5% accreta, 1 C/S=24% accreta, 4
C/S=67% accreta).
4. Uterine rupture: Could this be uterine rupture? Risk factors? (scar, prior difficult
delivery, tumultuous labor, oxy, tumor, myoma, PID, oophorectomy, C/S)
5. Abruptio: Could this be abruptio? Risk factors? (HTN, mulip, uterine anomalies,
previous abruption, PROM).
B. Intraoperative Management
1. Anesthesia for a double set-up: The obstetricians want to perform a vaginal exam to
rule out previa. What is a double set-up? Is it reasonable for the obstetricians to request
that you remain in the hospital for procedure? Would ketamine be a good induction drug?
Is MAC any different in this patient? Would you perform an epidural? Clear fluid is
aspirated from the catheter. How can you test it for CSF?
2. CPR: During the exam, massive bleeding occurs and the patient becomes hypotensive
and looses consciousness. Response? Despite CPR, no palpable pulse is felt?
Response? What if LUD is ineffective? How is LUD effective?
3. Maintenance: What anesthetic would be most appropriate to minimize fetal depression?
4. Hysterectomy: After delivery, the bleeding continues. Possible therapies? What are the
indications for hysterectomy? (previa, acreta, rupture, atony, chorio, DIC, broad lig
hematoma, fibroids preventing uterine closure)
C. Postoperative Care
1. DIC: Diffuse oozing continues. Would you order FFP and cryoprecipitate? Likely
diagnosis? Can fibrinogen split products cause uterine atony? Would D-dimers be
useful? Why are they used?
2. Neonatal resuscitation: The baby is floppy, apneic, dusky, flaccid, and has a thready
heart rate of 120. What is the Apgar score? Is it predictive of survival? How would you
administer epinephrine or HC03? Ca2+?
3. Spinal headache: The patient complains of postoperative headache, nuchal rigidity, and
diplopia. Do you think this is a spinal headache? How would you evaluate? Would you
administer a blood patch? Do you routinely administer one prophylactically?
300 The Essential Oral Board Review

Obstetrics III (Session 1)

A 36 year old, 66 kg, 5'3" female presents for clipping of cerebral aneurysm.

HPI: Pt presented with sudden onset of the worst headache of her life today. There was no
loss of consciousness. CT scan of the head revealed an anterior communicating artery
aneurysm. The pt was admitted to the neurologic intensive care unit, where she was give
sedation and nimodipine was begun.

PMI: Pt is 24 weeks pregnant. Fetal ultrasound performed in the ICU reveals a normal
pregnancy. She had a history of rheumatic fever during childhood. During pregnancy,
she reports worsening dypsnea on exertion. Evaluation of a murmur revealed a case of
moderate mitral regurgitation.

PE: P 70, BP 160/85, R 20, T 37.0°C. She is currently sleepy, but is alert when aroused for
questioning. Her airway is normal. Breath sounds are clear. A 3/6 systolic ejection
murmur is heard in the apex, radiating into the axilla.

CXR: No apparent disease.

ECG: EKG shows Q waves in leads 2, 3, and aVF

Labs: Hct 25, C02 22 mEq/L, other electrolytes and coagulation studies are normal.
301 The Essential Oral Board Review

Obstetrics III (Session 2)

A 36 year old, 66 kg, 5'3" 24 week pregnant female presents with the worst headache of her life
for an emergency craniotomy. She is currently sleepy, but when aroused for questioning, reports
a history of rheumatic fever. Medications: nimodipine. P 70, BP 160/85, R 20, T 37.0°C, Hct 25,
EKG shows Q waves in leads 2, 3, and aVF.
302 The Essential Oral Board Review

Obstetrics III (Questions)

A 36 year old, 66 kg, 5'3" 24 week pregnant female presents with the worst headache of her life
for an emergency craniotomy. She is currently sleepy, but when aroused for questioning, reports
a history of rheumatic fever. Medications: nimodipine. P 70, BP 160/85, R 20, T 37.0°C, Hct 25,
EKG shows Q waves in leads 2, 3, and aVF.

A. Preoperative Evaluation
1. Intracranial aneurysm and pregnancy: What is nimodipine? What is her likely
diagnosis? Prognosis? Do the physiologic changes of pregnancy affect the chances of
aneurysm rupture? What factors do affect aneurysm rupture? Which is more important-
maintaining cerebral perfusion or preventing aneurysm rupture?
2. Intracranial pressure vs bleeding: Which is more important-preventing rupture or
providing adequate placental blood flow? How can you optimize both?
3. Mitral stenosis: What is the significance of her rheumatic fever? How would you
evaluate them in the 15 minutes you have before the operation?
4. EKG: What is the significance of her findings? Should they be evaluated further?
B. Intraoperative Management
1. Monitors: Is an AL needed? The surgeons have tried both radials without success. Can
you induce with just a NIBP? Are you going to monitor the fetal heart tones? What for, if
the fetus is not viable? At what age would you begin?
2. Induction: Would you consider her a full stomach? Is a rapid sequence induction
indicated? How would you induce anesthesia? What are your goals? What drugs should
you avoid due to fear of fetal effects or miscarriage?
3. Maintenance: What drugs would you use for maintenance? N20? Benzodiazepines?
Are you worried about teratogenicity? Effects of fetal HR? What MAP would you try to
achieve? What is the lower limit of uteroplacental autoregulation? What factors affect
uteroplacental BF?
4. Ventilation: How are you going to manage ventilation? What ETC02 would you try to
achieve? Do you need to confirm that with an ABG?
5. Fluids: How are you going to manage fluids? Would you give dextrose?
6. Induced hypotension: The aneurysm is quite large. Which will the surgeon's require-
induced hypotension or induced hypertension? Profuse bleeding occurs and the surgeons
request induced hypotension to 50. To 40. Which is the best drug to use? Compare
thiopental vs isoflurane vs nitroprusside.
7. CPR: The bleeding is controlled, but the MAP is still only 45. What you do? Would you
deliver the fetus?
C. Postoperative Care
1. Hypervolemic hypertension and hemodilution: The surgeons want you to place a PAC
for postoperative care? They want to induce hypervolemic hypertension with
hemodilution. Why?
2. Pulmonary edema: The CXR the next day shows pulmonary edema. Why? Do you
have choose between decreasing the risk of vasospasm or treating the pulmonary
edema? The surgeons give lasix. Is that a good choice?
3. Premature labor: Contractions are noted in the neuro ICU. The nurse there asks if
pitocin should be given to stop them. Your response? Any laboratory values you would
like to see before starting terbutaline?
304 The Essential Oral Board Review

Obstetrics IV (Session 1)

A 24 year old, 60 kg, 5'6" female who is scheduled for an emergency appendectomy.

HPI: Pt developed right lower quadrant pain 2 days ago. She has had little oral intake since
then, except for a few sips of water.

PMI: Pt is 28 weeks pregnant. She also has a history of well-controlled hypertension and
asthma. Her medications include lasix, propranolol, and albuterol inhalers PRN.

PE: P 90, BP 110/40, R 24, Temp 38.7°C, Hct 36.

CXR: None.

ECG: None.

Labs: WBC 19.5, Hgb 10 gm/dl, platelets 227K, Na+ 145 mEq/L, K+ 4.6 mEq/L, C02 20 mEq/L,
CI" 110 mEq/L
0*S

&* 305 The Essential Oral Board Review

Obstetrics IV (Session 2)

A 24 year old, 60 kg, 5'6" female who is 28 weeks pregnant is scheduled for an emergency
appendectomy. She has a history of hypertension and asthma. Medications: lasix, propranolol,
and albuterol inhalers PRN. P 90, BP 110/40, R 24, Temp 38.7°C, Hct 36.

3>

00$

ipi
306 The Essential Oral Board Review

Obstetrics IV (Questions)

A 24 year old, 60 kg, 5*6" female who is 28 weeks pregnant is scheduled for an emergency
appendectomy. She has a history of hypertension and asthma. Medications: lasix, propranolol,
and albuterol inhalers PRN. P 90, BP 110/40, R 24, Temp 38.7°C, Hct 36.

A. Preoperative Evaluation
1. Hypertension: How would you distinguish hypertension from PIH? How do you diagnose
PIH? If PIH, is this severe? Effects on the fetus? Cause of PIH? Whatever the cause,
what are your concerns about HTN. Would you add additional antihypertensive agents?
2. Nonobstetric surgery: What are your concerns about nonobstetric surgery in the
parturient? When is the safest time to perform an elective case?
3. Magnesium: The obstetricians begin a Mg infusion. Why? (antiepileptic, antiHTN,
tocolysis) What are therapeutic levels? (4-8 mEq/L, 10 DTR, 12 relax, 15 resp, 30 CV).
Should you give steroids and terbutaline (like ritodrine, less tachy). Concerns? (Ped,
tachy, hypotension, hypoK, hyperglycemia, arrhythmias)
B. Intraoperative Management
1. Monitors: Would you place an AL? CVP? What would you expect it to show? PAC?
FHR? At what age is FHR monitoring possible? The ABG shows pH 7.45/32/115.
Interpretation?
2. Choice of anesthetics: Epidural vs GA? Pros and cons of each? Which coagulation
labs are most important?
3. Induction: What are your goals for induction? How would you balance the needs of the
mother with the needs of the fetus? (Mom: deep with cricoid, lower MAC; Fetus: stable,
no hypoxia, hypovolemia, SNS stim, anemia) Do you need a neuromuscular stimulator or
is waiting for fasciculations sufficient? (Mg2+)
4. Maintenance: Which drugs are teratogenic? What is the problem with N20? (methionine
synthetase for myelin and thymidine for DNA; but OK for in vitro fert and levels normal
after 60% x 4 hrs, OK to use!) Are you concerned about teratogenicity? (3-8 wks)
(Historically safe: STP, MS, meperidine, relaxants, low dose NzO)
5. Tachycardia: HR 180 with ST segment depression. Rx? Esmolol is given with hypoxia.
Your response? Wheezing is heard. Would you give albuterol?
C. Postoperative Care
1. Morbidity: For what type of M/M is she and her baby at risk? (IUGR, abruptio, UPI)
2. Apnea: What is your DDx for postop apnea? (02, C02, CVA, edema, Mg2+, reversal,
postictal, glucose, Na) How does Mg potentiate relaxation? (Ach release, decreasing Ach
depolarization, decrease muscle excitability)
308 The Essential Oral Board Review

Obstetrics V (Session 1)

A 32 year old, 77 kg, 60" female requires a stat cesarean section for a prolapsed cord and breech
presentation.

HPI: Pt is 38 weeks pregnant. She and her husband have been trying to conceive a child for 7
years. She has been in labor for 20 hrs, throughout which time the membranes have
been ruptured. The fetus has had variable decelerations during the last several hours
with poor beat-to-beat variability.

PMI: She has a history of normal pressure hydrocephalus and migraine headaches.
According to her medical record, she was extremely difficult to intubate during a
laparoscopy performed 1 year ago. No further details are available.
PE: P 80, BP 110/50, R 22, T 38.7°C. Her airway exam is normal. The lungs are clear to
auscultation. The heart has a regular rate and rhythm, without rubs, clicks, or mummers.
The pt is in acute distress secondary to labor pains.

CXR: None.

ECG: None.

Labs: Hct 32, WBC 12,

The pt is crying hysterically, "I don't care about me~someone please just save my baby!"
fX7\

•^ 309 The Essential Oral Board Review

17 Obstetrics V (Session 2)

A 32 year old, 77 kg, 60" female who is 38 weeks pregnant requires a stat cesarean section for a
A) prolapsed cord and breech presentation. She has a history of normal pressure hydrocephalus
^jg\ past.
and migraine
P 80, BP headaches.
110/50, According
R 22, T to38.7°C,
her medical record, she was difficult to intubate in the
Hct 32.

f*W*/
310 The Essential Oral Board Review

Obstetrics V (Questions)

A 32 year old, 77 kg, 60" female who is 38 weeks pregnant requires a stat cesarean section for a
prolapsed cord and breech presentation. She has a history of normal pressure hydrocephalus
and migraine headaches. According to her medical record, she was difficult to intubate in the
past. P 80, BP 110/50, R 22, T 38.7°C, Hct 32.

A. Preoperative Evaluation
1. Breech presentation: Types? Risks factors? (epidural in 1st stage?, previa, uterine
anomalies, tumor, fetal anomalies, hydrocephalus, multiple gestation, polyhydramnios)
Risks? (cervical laceration, perineal injury, shock, post-partum bleed, retained placental,
neonatal neuro injury, cord compression, esp frank and incomplete) Safe to deliver
vaginally? What if head stuck?
2. Fetal heart tracings: The baby develops late decelerations. Significance? What
different patterns exist? (BTBV - 3-6; long term variability 6-25; early = nl, late = UPI,
variable = cord, asphyxia if >70 bpm or >60") Are there 2 types of late? (vagal reflex and
myocardial ischemia) What causes bradycardia? (hypoxia, BB, post-term, hypothermia,
CHB with SLE) Tachycardia? (fever, chorio, p-agonist, atropine, hydrops, T3) Loss of
BTBV? (narc, barb, bzd, Mg2\ atropine, dysrhythmias, anencephaly)
B. Intraoperative Management
1. Choice of anesthetics: How are you going to induce anesthesia? Epidural? What about
her hydrocephalus? Is this like pseudotumor cerebri? (taps therapeutic) Which agent?
Why/why not is chloroprocaine? (Na+ bisulfite neurotoxicity, now diNa EDTA paralumbar
pain, metabolite interferes with narcotics/bupiv) Goals for test dose? Ideal single test
dose? (LA for T10 won't r/o IV injection, so either use 2 doses of LA or 1 dose with 15-20
mc epi)
2. Monitors: Is a pulse oximeter necessary for placement of epidural or spinal?
3. Preoxygenation: How long would you preoxygenate? Is 4 vital capacity breaths
adequate?
4. Wet tap: Clear solution is aspirated from the catheter. Is this CSF? Would you use 12 or
15 mg of bupivicaine for a spinal? Would you add epinephrine? Pros and cons?
Sufentanil? Would you perform a prophylactic blood or saline patch?
5. Hypotension: Would you prehydrate? How much of what solution? Why? After the
spinal, the BP is 80/50. All you have is phenylephrine. Would you give it? Potential
problems? Why is ephedrine usually preferred?
6. Failed spinal: The patient complains of pain in her LLQ with incision. Your response?
Which is best, propofol, ketamine, or N20? Which one crosses the placenta? (ketamine
can cause neonatal depression >1 mg/kg). The patient becomes agitated and the
surgeons request GA.
C. Postoperative Care
1 • Neonatal resuscitation: The neonate's Apgar score is 5. Significance? Rx? The
pediatricians blame it on the combination of the epidural, ketamine, and succinylcholine.
Agree? (not if hypotension is treated, and not unless >10 mg/kg sux or >1 mg/kg ketamine
are given) Is GA worse? (yes at 1 but not necessarily at 5 minutes, thus not due to
hypoxia; NB that hypoxia, hyperventilation, aortocaval compression, and anesthetic/N20
can all cause neonatal depression). What is the most important factor in determining fetal
hypoxia and acidosis? (3 minute uterine incision-to-delivery time, or fast delivery following
GA)
2. Spinal headache: The patient has diploplia and a severe headache. Would you give
D40 epidural patch if the mother had a fever?
312 The Essential Oral Board Review

Obstetrics VI (Session 1)

You are asked to consult on a 24 year old G2Ab1 female who is 29 weeks pregnant and develops
painless vaginal bleeding during labor.

HPI: Pt had no medical care during her pregnancy. During the last trimester, she had
increased dyspnea climbing 2 flights of stairs.

PMI: IV drug abuser.

PE: P 120, BP 96/56, R 28, T 37.5°C. She is lying flat in bed, complaining of shortness of
breath. Her airway appears normal. Jugular venous distention can be seen. There are
numerous needle tracks on her arms. Pedal edema is present. Her lungs are clear to
auscultation. Heart tones are diminished. There is slight hepatomegaly.

CXR: Cardiomegaly and a prominent main pulmonary artery.

ECG: Right axis deviation, RVH, right atrial enlargement.

Labs: Hct 40, ABG pH 7.40, pC02 43, p02 78 on room air. PT/PTT 10/20.

The obstetricians are considering a cesarean section vs a vaginal examination.


/ftey*

/Hi/ 3 I J The Essenlial Oral Board Review

fws'
msj
Obstetrics VI (Session 2)

JO
rwv You are asked to consult on a 24 year old G2Ab1 female with a 29 week pregnancy develops
X) painless vaginal bleeding during labor. She is lying flat in bed, has needle tracks on her arms,
j£v jugular cardiomegaly
™ shows venous dissension,
and aand is short main
prominent of breath. P 120,artery.
pulmonary BP 96/56,
EKG Rshows
28, Tright
37.5°C,
axis Hct 40. CXR
deviation,
JQ RVH, right atrial enlargement.
/»/

i f M

/0ff

/ fl y

/f»™)
314 The Essential Oral Board Review

Obstetrics VI (Questions)

You are asked to consult on a 24 year old G2Ab1 female with a 29 week pregnancy develops
painless vaginal bleeding during labor. She is lying flat in bed, has needle tracks on her arms,
jugular venous dissension, and is short of breath. P 120, BP 96/56, R 28, T 37.5°C, Hct 40. CXR
shows cardiomegaly and a prominent main pulmonary artery. EKG shows right axis deviation,
RVH, right atrial enlargement.
A. Preoperative Evaluation
1. Drug abuse: What would you do about her suspected drug abuse? (H/P targeting CV
system, urine drug screen, HIV, alert social services and pediatrics, consider withdrawal)
2. Placenta previa: Is this placenta previa until proven otherwise? Can you make the
diagnosis at this gestational age? (26 wks) What is it? Types? (low-lying or marginal,
partial, complete) Risk factors? (multip, adv age, c/s, uterine scar) Would you
recommend delivery or tocolytic therapy? Mg or ritodrine? (Mg more hypotension in ewes)
Is vaginal delivery an option? (low-lying only) How would you distinguish this from other
causes of antepartum bleeding (abruptio - less bleed, rupture/dehiscence - loss of
contractions and FHR)
3. Pulmonary HTN: Do you think her symptoms can be explained by the supine
hypotension syndrome? What else could it be? (CHF, P. HTN, PE, AFE) Does her
history of drug abuse affect your DDx? What is the significance of the EKG and CXR
findings? How do you make the diagnosis of pulmonary HTN?
B. Intraoperative Management
1. Double setup: The obstetricians request a double setup. What is it? What is your role?
How will you prepare?
2. Monitors: The obstetrician just wants you to be in the hospital for the exam. Agree?
Would you place an AL for the vaginal exam? A central line? What would you expect the
CVP to show? What wave would you see with pulmonary HTN? (cannon a wave or large
c wave from TR) What PAC findings would be consistent with primary pulmonary HTN?
(PAP>30/15, mean PAP>25, with normal PCWP)
3. Induction: Severe bleeding occurs. The surgeon tells you to put her to sleep for a stat
C/S. Is a spinal appropriate? An epidural? Is ketamine the drug of choice for GA?
4. Pulmonary HTN: Which is more important with pulmonary HTN, maintaining preload or
afterload? What are your goals for managing a pt with pulmonary HTN (avoid: marked
decreases in preload, pulmonary HTN, drops in SVR since CO fixed, and decreases in RV
contractility)
5. Hemorrhage: How would you treat the bleeding? The obstetrician says to administer
FFP. Do you agree for this patient? Do coagulopathies occur as they do for abruptios?
6. Acreta: With removal of the placenta, more bleeding occurs. Why? (acreta - into
myometrium, increta - invade, percreta - penetrate) Rx acreta? (immediate hyst unless
partial or focal)
C. Postoperative Care
1. Hemorrhage: Brisk bleeding continues. What could you recommend to stop it? Is
ligation of the hypogastric artery an option? (one of 2 branches of common iliac, posterior
division supplies fascia, viscera, pelvic muscles, anal areas) Hysterectomy? (acreta,
rupture, large uterine incision, abruptio with DIC, chorio, leiomyomata) Now is there a risk
for DIC? (yes)
2. Needle stick: Your colleague sticks himself with a hollow point needle.
Recommendations? The patient is HIV positive by the ELISA assay. What would you do?
Would you recommend AZT?
316 The Essential Oral Board Review

Obstetrics VII (Session 1)

The obstetricians request epidural catheter placement for a 34 year old, 70 kg, 5'6" female who is
34 weeks pregnant in active labor.

HPI: Pt began having contractions 12 hours ago, and is now 3 cm dilated. She does not feel
labor pains.

PMI: T6 paraplegia secondary to falling down the stairs at 16 years of age. She has no lower
body sensation. She also has a history of asthma, for which she was hospitalization 3
months ago requiring intubation, mechanical ventilation, and high dose steroids. She has
had several urinary tract infections and decubiti ulcers during pregnancy. Her
medications include trimethephan-sulfamethoxazole, vitamins, albuterol inhalers.

PE: P 62, BP 190/108, R 34, Temp 34.7°C. Her airway exam is normal. Lung auscultation
reveals bilateral crackles. Her heart tones are normal. She has pedal edema. She
complains of a headache.

CXR: Mild increase in vascular markings.

ECG: NSR, LAD.

Labs: Hct 26.


317 The Essential Oral Board Review

Obstetrics VII (Session 2)

i v The obstetricians request epidural catheter placement for a 34 year old, 70 kg, 5'6" female who is
k) 34 weeks pregnant in active labor. She is a T6 paraplegic and does not feel labor pains. She
r^v has medications
*^ Her a history of include
asthma, trimethephan-sulfamethoxazole,
several urinary tract infections, vitamins,
and decubiti ulcers inhalers.
albuterol during pregnancy.
P 62, BP
Q 190/108, R 34, Temp 34.7°C. She has pedal edema and complains of a headache. Hct 26.

iv
fits)

fe)

8$!f

W&l

W
W
318 The Essential Oral Board Review

Obstetrics VII (Questions)

The obstetricians request epidural catheter placement for a 34 year old, 70 kg, 5'6" female who is
34 weeks pregnant in active labor. She is a T6 paraplegic and does not feel labor pains. She
has a history of asthma, several urinary tract infections, and decubiti ulcers during pregnancy.
Her medications include trimethephan-sulfamethoxazole, vitamins, albuterol inhalers. P 62, BP
190/108, R 34, Temp 34.7°C. She has pedal edema and complains of a headache. Hct 26.

A. Preoperative Assessment
1. Autonomic hyperreflexia (AH): Does the pt have a syndrome? PIH? Do you think her
BP is always that high (or just with contractions)? With each contraction, fetal tachycardia
occurs. Why? (transplacental passage of maternal catecholamines)
2. Epidural: Would you place? Benefits? Test dose? How to R/O intrathecal? How to
determine level? Temperature changes? Safe to use epinephrine (given enhanced
sensitivity to catecholamines)? Are you worried about loss of FRC? What if the pt
became apneic during injection? Can epidural narcotics treat AH? Meperidine?
Nalbuphine (kappa)? Single shot spinal?
3. Hypotension: How would you treat? Any problems with ephedrine?
4. Trial of labor: Appropriate? (mother can't push) Her legs are contracted, OB unable to
examination baby. Can you help? (intrathecal baclofen)
5. Asthma: How is the course of asthma affected by pregnancy?
B. Intraoperative Management
1. Fetal distress: How to assess fetal well-being? What is fetal distress? Different from a
nonreassuring fetal HR pattern? Draw late decelerations. Could a prolapsed cord cause
late decelerations? What are sinusoidal waves? (distress, Rh-isoimmunization, narcotics)
What would you do?
2. C/S: A stat C/S is called. Is an epidural adequate? Swelling at insertion site, clear fluid is
leaking out. How would you induce GA? Is she at risk for aspiration? (no cough) Goals?
How about a rapid sequence induction with succinylcholine and ketamine?
3. Bronchospasm: After intubation, the PIP is high, and there is little ETC02. Your
response? B2-agonist? Inhaled or IV? Would you check the BP first? 200/120. Would
you still give IV B2-agonist? Which? DDx? Are steroids indicated?
4. Monitoring: Would you place an AL first? Due to difficulty, you are unable to place but
the AL is in the stat mode. Is that OK?
5. Fetal distress: The cord ABG reads pH 7.20. Is this OK?
6. Maintenance: What are your goals? How about halothane?
7. AH: After incision, the HR is 44. Response? The BP is 280/140 with new ST segment
depression. Response? Thiopental, nitroprusside, or nitroglycerine? Why did
bradycardia occur? What is the role of the medulla oblongata?
C. Postoperative Care
1. Headache: The pt complains of constant headache. Could it be due to a wet tap? How
would you treat it? What if there were diplopia? What if there were nuchal rigidity? At
what point would you perform a blood patch? At what point would you order a CT? At
what point would you perform an LP?
2. Postop epidural: Assume epidural had worked during case. Would you use it postop?
What solution? Why?
3. DVT: Is this pt at risk for DVT formation? How would you reduce that risk? Would you
recommend heparin?
4. Epidural hematoma: Would you D/C the catheter while the pt was still heparinized? Are
you concerned about epidural hematoma formation? Why if she's already paralyzed?
320 The Essential Oral Board Review

Obstetrics VIII (Session 1)

A 32 year old, 66 kg, 5'4" female who is 36 weeks pregnant with twins requests a labor epidural.

HPI: Pt has been on complete bedrest for the past month due to preterm labor and an
incompetent cervix. She has been taking terbutaline since then. Contractions started 10
hrs ago with rupture of her membranes. Her cervix is now 6 cm dilated. Ultrasound
reveals polyhydramnios and a possible small abruptio placenta. An pitocin infusion is
being administered.

PMI: AT3 deficiency, a history of pulmonary hypertension, and DVTs, for which she has been
taking subcutaneous heparin during her pregnancy. She becomes short of breath with
moderate exertion. Echocardiogram reveals RVH with a 45% ejection fraction.

PE: P 100, BP 120/70, R 30, Temp 37.1°C. Her airway looks normal. Her lungs are clear to
auscultation. There is no clubbing. The rest of her exam is unremarkable.

CXR: NAD.

ECG: NSR, RBBB, occasional multifocal PVCs.

Labs: Hct 29. PT/PTT = 8.9/14.7. Other labs and coagulation studies are normal.

The pt is in a birthing room and is in the care of a nursing midwife.


")
32 1 The Essential Oral Board Review

^ Obstetrics VIII (Session 2)


/IPS'

. A 32 year old, 66 kg, 5'4" female who is 36 weeks pregnant with twins requests a labor epidural.
JP She has been on complete bedrest for the past month due to preterm labor and an incompetent
jrf\
*** cervix.
She hasShe alsotaking
been has AT3 deficiency,
terbutaline a history of pulmonary
and subcutaneous heparin. P hypertension, and previous
100, BP 120/70, DVTs.
R 30, Temp
37.1°C. Hct 29, PT/PTT = 8.9/14.7. She is currently receiving pitocin.

/tlv

$&w

0m

0%{
322 The Essential Oral Board Review

Obstetrics VIII (Questions)

A 32 year old, 66 kg, 5'4" female who is 36 weeks pregnant with twins requests a labor epidural.
She has been on complete bedrest for the past month due to preterm labor and an incompetent
cervix. She also has AT3 deficiency, a history of pulmonary hypertension, and previous DVTs.
She has been taking terbutaline and subcutaneous heparin. P 100, BP 120/70, R 30, Temp
37.1°C. Hct 29, PT/PTT = 8.9/14.7. She is currently receiving pitocin.

A. Preoperative Assessment
1. AT3 deficiency: Which factors increase during pregnancy? (all except platelets, 11, and
13) Significance of AT3 deficiency? Types? (some 5% level, others qualitative).
2. Anticoagulation: Should pt be on Coumadin? (nasal hypoplasia, epiphyseal stippling,
MR) Does heparin cross placenta?
3. Epidural: Would you even place with pt on heparin? Should you give FFP prior to
epidural? Pros/cons? Is labor a good idea? (heparin increases IVH, best with vertex-
vertex)
B. Intraoperative Management
1. DVT: What is Virchow's triad? (vessel, hypercoaguable, stasis) Risks? (prior
thromboembolism, surgery, bedrest)
2. Sign out: You are relieved by a SRNA: What vital information would you include in your
sign out?
3. Arrest: Later, you are called into the pt's room stat. The pt is cyanotic, barely conscious,
and barely breathing. What would you do? DDx? (Spinal, LA, eclampsia, thrombus,
CVA, acute MR, embolus of air from transfusion or uterine rupture, PE, AFE, ruptured
uterus, abruptio, previa, inversion)
4. Difficult intubation: You are unable to intubate. Would you give succinylcholine? No
ETC02 is detected by colorometric device. What would do?
5. Amniotic fluid embolism (AFE): How does it typically present? (chills, sweat, anxiety,
cough, resp distress, shock, seizures, most with 1st stage) Risk factors? (abruption in
50%, previa, 2nd trimester Ab with NS/PG/urea, hysterectomy, just after delivery, 32 hrs
postpartum, ?oxytocin) Dx? (wheezing, pulmonary edema, large RV by CXR, elevated
CVP, DIC) Rx? (CPR, volume, coagulopathy, methylergonovine) Dopamine or
isoproterenol? Pathogenesis? (amniotomy, laceration or endocervical or uterine vessels,
pressure gradient) Complications? (arrest, DIC, atony)
6. Amniotic fluid: Does toxicity depend on contents? (particulate matter, meconium,
thomboplastin, trophoblasts)
7. Pulmonary HTN/RV failure: Which ventricle concerns you the most? How does RV
ischemia differ from LV ischemia? How does the pulmonary HTN affect your
management?
8. C/S: Resuscitation unsuccessful. Now what? C/S called stat.
9. DIC: Oozing from IV sites. Rx? Heparin? Will it work with AT3 deficiency?
C. Postoperative Care
1. Twin delivery: Anesthetic considerations? (mom: PIH, anemia, premature labor,
prolonged labor, bleeding, aortocaval compression, FRC, maternal mortality; baby:
transfusion, prematurity, congenital, prolapse cord, depression of B; anesthesiologist:
version/extraction, breech, C/S, forceps). Had VD have proceeded what would you have
done if the uterus contracts after delivery of first twin?
2. Uterine atony: After delivery, the uterus continues to bleed. What would you do? Side
effects of methergine and PGF2-a?
3. PE: How to Dx DVT? (venography, CXR, hemidiaphram, effusion, Doppler, impedance
plethysmography, 1-125 fibrinogen scan, Tc-99m venography, V/Q scan) V/Q scan shows
high probability for PE. How does it cause harm? (block many small vessels or one big
vessel-RLL) Would you start TPA? How does it work? (unlike urokinase and
streptokinase, which activate plasminogen in circulation, it only activates plasminogen
bound to fibrin, so no disseminated lysis)

Potrebbero piacerti anche